Download as pdf or txt
Download as pdf or txt
You are on page 1of 239

Table of Contents

ENVIRONMENT ECOLOGY.......................................................................................... 3
ECOLOGY ................................................................................................................... 3
TERRESTRIAL ECOSYSTEM .................................................................................. 18
AQUATIC ECOSYSTEM ........................................................................................... 29
ENVIRONMENTAL POLLUTION AND OTHER ISSUSES .................................... 38
HEALTH ISSUES .................................................................................................. 52
EFFORTS TO CONTROL POLLUTION ................................................................ 54
RENEWABLE ENERGY AND NON RENEWABLE ENERGY ................................ 58
ENVIRONMENTAL IMPACT ASSESSMENT.......................................................... 64
BIODIVERSITY ............................................................................................................ 67
BIODIVERSITY ................................................... Error! Bookmark not defined.
ANIMAL BIODIVERSITY ......................................................................................... 70
PLANT BIODIVERSITY ........................................................................................... 77
MARINE ORGANISM\ ............................................................................................. 83
PROTECTED AREA NETWORK .............................................................................. 86
CONSERVATION EFFORT ...................................................................................... 90
RED BOOK ............................................................................................................... 101
CLIMATE CHANGE ................................................ Error! Bookmark not defined.
CLIMATE CHANGE ............................................................................................... 106
OCEAN ACIDIFICATION ....................................................................................... 109
OZONE DEPLETION.............................................................................................. 109
IMPACT OF CLIMATE CHANGE ........................................................................... 111
MITIGATION STRATEGIES ................................................................................... 112
CLIMATE CHANGE ORGANISATION .................................................................. 122
INDIA AND CLIMATE CHANGE ............................................................................127
AGRICULTURE....................................................... Error! Bookmark not defined.
AGRICULTURE ...................................................................................................... 136
CONSERVATION EFFORTS ..................................................................................... 148
NATIONAL PARKS AND SANCTUARIES ............................................................. 148
ACTS AND POLICIES ............................................................................................. 166
INSTITUTION AND MEASURES ...........................................................................176
REPORTS ................................................................................................................ 192
INTERNATIONAL EFFORTS ................................................................................ 198
ENVIRONMENTAL ORGANIZATIONS ................................................................ 216
INTERNATIONAL ENVIRONMENTAL CONVENTIONS .................................... 225
ENVIRONMENT ECOLOGY

ECOLOGY

1. An indicator species is the one whose status provides information on the


overall condition of the ecosystem and of other species in that ecosystem.
Consider the following species and the indications they show in an
ecosystem.

1) Mosses: A. Help indicate acidic soil


2) Lichens: B. Help indicate air pollution
3) Fungi: C. Help indicate old-growth forests where an abundance
coarse woody debris exists

Select the correct match using the codes given below.

a) 1A, 2B, 3C
b) 1C, 2B, 3A
c) 1B, 2C, 3A
d) 1A, 2C, 3B

Correct Answer : A
Justification:
Some of the examples of indicator species are given below.

They reflect the quality and changes in environmental conditions as well


as aspects of community composition.

Stoneflies: indicate high oxygen water - Stoneflies spend the majority of


their lives as nymphs. Many species require a high concentration of
dissolved oxygen and are found in clean swift streams with gravel or stone
bottom.

Mosses: some moss species indicate acidic soil. Delicate mosses found on
rocks and trees in cities around the world can be used to measure the
impact of atmospheric change and could prove a low- cost way to monitor
urban pollution.
The “bioindicator” responds to pollution or drought-stress by changing
shape, density or disappearing, allowing scientists to calculate
atmospheric alterations

Lichens: some species indicate low air pollution. Lichens as a group have a
worldwide distribution and grow almost on any surface, for example soil,
bark, roof tiles or stone. Because lichens get all their nutrients from the
air, many species are very sensitive to air pollution.

Fungi: Can indicate old-growth forests where an abundance of coarse


woody debris exists.
Mollusca: numerous bivalve molluscs indicate water pollution status.
Mollusca, and quite often bivalve molluscs are used as bioindicators to
monitor the health of an aquatic environment, either fresh- or seawater.

2. A phenotype usually results from the interaction of

a) Genotype and environment


b) Ecosystem and biome
c) Species and Genotype
d) Alleles and Species

Solution : A
Justification :

Concept and learning:

A phenotype results from the expression of an organism's genetic code, its


genotype, as well as the influence of environmental factors and the
interactions between the two.

For e.g. while two fishes may carry similar genes, but they exhibit different
colours on their fins due to being in different habitat.

Phenotypic variation (due to underlying heritable genetic variation) is a


fundamental prerequisite for evolution by natural selection.

It is the living organism as a whole that contributes (or not) to the next
generation, so natural selection affects the genetic structure of a
population indirectly via the contribution of phenotypes.

Without phenotypic variation, there would be no evolution by natural


selection.
The interaction between genotype and phenotype has often been
conceptualized by the following relationship:
Genotype (G) Environment (E) → Phenotype (P)

3. Eutrophication of a water body necessarily involves

1) Gradual heating of the water body


2) Inflow of nutrients in the water body
3) Calcification of bed deposits

Select the correct answer using the codes below.

a) 1 only
b) 2 and 3 only
c) 2 only
d) 1, 2 and 3
Correct Answer : C
Answer Justification :

Justification:

It is the process by which a body of water acquires a high concentration of


nutrients, especially phosphates and nitrates.
These typically promote excessive growth of algae. As the algae die and
decompose, high levels of organic matter and the decomposing organisms
deplete the water of available oxygen, causing the death of other organisms,
such as fish.
Statement 1: Heated Industrial effluents increase the temperature of the
water body and typically decrease the level of dissolved oxygen of water.
This can harm aquatic animals such as fish, amphibians and other aquatic
organisms. But, this is not necessary with Eutrophication.
Statement 2 and 3: Eutrophication is a natural, slow-aging process for a
water body, but human activity (industrial effluents for e.g.) greatly speeds
up the process.
A keystone species is a plant or animal that plays a unique and crucial role
in the way an ecosystem functions (e.g. sea star). Keystone species may be
severely affected by Eutrophication.

4. Humus is the dark organic matter that forms in the soil when plant and
animal matter decays.

Consider the following about it.

1) It contains nitrogen that is essential for plant growth and helps in nutrient
retention.
2) It contributes to moisture retention in the soil by increasing
microporosity.

Which of the above is/are correct?

a) 1 only
b) 2 only
c) Both 1 and 2
d) None

Correct Answer : C
Answer Justification :

Learning: Humus should be differentiated from decomposing organic


matter. The latter is rough- looking material and remains of the original
plant are still visible. Fully humified organic matter, on the other hand,
has a uniform dark, spongy, jelly-like appearance, and is amorphous. It
may remain like this for millennia or more.
The process of humification can occur naturally in soil, or in the
production of compost.
Humus has a characteristic black or dark brown color and is organic due
to an accumulation of organic carbon.
The rate at which raw organic matter is converted into humus promotes
(when fast) or limits (when slow) the coexistence of plants, animals, and
microbes in soil.
Effective humus and stable humus are further sources of nutrients to
microbes, the former provides a readily available supply, and the latter
acts as a longer-term storage reservoir.

5. The pyramid of energy in an ecosystem is always upright because

1) The continuous loss of energy due to metabolic activity puts limits on how
much energy is available to higher trophic levels.
2) There is loss of energy at each trophic level in being transferred to the
higher trophic level.

Which of the above is/are correct?

a) 1 only
b) 2 only
c) Both 1 and 2
d) None

Correct Answer : C
Answer Justification :

Justification & Learning:

Pyramid of energy is a graphic representation of the amount of energy


trapped per unit time and area in different trophic level of a food chain
with producers forming the base and the top carnivores at the tip.

Pyramid of energy is always upright. It is so because at each transfer about


80 - 90% of the energy available at lower trophic level is used up to
overcome its entropy and to perform metabolic activities. Only about 10%
of the energy is available to next trophic level.
6. Usually high biodiversity is witnessed in Ecotones. This is because

1) Ecotone is a junction between two or more diverse ecosystems.


2) An ecotone has the highest primary productivity amongst all ecosystems.

Which of the above is/are correct?

a) 1 only
b) 2 only
c) Both 1 and 2
d) None

Correct Answer : A
Answer Justification :

Justification: Because an ecotone is the zone in which two communities


integrate, many different forms of life have to live together and compete
for space. Therefore, an ecotone can create a diverse ecosystem.

So, the ecotone contains not only species common to the communities on
both sides; it may also include a number of highly adaptable species that
tend to colonize such transitional areas Ecotones may display edge effect
where the population density of some species is more in ecotone than
adjoining communities.
If different species can survive in both communities of the two biomes,
then the ecotone is considered to have species richness that may be
greater than adjoining communities.

Statement 2: It is not necessarily true. For e.g. an ecotone with desert and
lake junction will not have a higher productivity than an estuary
ecosystem.

7. With reference to a Biotope, consider the following statements.

1) It is an ecological area that is usually larger than an ecosystem.


2) It is a common practice to isolate biotopes from each other for niche
biodiversity propagation.

Which of the above is/are correct?


a) 1 only
b) 2 only
c) Both 1 and 2
d) None

Correct Answer : D
Answer Justification :

Justification: Statement 1: It is an ecological area that supports a


particular range of biological communities.
Biotope is almost synonymous with the term habitat.
A biotope is generally not considered to be a large-scale phenomenon. For
example, a biotope might be a neighbouring park, a back garden, even
potted plants or a fish tank on a porch.
In other words, the biotope is not a macroscopic but a microscopic
approach to preserving the ecosystem and biological diversity.
Statement 2: It is commonly emphasised that biotopes should not be
isolated. Instead biotopes need to be connected to each other and other
surrounding life for without these connections to life-forms such as
animals and plants, biotopes would not effectively work as a place in
which diverse organisms live.
So one of the most effective strategies for regenerating biotopes is to plan
a stretch of biotopes, not just a point where animals and plants come and
go. (Such an organic traffic course is called a corridor.
8. What do you understand by Ecological footprint?

a) Non-monetary assessment of ecosystem integrity, health or resilience


b) The way in which ecological agents reveal their preferences through
ecological activity
c) A degree of impairment to an ecosystem, which when surpassed is too
severe to allow recovery of that ecosystem
d) An index of the area of a productive ecosystem required to produce the
resources used and to assimilate the wastes produced by a defined
population

Correct Answer : D
Justification: As per the Millennium Eco Assessment it is an index of the
area of productive land and aquatic ecosystems required to produce the
resources used and to assimilate the wastes produced by a defined
population at a specified material standard of living, wherever on Earth
that land may be located.
Ecological footprint analysis is widely used around the Earth in support of
sustainability assessments.
It can be used to measure and manage the use of resources throughout the
economy and explore the sustainability of individual lifestyles, goods and
services, organizations, industry sectors, neighborhoods, cities, regions
and nations.
In 2013, the Global Footprint Network estimated the global ecological
footprint as 1.6 planet Earths. This means that, according to their
calculations, the planet's ecological services were being used 1.6 times
faster than they were being renewed.
9. Depending upon the amount of net primary productivity the various
ecosystems can be arranged in a decreasing sequence of yield. Which of
these is the correct order?

a) Tropical seasonal forest – Tropical rain forest – Temperate Grassland


– Woodland and Shrubland.
b) Tropical seasonal forest – Tropical rain forest – Woodland and
Shrubland – Temperate Grassland
c) Tropical rain forest – Temperate Grassland – Woodland and
Shrubland – Tropical seasonal forest.
d) Tropical rain forest – Tropical seasonal forest – Woodland and
Shrubland – TemperateGrassland.

Correct Answer : D
Answer Justification :

Justification: The order can be logically arrived at by elimination.


Tropical rain forests receive overall more rainfall than seasonal forests
and grasslands. So, (a) and
(b) are eliminated.
Also, a forest will certainly be more productive than grassland. So, (c) is
also eliminated. Correct answer will be (d).

10. What symbiotic relationship is exemplified by lichens?

a) Algae provides shelter, water and minerals; and fungus provides food
b) Fungus provides shelter, water and minerals; and algae provides food
c) Fungus provides minerals and bacteria provides decayed matter fungi
d) Bacteria provides protection against parasites; and algae provides food

Correct Answer : B
Answer Justification :

Justification: Lichens are composite, symbiotic organisms made up


from members of as many as three kingdoms.
The dominant partner is a fungus. Fungi are incapable of making their
own food. They usually provide for themselves as parasites or
decomposers. The lichen fungi cultivate partners that manufacture food by
photosynthesis. Sometimes the partners are algae, other times
cyanobacteria, formerly called blue-green algae. Some enterprising fungi
exploit both at once.
Lichens occur from sea level to high alpine elevations, in a very wide range
of environmental conditions, and can grow on almost any surface.
11. If there are more number of Ecotones

1) There will be more number of eco transition zones


2) There will be greater species diversity

Which of the above is/are correct?

a) 1 only
b) 2 only
c) Both 1 and 2
d) None

Correct Answer : C
Answer Justification :

Justification: A more diverse ecosystem is more stable, remember this


as the golden rule.
If there is less evenness among species, there will be greater diversity. If
there are more number of Ecotones, there will be a lot of transition zones
(e.g. ponds, grasslands etc), and there will be greater species diversity and
inter-dependence.
Finally, if less number of species dominate, it is good for the ecosystem
diversity even more.

12. Which of these are non-biodegradable materials?

1) Cotton cloth
2) Ball-point pen refills
3) Glass objects
4) Silver Foil

Select the correct answer using the codes below.

a) 1, 2 and 3 only
b) 2 and 3 only
c) 2, 3 and 4 only
d) 1 and 4 only

Correct Answer : C
Answer Justification :

Justification: The waste materials which cannot be broken down into


non-poisonous or harmless substances in nature are called non-
biodegradable waste. Examples are plastics, polythene bags, ball-point
pen refills, synthetic fibres, and glass objects, metal articles like
aluminium cans, iron nails, silver foil and radioactive wastes.

Cotton cloth, paper, woollen clothes, wood etc. are bio-degradable


13. Among the following, the largest population is likely to be found in

a) Community
b) Ecosystem
c) Biome
d) Biosphere

Correct Answer : D
Answer Justification :

Justification: In the diagram below, several levels of ecosystem have


been given.

A biome harbours a large number of species, and not a particular species.


Some of the major biomes of the world are: forest, grassland, desert and
tundra biomes.
However, a biosphere is the largest entity that subsumes ecosystem,
biomes etc. all.
14. The ‘Ecosystem approach’ relies on

1) Integrated management of land, water and living resources that promotes


conservation and sustainable use in an equitable way
2) Adopting completely organic and natural methods, bypassing
anthropogenic scientific methodologies, focused on levels of biological
organization harnessing mutual synergy

Which of the above is/are correct?

a) 1 only
b) 2 only
c) Both 1 and 2
d) None

Correct Answer : A
Answer Justification :

Justification: As described by the Conference of the Parties to the CBD,


the ecosystem approach is the primary framework for action under the
Convention.
"Ecosystem" means a dynamic complex of plant, animal and micro-
organism communities and their non-living environment interacting as a
functional unit". It is a holistic approach to conservation.
It is based on the application of appropriate scientific methodologies
focused on levels of biological organization which encompass the essential
processes, functions and interactions among organisms and their
environment. It recognizes that humans, with their cultural diversity, are
an integral component of ecosystems.

15. With reference to Ecological value, consider the following statements.

1) It is a monetary assessment of minimum requirements for ecosystem


service provision.
2) It includes assessment of ecosystem integrity, health and resilience.
3) It accrues to both humans and non-humans.

Select the correct answer using the codes below.

a) 1 only
b) 1 and 2 only
c) 1, 2 and 3
d) 2 and 3 only

Correct Answer : D
Answer Justification :

Justification: Ecological value is a non-monetary assessment of


ecosystem integrity, health, or resilience, all of which are important
indicators to determine critical thresholds and minimum requirements for
ecosystem service provision.
Essentially, it is the worth attributed to an organism, ecosystem, product,
resource or activity, in terms of benefits to the environment.
It is non-monetary because the value of Ecology is evaluated in terms of its
intrinsic value as a part of earth, and not because it provides secondary
benefits that can be capitalized by humans.
For e.g. the value of pollination by bees can hardly be measured
monetarily, and is beneficial to both humans and non-humans.

16. Consider the following statements.

Assertion (A): Native ecosystems that have undergone human-


induced disturbance are often more prone to alien invasions than
undisturbed ecosystems.
Reason (R): Ecosystems, invaded by alien species, may not have the
natural predators and competitors present in its native environment that
would normally control their populations.
In the context of the above, which of these is correct?

a) A is correct, and R is an appropriate explanation of A.


b) A is correct, but R is not an appropriate explanation of A.
c) A is correct, but R is incorrect.
d) Both A and R are incorrect.

Correct Answer : B
Answer Justification :

Justification: Invasive alien species (IAS) are species whose


introduction and/or spread outside their natural past or present
distribution threatens biological diversity.
For a species to become invasive, it must successfully out-compete native
organisms, spread through its new environment, increase in population
density and harm ecosystems in its introduced range. To summarize, for
an alien species to become invasive, it must arrive, survive and thrive.
Native ecosystems that have undergone human-induced disturbance are
often more prone to alien invasions because there is less competition from
native species. So, A is correct, but R does not explain it because it does
not take into account the context of human intervention.
For example, imported red fire ants (Solenopsis invicta Buren) are more
successful in establishing themselves in disturbed areas such as roadsides
and agricultural fields and rarely colonize intact closed forests.
Learning: Invasive alien species occur in all taxonomic groups, including
animals, plants, fungi and microorganisms, and can affect all types of
ecosystems.
Common characteristics of IAS include rapid reproduction and growth,
high dispersal ability, phenotypic plasticity (ability to adapt
physiologically to new conditions), and ability to survive on various food
types and in a wide range of environmental conditions.
A good predictor of invasiveness is whether a species has successfully or
unsuccessfully invaded elsewhere.
Islands are especially vulnerable to IAS because they are naturally isolated
from strong competitors and predators. Islands often have ecological
niches that have not been filled because of the distance from colonizing
populations, increasing the probability of successful invasions.

17. What is Land Footprint in ecology?

a) Area of land that is used to grow feed for animals within a country
b) The proportion of built-up land as against the unused land
c) The percentage of land to grow crops to the non-crop land
d) Amount of land that is needed to produce a product by an organization
or a nation

Correct Answer : D
Answer Justification :

Learning: Land footprint is a consumption-based indicator, i.e. it looks


at the resources needed to create a final product, or by an organisation or
country, wherever they are in the world.
Ecological footprint is measured using a different approach, with two key
differences to land footprint:
Ecological footprint adds together both real land use and an calculated
(but not really existing) area of forest to absorb CO2 emissions, to
incorporate part of the impacts of climate change. In contrast, land
footprint looks only at real land use, and is often used in association with
carbon footprint to cover all climate changing emissions

Ecological footprint adjusts land areas to global hectares, while land


footprint is based on real land area - or an estimate of it

18. The philosophy of Environmentalism advocates that

1) One should learn to live in harmony with the rhythms of the ecosystem
2) Humans should not manipulate the natural environment to serve their
immediate interests
3) There is no way Humans can preserve or restore the nature to its
undisturbed form

Select the correct answer using the codes below.

a) 1 and 2 only
b) 2 and 3 only
c) 1 and 3 only
d) 1 only

Correct Answer : A
Answer Justification :
Justification: S1 and 2: Environmentalists maintain that human beings
should learn to live in harmony with the rhythms of the ecosystem and not
manipulate the natural environment to serve their immediate interests.
They believe that mankind is using up and destroying natural resources to
such an extent that we will bequeath only a barren earth, poisoned rivers
and polluted air to future generations.
The roots of environmentalism can be traced back to the nineteenth
century revolt against industrialisation.
S3: It does not say that we cannot restore the environment.
It is an attempt to balance relations between humans and the various
natural systems on which they depend in such a way that all the
components are accorded a proper degree of sustainability.

19. Humus is least in which of the following soils?

a) Laterite soils
b) Black soil
c) Alluvial soil
d) Soils of temperate forests
Correct Answer : A

Answer Justification :

Learning: This is mainly due to the intense leaching of these


soils by rainfall and high temperature.

The thick brown or black substance that remains after most of the
organic litter has decomposed is called humus. Earthworms often
help mix humus with minerals in the soil.
Humus contains many useful nutrients for healthy soil. One of
the most important is nitrogen.

20. Decomposition of organic matter Its speed is determined by which


of these major factors?
1) Soil organisms
2) Temperature conditions
3) Moisture conditions
4) Quality of the organic matter

Select the correct answer using the codes below.


a) 1 and 2 only
b) 2, 3 and 4 only
c) 3 and 4 only
d) 1, 2, 3 and 4
Correct Answer : D

Answer Justification :

Justification: It is largely a biological process that occurs

naturally. High temperature and high moisture aide the process of

decomposition.

In the decomposition process, different products are released:


carbon dioxide (CO2), energy, water, plant nutrients and
resynthesized organic carbon compounds.

Learning: Successive decomposition of dead material and


modified organic matter results in the formation of a more complex
organic matter called humus.

This process is called humification. Humus affects soil


properties.

As it slowly decomposes, it colours the soil darker; increases soil


aggregation and aggregate stability; increases the CEC (the ability
to attract and retain nutrients); and contributes N, P and other
nutrients.

Soil organisms, including micro-organisms, use soil organic


matter as food. As they break down the organic matter, any
excess nutrients (N, P and S) are released into the soil in forms
that plants can use.

This release process is called mineralization.


TERRESTRIAL ECOSYSTEM

21. Mangrove forests can be found in which of the following river deltas?

1) Ganga-Brahmaputra
2) Mahanadi
3) Godavari
4) Krishna

Select the correct answer using the codes below.

a) 1 and 2 only
b) 2, 3 and 4 only
c) 1 and 4 only
d) 1, 2, 3 and 4

Correct Answer : D
Answer Justification :

Justification: In India, the mangrove forests spread over 6,740 sq. km


which is 7 per cent of the world’s mangrove forests.
They are highly developed in the Andaman and Nicobar Islands and the
Sunderbans - the largest in the world, located in the Ganges River delta in
Bangladesh and West Bengal.
Other areas of significance are the Mahanadi, the Godavari and the
Krishna deltas.
The table below shows that mangroves can be found in every coastal state
with West Bengal being the highest.
22. Estuaries are among the most productive ecosystems in the world. What
could be a possible reason?

1) Some estuaries forms an ecotone of freshwater and salty seawater.


2) Estuaries are not subject to disturbing forces like tides and waves.

Which of the above is/are correct?

a) 1 only
b) 2 only
c) Both 1 and 2
d) None

Correct Answer : A
Answer Justification :

Justification: Statement 1: Estuaries and their surrounding wetlands are


bodies of water usually found where rivers meet the sea.
Estuaries are home to unique plant and animal communities that have
adapted to brackish water—a mixture of fresh water draining from the
land and salty seawater.
An ecotone is a transition area between two biomes. It is where two
communities meet and integrate.
It has some of the characteristics of each bordering biological community
and often contains species not found in the overlapping communities.
For e.g. the mangrove forests represent an ecotone between marine and
terrestrial ecosystem. Other examples are grassland (between forest and
desert.
Statement 2: They are subject both to marine influences—such as tides,
waves, and the influx of saline water—and to riverine influences—such as
flows of fresh water and sediment.
The inflows of both sea water and fresh water provide high levels of
nutrients both in the water column and in sediment, making estuaries
among the most productive natural habitats in the world.
The banks of many estuaries are amongst the most heavily populated
areas of the world

23. Kelp forests are underwater areas with a high density of

a) Brown algae seaweed


b) Coral reefs
c) Mangroves
d) Hydrothermal vents with vegetation

Correct Answer : A
Answer Justification :

Learning: Kelps are large brown algae seaweeds. Kelp grows in


"underwater forests" (kelp forests) in shallow oceans. They are recognized
as one of the most productive and dynamic ecosystems on Earth.
Frequently considered an ecosystem engineer, kelp provides a physical
substrate and habitat for kelp forest communities
For e.g. along the Norwegian coast these forests cover 5800 km2, and a
wide range of sea life uses kelp forests for protection or food, including
fish, particularly rockfish, and many invertebrates.
However, the influence of humans has often contributed to kelp forest
degradation. Of particular concern are the effects of overfishing nearshore
ecosystems, which can release herbivores from their normal population
regulation and result in the overgrazing of kelp and other algae.
Charles Darwin once said, “"I can only compare these great aquatic
forests...with the terrestrial ones in the intertropical regions. Yet if in any
country a forest was destroyed, I do not believe so nearly so many species
of animals would perish as would here, from the destruction of kelp.”
24. Which of the following can be best characterized as a “biome”?

a) Tundra region
b) Lake Baikal
c) Western Himalayan forests
d) Ganges River

Correct Answer : A
Answer Justification :

Justification: Biome is a community of flora and fauna that covers a


large geographical area. In the diagram below, several levels of ecosystem
have been given

A biome may or may not have a collection of endemic species. It is not a


meeting boundary, and distinct from small ecosystems or communities.

Some of the major biomes of the world are: forest, grassland, desert and
tundra biomes
25. In terms of self sustenance and endurance, which among the following is
the most stable ecosystem?

a) Forest
b) Desert
c) Ocean
d) Mountains

Correct Answer : C
Answer Justification :

Ocean is a self-sustaining ecosystem and covers nearly 2/3rd of earth’s


area.

It is the oldest ecosystem


It is least affected by natural calamities.

It is a highly diverse (from phytoplankton to sea grass to whales) and


productive ecosystem and contains earliest evolutionary life forms.

The evolutionary history has also been more stable than in terrestrial
environments.

26. With reference to Peatlands, consider the following statements.

1) Peatlands are wetlands that contain mixture of decomposed organic


material.
2) Peatlands often lack oxygen as they are partially submerged in layer of
water.
3) If they are drained, their high carbon content makes them vulnerable to
incineration.

Select the correct answer using the codes below.

a) 1 only
b) 2 and 3 only
c) 1 and 3 only
d) 1, 2 and 3

Correct Answer : D
Answer Justification :

Justification: The complex biodiversity of peatlands means they are


home to variety of species. Their high carbon content makes them
uniquely vulnerable to incineration if they are drained.
They are globally important carbon store. The unregulated exploitation of
peatlands can potentially
be detrimental to environment and to climate, as it could release carbon
emissions that have been locked in for millennia.
Learning and Context: The Brazzaville declaration was signed to
promote better management and conservation world’s largest tropical
peatlands-Cuvette Centrale region in Congo Basin from unregulated land
use and prevent its drainage and degradation.
It was signed jointly by Democratic Republic of Congo (DRC), Republic of
Congo and Indonesia on the sidelines of Third Partners Meeting of Global
Peatlands Initiative held in Brazzaville, Republic of Congo.
The Cuvette Centrale region in Congo Basin is world’s largest natural
tropical peatlands, which are about size of England. It stores three years
equivalent of global greenhouse gas emissions

27. The Oak trees of the Himalayan region are a concern to the region because

1) They are exotic species.


2) Their decay produces alkaline compounds that disturb the natural pH of
forest soil.
3) They introduction has reduced the population of keystone species in the
region.
4) They soak a large amount of groundwater.

Select the correct answer using the codes below.

a) 4 only
b) 1 and 3 only
c) 1, 2 and 4 only
d) 2 and 3 only

Correct Answer : A
Answer Justification :

Justification: Statement 1: Exotic means introduced from outside. But,


these Oka trees are native to the region.
Statement 2: Decay of organic matter in this case would yield acidic
products, not alkaline.
Statement 3: Based on statement 1, statement 3 is also wrong, because
native trees help maintain ecosystem balance rather than hurting keystone
species.
Statement 4: Though they soak nearly 60% of groundwater of the
region.

28. Sandalwood and mulberry trees together can be found in

a) Tropical deciduous forests


b) Tropical evergreen forests
c) Montane forests
d) Temperate evergreen forests

Correct Answer : A
Answer Justification :
Learning: These forests are found in areas where the rainfall is between
100cm to 200 cm.
They are found in the states of Jharkhand, West Orissa and Chhattisgarh.
Moist Deciduous Forests are also found on the eastern slopes of the
Western Ghats.
Teak trees are the most dominant species of trees found in these forests.
Bamboos, sal, shisham, sandalwood, khair, kusum, arjun, mulberry are
some of the other commercially important species found here.

29. Desert plants have which of the following adaptations for survival?

1) Stomata arranged in deep pits to minimise water loss through


transpiration
2) Special photosynthetic pathway enabling their stomata to remain closed
during day time
3) Roots performing photosynthetic functions

Select the correct answer using the codes below.

a) 1 and 2 only
b) 2 and 3 only
c) 1 only
d) 1, 2 and 3

Correct Answer : A
Answer Justification :

Justification: Many desert plants have a thick cuticle on their leaf


surfaces and have their stomata arranged in deep pits to minimise water
loss through transpiration. So, 1 is correct.
They also have a special photosynthetic pathway (CAM) that enables their
stomata to remain closed during day time. So, 1 is correct.
Some desert plants like Opuntia, have no leaves – they are reduced to
spines–and the photosynthetic function is taken over by the flattened
stems. So, 3 is correct.

30. Consider the following statements. Sacred groves

1) Are pristine stretches of forests that must be registered under


National parks
2) Have been left untouched by the local tribals as any interference
with them is banned by the local community

Which of the above is/are correct?


a) 1 only
b) 2 only
c) Both 1 and 2
d) None
Correct Answer : B

Answer Justification :

Learning: Nature worship is an age old tribal belief based on the


premise that all creations of nature have to be protected.

Such beliefs have preserved several virgin forests in pristine


form called Sacred Groves (the forests of God and Goddesses).

These patches of forest or parts of large forests have been left


untouched by the local people and any interference with them is
banned.

Certain societies revere a particular tree which they have


preserved from time immemorial.

For e.g. the Mundas and the Santhal of Chhota Nagpur region
worship mahua and kadamba trees.

31. Rhododendrons are found in their natural habitat in

a) Eastern Himalayas
b) Central India
c) Western Himalayas
d) Both (a) and (c)
Correct Answer : D

Answer Justification :

Learning: Rhododendron is the state flower of J&K, Himachal


Pradesh, Nagaland; state tree of Sikkim and state tree of the
Uttarakhand.

In Himachal Pradesh, rhododendron flowers have been used for


some time to make popular fruit and flower wines.

Most species of Rhododendron have brightly coloured flowers


which bloom from late winter through to early summer.
32. Hardwood trees like rosewood, ebony, mahogany are common in

a) Tropical evergreen forests


b) Tropical deciduous forests
c) Temperate Evergreen Forests
d) Mediterranean Vegetation
Correct Answer : A

Answer Justification :

Justification: Option B: Hardwood trees found in deciduous


forests are sal, teak, neem and shisham.

Option C: The temperate evergreen forests comprise both hard


and soft wood trees like oak, pine, eucalyptus, etc.

33. Mediterranean trees adapt themselves to dry summers with the


help of their thick barks and wax coated leaves. These help them
to

a) Reduce transpiration
b) Increase rate of water flow from roots to shoots
c) Cut down the need for photosynthesis
d) Increase the area of the leaves to step up glucose intake
Correct Answer : A

Answer Justification :

Learning: Transpiration is the loss of water from the leaves


through stomata.

This creates a suction pull in plants that creates demand for


more water from the roots.

If transpiration is high due to large area of large pores of the


leaves, water demand for the plant is likely to be high, which would
not work for a climate like Mediterranean. So, waxy leaves and
thick barks reduce transpiration and thus water need.

34. Consider the following about the Velds.

1) Velds have a mild climate due to the influence of the Pacific


Ocean.
2) Velds receive rainfall mainly in the winter months due to the cold
ocean currents washing its shores.
Which of the above is/are correct?
a) 1 only
b) 2 only
c) Both 1 and 2
d) None
Correct Answer : D

Answer Justification :

Justification: Statement 1: This is due to the influence from


Indian ocean.

Winters are cold and dry. Temperatures vary between 5°C and 10°C
and July is the coldest month. Summers are short and warm.

Statement 2: The velds receive rainfall mainly in the summer


months from November to February. This is mainly because of the
warm ocean currents that wash the shores of the velds. If the
rainfall is scanty in the winter months from June till August,
drought may occur.

Learning: The velds have rich reserve of minerals. Iron and steel
industry has developed where coal and iron are present. Gold and
diamond mining are major occupations of people of this
region.Johannesburg is known for being the gold capital of the
world. Kimberley is famous for its diamond mines.

35. Mosses and lichens together are most likely to be found in which
climatic zone?

a) Tropical
b) Temperate
c) Tundra
d) Desert
Correct Answer : C

Answer Justification :

Learning: Tundra is the global biome that consists of the treeless


regions in the north (Arctic tundra) and high mountains (alpine
tundra).

The vegetation of tundra is low growing, and consists mainly of


sedges, grasses, dwarf shrubs, wildflowers, mosses, and lichens.
Mosses and lichens grow during the very short summer.

36. Which of these is INCORRECT about Temperate Deciduous


Forests?

a) It can be found in the coastal regions of Western Europe.


b) Oak, ash and beech trees could be found here.
c) Pheasants and monals could be found in these forests.
d) None of the above
Correct Answer : D

Answer Justification :

Learning: As we go towards higher latitudes, there are more


temperate deciduous forests than tropical forests.

These are found in the north-eastern part of USA, China, New


Zealand, Chile and also found in the coastal regions of Western
Europe. They shed their leaves in the dry season.

The common trees are oak, ash, beech, etc. Deer, foxes, wolves are
the animals commonly found. Birds like pheasants, monals are
also found here.

37. The boundaries of different biomes on land are determined


mainly by

a) Climate
b) Political boundaries
c) Drainage patterns
d) Altitude
Correct Answer : A

Answer Justification :

Justification: Ecosystems are of two major types: terrestrial


and aquatic. Terrestrial ecosystem can be further be classified
into ‘biomes’.

A biome is a plant and animal community that covers a large


geographical area.
Therefore, a biome can be defined as the total assemblage of
plant and animal species interacting within specific conditions.
These include rainfall, temperature, humidity and soil
conditions.

Some use broad classifications and count as few as six biomes.


These are forest, grassland, freshwater, marine, desert, and
tundra.

AQUATIC ECOSYSTEM

38. The Ocean/Sea that has the most coral species is?

a) Pacific Ocean
b) Atlantic Ocean
c) Indian Ocean
d) Mediterranean Sea

Correct Answer : A
Answer Justification :

Learning: Reef-building corals are restricted in their geographic


distribution by factors such as the temperature and the salinity (salt
content) of the water. The water must also be clear to permit high light
penetration.
Because of these environmental restrictions, reefs generally are confined
to tropical and semitropical waters. The diversity of reef corals (the
number of species), decreases in higher latitudes up to about 30° north
and south, beyond which reef corals are usually not found.
Generally, there are about twice as many coral species in Pacific Ocean
reefs as in Atlantic Ocean reefs.

39. With reference to Corals, consider the following statements.

1) Corals are marine vertebrate animals.


2) Each individual coral animal is a polyp.
3) Corals are not found at either of the poles of the earth.
4) All corals have a rock-like calcareous skeleton.

Select the correct answer using the codes below.

a) 1, 3 and 4 only
b) 2 only
c) 2 and 4 only
d) 1, 2, 3 and 4

Correct Answer : B
Answer Justification :
Justification: Statement 1: Corals are invertebrate animals belonging to a
large group of colourful and fascinating animals called Cnidaria. Other
animals in this group that you may have seen in rock pools or on the beach
include jelly fish and sea anemones.
Statement 2: Each individual coral animal is called a polyp, and most live
in groups of hundreds to thousands of genetically identical polyps that
form a 'colony'. The colony is formed by a process called budding, which is
where the original polyp literally grows copies of itself.
Over the course of many years, stony coral polyps can create massive reef
structures. Reefs form when polyps secrete skeletons of calcium carbonate
(CaCO3).
Statement 3 and 4: Coral are generally classified as either "hard coral" or
"soft coral". There are around 800 known species of hard coral, also
known as the 'reef building' corals.
Soft corals, which include seas fans, sea feathers and sea whips, don't have
the rock-like calcareous skeleton like the others, instead they grow wood-
like cores for support and fleshy rinds for protection.
Soft corals also live in colonies, that often resemble brightly coloured
plants or trees, and are easy to tell apart from hard corals as their polyps
have tentacles that occur in numerals of 8, and have a distinctive feathery
appearance.
Statement 4: It is true that Corals are usually found in warm waters absent
at the poles, but Soft corals are found in oceans from the equator to the
north and south poles, generally in caves or ledges.

40. Wetlands are the vital link between land and water. What is/are the
importance of wetlands?

1) They are among the most productive ecosystems in the world.


2) They protect shores from wave action and reduce the impacts of floods.
3) They absorb pollutants and improve water quality.
4) Many species of birds and mammals rely on wetlands for food, water and
shelter.

Select the correct answer using the codes below.

a) 1 and 2 only
b) 2, 3 and 4 only
c) 1, 3 and 4 only
d) 1, 2, 3 and 4

Correct Answer : D

Answer Justification :

Justification: Wetlands play an integral role in the ecology of the


watershed. The combination of shallow water, high levels of nutrients and
primary productivity is ideal for the development of organisms that form
the base of the food web and feed many species of fish, amphibians,
shellfish and insects.

Wetlands are among the most productive ecosystems in the world,


comparable to rain forests and coral reefs. An immense variety of species
of microbes, plants, insects, amphibians, reptiles, birds, fish and
mammals can be part of a wetland ecosystem.

Wetlands are a critical part of our natural environment. They protect our
shores from wave action (because they form a critical link between land
and water), reduce the impacts of floods, absorb pollutants and improve
water quality.

They provide habitat for animals and plants and many contain a wide
diversity of life, supporting plants and animals that are found nowhere
else.

Wetlands provide an important range of environmental, social and


economic services. Many wetlands are areas of great natural beauty and
many are important to Aboriginal people.

Wetlands function as natural sponges that trap and slowly release surface
water, rain, snowmelt, groundwater and flood waters.

Wetlands also provide important benefits for industry. For example, they
form nurseries for fish and other freshwater and marine life and are
critical to Australia's commercial and recreational fishing industries.

41. Arrange the following marine organisms in the order of increasing


hierarchies in the food chain?

a) Dinoflagelletes = Diatoms < Copepods < Ocean Sunfish


b) Dinoflagelletes < Copepods = Diatoms < Ocean Sunfish
c) Diatoms < Dinoflagelletes < Copepods = Ocean Sunfish
d) Diatoms < Dinoflagelletes < Ocean Sunfish < Copepods

Correct Answer : A
Answer Justification :

Justification:
Learning: Apart from the above food web, Krills are considered an
important trophic level connection – near the bottom of the food chain –
because they feed on phytoplankton and to a lesser extent zooplankton,
converting these into a form suitable for many larger animals for whom
krill makes up the largest part of their diet.
42. Consider the following statements. Coral reefs

1) Break the power of the waves during storms, cyclones and tsunamis
2) Form the spawning and nursery grounds for several fish species
3) Are being used in the treatment of cancer

Select the correct answer using the codes below.

a) 1 only
b) 1 and 2 only
c) 2 and 3 only
d) 1, 2 and 3

Correct Answer : D
Answer Justification :

Justification: Healthy coral reefs are among the most biologically


diverse and economically valuable ecosystems on earth, providing
valuable and vital ecosystem services.
Coral ecosystems are a source of food for millions; protect coastlines from
storms and erosion; provide habitat, spawning and nursery grounds for
economically important fish species; provide jobs and income to local
economies from fishing, recreation, and tourism; are a source of new
medicines, and are hotspots of marine biodiversity.
Statement 1: By helping to prevent coastal erosion, flooding, and loss of
property on the shore, the reefs save billions of dollars each year in terms
of reduced insurance and reconstruction costs and reduced need to build
costly coastal defences.
Statement 2: They form the nurseries for about a quarter of the ocean's
fish, and thus provide revenue for local communities as well as national
and international fishing fleets.
Statement 3: A new class of proteins capable of blocking the HIV virus
from penetrating T-cells has been discovered. The proteins have been
found in a coral from Australia's northern coast. Coral reefs are also being
used in treatment of cancer.

43. Which of the following will NOT be characterized as a Wetland in India?

a) Sundarbans delta
b) Reservoirs of the Deccan Plateau
c) Gulf of Kachchh
d) Khasi and Jaintia region

Correct Answer : D
Answer Justification :

Justification: Option D talks about hill tracts in NE India. They aren’t


wetlands.
Learning: A wetland is a body that is saturated with water.
The country’s wetlands have been grouped into eight categories, viz.
the reservoirs of the Deccan Plateau in the south together with the lagoons
and other wetlands of the southern west coast;
the vast saline expanses of Rajasthan, Gujarat and the Gulf of Kachchh;
freshwater lakes and reservoirs from Gujarat eastwards through Rajasthan
(Keoladeo National Park) and Madhya Pradesh;
the delta wetlands and lagoons of India’s east coast (Chilika Lake); (v) the
freshwater marshes of the Gangetic Plain;
the floodplains of the Brahmaputra; the marshes and swamps in the hills
of northeast India and the Himalayan foothills;
the lakes and rivers of the montane region of Kashmir and Ladakh; and
the mangrove forest and other wetlands of the island arcs of the Andaman
and Nicobar Islands.
44. How does silting of water bodies affect aquatic life?

1) It results into increased rates of egg survival causing population boom in


water bodies.
2) Coral polyps grow more favourably in silted and shallow water bodies.
3) Silt acts as a vehicle for certain pesticides into water bodies which
adversely affect aquatic life.

Select the correct answer using the codes below.

a) 1 only
b) 3 only
c) 2 and 3 only
d) 1, 2 and 3

Correct Answer : B
Answer Justification :

Justification: Statement 1: It affects the spawning ground of fisheries


and their egg laying patterns. As bed gravel become compacted, space for
eggs is taken up by silt that considerably affects egg survival rates in the
water.
Statement 2: Hard bottom communities like corals and mussel banks
(including oysters) are more sensitive to siltation. Siltation adversely
affects coral population.
Statement 3: For e.g. in rural areas the erosion source is typically soil
degradation due to intensive or inadequate agricultural practices, leading
to soil erosion, especially in fine-grained soils such as loess. The result will
be an increased amount of silt and clay in the water bodies that drain the
area often adding unwanted fertilizers to these bodies.

45. The world’s biggest single structure made by living organisms which is
also a World Heritage site is

a) Lord Howe Island Group


b) The Sundarbans
c) Tropical Rainforest Heritage of Sumatra
d) The Great Barrier Reef

Correct Answer : D
Answer Justification :

Learning: The Great Barrier Reef is the world’s largest coral reef system
composed of over 2,900 individual reefs and 900 islands stretching for
over 2,300 kilometres over an area of approximately 344,400 square
kilometres.
The reef is located in the Coral Sea, off the coast of Queensland, Australia.
The Great Barrier Reef can be seen from outer space and is the world’s
biggest single structure made by living organisms.
This reef structure is composed of and built by billions of tiny organisms,
known as coral polyps. It was selected as a World Heritage Site in 1981.

46. What is/are the contribution(s) of Coral Reefs to an ecosystem?

1) These support fish production by providing them with food and shelter.
2) They control the concentration of carbon dioxide in the lower atmosphere
and in the sea water.
3) They check sea waves against going towards sea coasts.
4) They have the highest primary productivity amongst all ecosystems.

Select the correct answer using the codes below.

a) 1, 3 and 4 only
b) 1, 2 and 3 only
c) 2 and 4 only
d) 1, 2, 3 and 4

Correct Answer : B
Answer Justification :

Justification: Statement 2: The excess CO2 is used for calcification of


reefs and in this way the additional CO2 is taken care of.
So, coral reefs are helpful in controlling Green House Effect as they absorb
carom dioxide.
Statement 3: They usually form the first barrier against a tsunami or
cyclone or sea waves by blocking the force of the wave.
Statement 4: They have the highest primary productivity WITHIN the sea,
not outside. Amongst all, estuaries are the one with the highest primary
productivity.

47. Tamil Nadu govt in collaboration with IIT Madras has been restoring Vaan
Island in Gulf of Mannar by deploying

a) Kelp forests
b) Dune barriers
c) Artificial reefs
d) Coarse seabed raisins

Correct Answer : C
Answer Justification :

Justification: Vaan Island is located 2 km off the Tuticorin coast, Tamil


Nadu. It is one of 21 uninhabited islands in the Gulf of Mannar.
The objective was to control the erosion of the island and to facilitate coral
rehabilitation in surrounding areas.
Coral mining was once rampant in this area, and that combined with
rising sea levels have over the years harmed the island.
It is first time in India protecting and restoring a sinking island.
The project has been funded by NAFCC of the Ministry of Environment,
Forests and Climate Change (MoEFCC).
Learning: The Gulf of Mannar Biosphere Reserve is the first marine
biosphere reserve in Asia, located in the Southern part of the Bay of
Bengal.
The reserve covers 10,500 sq. km and has 21 islands with continuous
stretches of coral reef.
Fauna present includes globally endangered dugong, several species of
dolphins and whales and three species of endangered sea turtles.
It is a part of UNESCO MAB (Man and Biosphere) programme.

48. Which of the following is most likely to survive in deep ocean waters?

a) Green Algae
b) Red Algae
c) Brown Algae
d) Yellow Algae

Correct Answer : B
Answer Justification :

Justification: Light is a limiting factor for life in deep oceans. Not all the
colour components of the visible spectrum are available for marine plants
living at different depths of the ocean.
Sunlight is broken into its seven component colours as it penetrates
through the ocean water by dispersion. Longer wavelengths cannot
penetrate deeper whereas shorter wavelengths do.
Algae use photosynthesis to manufacture food. So, the ones utilizing the
short wavelength for photosynthesis survive.
Red algae have red pigment (phycoerythrin) which carries out
photosynthesis using only short wavelength light found in greater depths.
They can be found as deep as 200 m in ocean water.

49. If patches of green are seen amidst huge bodies of water, that signals presence of
phytoplankton. A large such patch would signify that the

a) Primary productivity of that water region is high.


b) The region is oxygen deprived.
c) Prevailing water is unusually hot
d) No waves or currents are found in its vicinity.
Correct Answer : A

Answer Justification :

Justification & Learning: Phytoplankton, also known as microalgae, are


similar to terrestrial plants in that they contain chlorophyll and require
sunlight in order to live and grow.

Most phytoplankton are buoyant and float in the upper part of the ocean, where
sunlight penetrates the water.

Phytoplankton also require inorganic nutrients such as nitrates, phosphates, and


sulfur which they convert into proteins, fats, and carbohydrates.

Phytoplankton is the base of several aquatic food webs. In a balanced ecosystem,


they provide food for a wide range of sea creatures including whales, shrimp,
snails, and jellyfish.

50. It is the largest coastal lagoon in India and the largest wintering ground for
migratory birds on the Indian sub-continent. A wetland of international
importance under the Ramsar Convention, it is

a) Vembanad Lake
b) Chilka Lake
c) Kolleru Lake
d) Sultanpur Lake
Correct Answer : B

Answer Justification :

Learning: In 1981, Chilika Lake was designated the first Indian wetland of
international importance under the Ramsar Convention due to its rich
biodiversity as shown by the facts that:

Over a million migratory waterfowl and shorebirds winter here. Several rare and
endangered species are found in the region.

As an estuarine lagoon, it supports a unique assemblage of marine, brackish and


freshwater species.

The lake supports fisheries that are the lifeline of the community.
The lake is of great value in preserving genetic diversity.
ENVIRONMENTAL POLLUTION AND OTHER ISSUSES

51. Consider the following statements. Furnace oil


1) refers to the Heaviest commercial fuel that can be obtained from crude
oil
2) is used as a feed stock for fertiliser manufacturing

Which of the above is/are correct?

a) 1 only
b) 2 only
c) Both 1 and 2
d) None

Correct Answer : C
Answer Justification :

Justification and Learning: Furnace oil is a dark viscous residual


product used as a fuel in different types of combustion equipment. It
conforms to IS:1593-1982 for fuel oils.
The term fuel (furnace) oil is also used in a stricter sense to refer only to
the heaviest commercial fuel that can be obtained from crude oil, i.e.,
heavier than gasoline and naphtha.
It is any liquid fuel that is burned in a furnace or boiler for the generation
of heat or used in an engine for the generation of power, except oils
having a flash point of approximately 42 °C (108 °F) and oils burned in
cotton or wool-wick burners.
It is made of long hydrocarbon chains, particularly alkanes, cycloalkanes
and aromatics. It is used in:
Steam Raising
In the process industry and thermal power stations.
Industrial Furnaces
Metallurgical furnaces, pottery and brick kilns, glass furnaces etc.
In marine engines and slow speed engines for power generation.
For drying tea leaves
In gas turbines for power generation
As a feed stock for fertiliser manufacturing
In thermic fluid heaters and hot air generators
The Supreme Court on Friday requested all States and Union Territories
to move forward towards a nationwide ban on the use of pet coke and
furnace oil to power up industries, in a bid to fight pollution.
Emissions from bunker fuel burning in ships contribute to air pollution
levels in many port cities, especially where the emissions from industry
and road traffic have been controlled.
The switch of auxiliary engines from heavy fuel oil to diesel oil at berth can
result in large emission reductions, especially for SO2 and PM.
52. Nitrogen dioxide in excess in air is harmful to

1) Photosynthesis in plants
2) Textile fibres
3) Children due to risk of acute respiratory diseases

Select the correct answer using the codes below.

a) 1 only
b) 2 and 3 only
c) 1 and 3 only
d) 1, 2 and 3

Correct Answer : D
Answer Justification :

Justification: Higher concentrations of NO2 damage the leaves of plants


and retard the rate of photosynthesis.
The irritant red haze in the traffic and congested places is due to oxides of
nitrogen.
Nitrogen dioxide is a lung irritant that can lead to an acute respiratory
disease in children. It is toxic to living tissues also.
Nitrogen dioxide is also harmful to various textile fibres as it leads to
breaking of these fibres.

53. Smog severity in a region is often aggravated by

1) Stubble burning in neighbouring agricultural areas


2) Vehicular emissions
3) Persistent cold weather conditions with sunlight

Select the correct answer using the codes below.

a) 1 only
b) 2 and 3 only
c) 2 only
d) 1, 2 and 3

Correct Answer : D
Answer Justification :

Justification: Statement 1: This is a major problem in Delhi where


neighbouring states of Haryana and Punjab often practice crop residue
burning and the PM from those fields reach Delhi aggravating the problem
of smog and pollution.
Statement 3: Cold temperatures mean that many nitrate and volatile
organic carbon particles remained in the atmosphere, whereas on hot
sunny days these would simply evaporate. This aggravates smog.
The major culprits from transportation sources are carbon monoxide
(CO), nitrogen oxides (NO and NOx), volatile organic compounds, sulfur
dioxide, and hydrocarbons. (Hydrocarbons are the main components of
petroleum fuels such as gasoline and diesel fuel.)
These molecules react with sunlight, heat, ammonia, moisture, and other
compounds to form the noxious vapors, ground level ozone, and particles
that comprise smog

54. What is/are the differences between a primary pollutant and a secondary
pollutant?

1) A primary pollutant is an air pollutant emitted directly from a source


unlike a secondary pollutant.
2) A primary pollutant has a much shorter life in the atmosphere than a
secondary pollutant.
3) A primary pollutant is characterized as less potent or harmful than a
secondary pollutant.

Select the correct answer using the codes below.

a) 1 only
b) 1 and 2 only
c) 2 and 3 only
d) 1 and 3 only

Correct Answer : A
Answer Justification :

Justification: A primary pollutant is an air pollutant emitted directly


from a source. A secondary pollutant is not directly emitted as such, but
forms when other pollutants (primary pollutants) react in the atmosphere.
Examples of a secondary pollutant include ozone, which is formed when
hydrocarbons (HC) and nitrogen oxides (NOx) combine in the presence of
sunlight; nitrogen dioxide (NO2), which is formed as nitric oxide (NO)
combines with oxygen in the air; and acid rain, which is formed when
sulfur dioxide or nitrogen oxides react with water

55. Photochemical smog consists of a mixture of air pollutants which may


include

1) Sulphur oxides
2) Tropospheric ozone
3) Volatile organic compounds
4) Peroxyacyl nitrates

Select the correct answer using the codes below.


a) 1 and 2 only
b) 2, 3 and 4 only
c) 1, 2 and 3 only
d) 1 and 4 only

Correct Answer : B
Answer Justification :

Justification: Photochemical smog is the chemical reaction of sunlight,


nitrogen oxides and volatile organic compounds in the atmosphere, which
leaves airborne particles and ground-level ozone.
This noxious mixture of air pollutants may include apart from Aldehydes
Nitrogen oxides, particularly nitric oxide and nitrogen dioxide
Peroxyacyl nitrates
Tropospheric ozone
Volatile organic compounds
The source of it is quite apparent.
The largest contributor is automobiles, while coal-fired power plants and
some other power plants also produce the necessary pollutants to facilitate
its production.
Due to its abundance in areas of warmer temperatures, photochemical
smog is most common in the summer.

56. Large scale deforestation in a forest can cause fertility of soil to decline.
Why?

1) Soil can be leached easily due to heavy rain


2) Microbial activity in soil increases significantly
3) Nutrient content of the soil changes.

Select the correct answer using the codes below.

a) 1 and 2 only
b) 2 and 3 only
c) 1 and 3 only
d) 1 only

Correct Answer : C
Answer Justification :

Explanation: Removal of the top layer of the soil during deforestation


exposes the lower, hard and rocky layers. This soil has less humus and is
less fertile. So, statement 3 is correct.
Microbial activity in soil actually decreases due to the loss of biodiversity
and the forest. Statement 2 is incorrect.
As the soil is exposed, rainwater can easily leach washing off nutrients in
run-off water. Statement 1 is correct.
Learning: Deforestation is a major cause which leads to the change in
soil properties.
Physical properties of the soil get affected by plantation and vegetation.
Trees prevent soil erosion. Fewer trees result in more soil erosion.
Gradually the fertile land gets converted into deserts. It is called
desertification.
Deforestation also leads to a decrease in the water holding capacity of the
soil.
The movement of water from the soil surface into the ground (infiltration
rate) is reduced. So, there are floods. The other properties of the soil like
nutrient content, texture, etc., also change because of deforestation.

57. Water can be considered polluted due to

1) High concentration of nutrients


2) Higher temperature
3) Presence of larvae eating fish

Select the correct answer using the codes below.

a) 1 and 2 only
b) 2 and 3 only
c) 1 and 3 only
d) None of the above

Correct Answer : A
Answer Justification :

Explanation: Freshwater fishes like mosquitofish are used to feed on


larvae to biocontrol mosquito population. Such fishes are not a sign of
water pollution. Statement 3 is incorrect.
Hot water released from plants and industries raises the average
temperature of the waterbody, adversely affecting the animals and plants
living in it. It is a form of pollution. Statement 2 is correct.
Statement 1 causes eutrophication (algal boom) in water bodies resulting
in higher consumption of oxygen and loss to aquatic life: hence, correct.

58. Among the following, maximum land in India has been degraded to
wasteland due to

a) Loss of top soil


b) Land degradation due to acidity
c) Salt panning of soil
d) Vegetal degradation with water erosion

Correct Answer : D
Answer Justification :
Major factors are loss of top-soil due to water erosion; land
degradation due to acidity; vegetal
degradation with water erosion; and gully formation.

59. Deforestation affects which of these soil properties?

1) Top soil content


2) Microbial activity
3) Nutrient content
4) Soil texture
5) Water holding capacity

Select the correct answer using the codes below.

a) 1, 2 and 3 only
b) 1, 4 and 5 only
c) 2 and 5 only
d) 1, 2, 3, 4 and 5

Correct Answer : D
Answer Justification :

Justification: Statement 1: Removal of the top layer of the soil during


deforestation exposes the lower, hard and rocky layers. This soil has less
humus and is less fertile.
Statement 2: Microbial activity in soil actually decreases due to the loss of
biodiversity and the forest.
Statement 3 and 4: As the soil is exposed, rainwater can easily leach
washing off nutrients in run-off water.
This also changes soil texture.

Statement 5: The movement of water from the soil surface into the ground
(infiltration rate) is reduced due to increased surface run off as water is
not stopped.

60. Burning coal generates fly ash which contains several toxic pollutants.
These include

1) Lead
2) Zinc
3) Arsenic
4) Plutonium
5) Cadmium

Select the correct answer using the codes below.

a) 1, 2, 3 and 5 only
b) 2, 3 and 5 only
c) 1, 4 and 5 only
d) 1, 2 and 3 only

Correct Answer : A
Answer Justification :

Learning: Burning coal, whether local or imported, generates large


quantities of fly ash containing toxic pollutants like lead, zinc, arsenic,
cadmium, sulphur, mercury and radioactive uranium/ thorium isotopes,
which adversely affect the health of the people near the power stations,
often the rural poor, whose disadvantage is worsened by these health
impacts.
Studies on people residing near coal-based power plants along the border
of Uttar Pradesh and Madhya Pradesh have revealed unsafe levels of
mercury in their blood samples, at times as high as 110 parts per billion.
Similarly, studies around a coal power plant in the Punjab have indicated
widespread radioactive contamination of the environment, impacting the
health of pregnant women and children.

61. Forest fires can cause which of the following adverse impacts on the
environment?

1) Degradation of water catchment areas resulting in loss of water


2) Change in micro climate of the area degrading living conditions
Which of the above is/are correct?

a) 1 only
b) 2 only
c) Both 1 and 2
d) None

Correct Answer : C
Answer Justification :

Justification: It can lead to:


Loss of valuable timber resources and depletion of carbon sinks
Degradation of water catchment areas resulting in loss of water
Loss Of biodiversity and extinction of plants and animals
Loss of wild life habitat and depletion of wild life
Loss of natural regeneration and reduction in forest cover and production
Global warming resulting in rising temperature
Loss of carbon sink resource and increase in percentage of CO2 in the
atmosphere
Change in micro climate of the area making it unhealthy living conditions
Soil erosion affecting productivity of soils and production
Ozone layer depletion
Health problems leading to diseases Indirect effects on agricultural
production: Loss of livelihood for the tribals as approximately 65 million
people are classified as tribals who directly depend upon
collection of non-timber forest products from the forest areas for their
livelihood.

62. The ‘Regulation of Polychlorinated Biphenyls Order, 2016’ bans import of


equipment containing polychlorinated biphenyls (PCBs). Why is there
concern about the use of PCBs?

1) PCBs are persistent organic pollutants (POPs).


2) PCBs are highly inflammable and volatile.
3) PCBs are carcinogens.

Which of the above is/are correct?

a) 1 and 2 only
b) 2 and 3 only
c) 1 only
d) 1 and 3 only

Correct Answer : D
Answer Justification :
Justification: The Union environment ministry has banned the
manufacture and import of polychlorinated biphenyls (PCBs) and directed
complete prohibition on its use in any form by the end of 2025.
What are PCBs?
PCBs are synthetic organic chemicals that were initially accepted as an
important industrial breakthrough for various usages, but were later
found to be highly toxic, leading to demands for a ban on it or reducing its
use.
They were used in the manufacture of electrical equipment, heat
exchangers and other materials.
Issues
The government’s decision is in line with the Stockholm Convention,
under which signatory countries are to prohibit or take necessary legal and
administrative measures to eliminate the production and use of PCBs.
India signed it in 2002.
PCBs are persistent organic pollutants (POPs) and are recognised by the
International Agency for Research on Cancer (IARC) as Group 1
carcinogens or cancer-causing substances.
Studies have also found that exposure to PCBs leads to serious health
problems—acute and chronic—like skin rashes, skin and fingernail
pigmentation changes, disturbances in liver function and the immune
system, irritation of the respiratory tract, depression, memory loss,
nervousness, impotence, elevated risk of cardiovascular disease,
hypertension and diabetes, liver damage and others.
As per estimates, 10,000 tonnes of PCBs have been recorded in India,
especially in the power sector. Though India never manufactured it but we
have stockpiles of it.

63. Land Degradation Neutrality (LDN) has been defined by the Parties to the
UNCCD as

a) Zero land degradation due to anthropogenic factors in a nation or a pre-


defined ecosystem
b) A state whereby the amount and quality of land resources, necessary to
support ecosystem functions and services remains stable or increases with
time
c) Control over the factors that cause large scale degradation with a
reasonable leeway for degradation that are beyond human intervention
d) The state when the amount of land degraded is compensated by
afforestation ensuring that the net carbon sink remains the same

Correct Answer : B
Answer Justification :

Learning: It is a state whereby the amount and quality of land resources,


necessary to support ecosystem functions and services and enhance food
security, remains stable or increases within specified temporal and spatial
scales and ecosystems.
To date, over 110 countries have engaged with the LDN target setting
programme and considerable progress has been made since the 2030
Agenda was adopted in 2015.
LDN represents a paradigm shift in land management policies and
practices.
It is a unique approach that counterbalances the expected loss of
productive land with the recovery of degraded areas. It squarely places the
measures to conserve, sustainably manage and restore land in the context
of land use planning.

64. Consider the following about Multi-layered plastics (MLP), recently in


news.

1) MLP is widely used in food packaging industries.


2) Some MLPs are non-recyclable and non-energy recoverable.
3) The Ministry of Environment recently notified that Multi-layered plastics
(MLP) is expected to be phased out from the country.

Select the correct answer using the codes below.

a) 1 and 2 only
b) 1, 2 and 3
c) 2 and 3 only
d) 1 and 3 only
Correct Answer : B
Answer Justification :

Justification: The ministry of environment notified an amendment to


the Plastic Waste Management (Amendment) Rules 2018, adding Multi-
layered plastics (MLP) to the list of plastics that it expects to be phased
out.
In 2016, it had added MPL to an exception list notifying that it can have
some alternate uses. But, in the recent amendment, it is phasing out the
MLPs that do no have any alternate use. MLP is the shiny plastic material
which is used to package chips, biscuit and ready-to-eat food products.
MLPs are non-recyclable, non-energy recoverable, and have no alternative
uses, and are hence a critical threat to the ecosystem.
The same amendment issued the prescription of a central system which
will carry the names of producers, brand-owners and importers. This will
allow agencies to better track and control the usage of plastics.

65. Which of the following is often included in the category of harmful E-


Waste?

1) Lead
2) Cadmium
3) Beryllium
4) Brominated flame retardants
Select the correct answer using the codes below.

a) 1 and 2 only
b) 1, 2 and 3 only
c) 3 and 4 only
d) 1, 2, 3 and 4

Correct Answer : D

Answer Justification :

Justification: Statement 1: Electronic scrap components contain


potentially harmful components such as nickel, phosphorous, lead,
cadmium, beryllium, or brominated flame retardants.

Recycling and disposal of e-waste may involve significant risk to health of


workers and great care must be taken to avoid unsafe exposure in
recycling operations and leaking of materials such as heavy metals from
landfills and incinerator ashes.

Statement 4: Brominated flame retardants (BFRs) are mixtures of man-


made chemicals that are added to a wide variety of products, including for
industrial use, to make them less flammable. They are used commonly in
plastics, textiles and electrical/electronic equipment.

66. Coastal erosion can be caused by

1) Global warming induced sea level rise


2) removal of beach sediments by wave action
3) Presence of coral reefs
4) Growth of mangroves near the seashore

Select the correct answer using the codes below.


a) 1 and 2 only
b) 2, 3 and 4 only
c) 1 and 3 only
d) 1, 2, 3 and 4
Correct Answer : A

Answer Justification :

Justification: Statement 1: According to a study, Parali I


island, one of biodiversity-rich uninhabited islands part of
Lakshadweep has disappeared due to coastal erosion and
another four such islands in Lakshadweep sea are shrinking
fast. Sea level rise is considered to be a major factor behind this.

The study has recommended of a bio protection strategy using


mangroves, in addition to the conventional physical protection
measures from coastal erosion.

Statement 2: Waves, generated by storms, wind, or fast moving


motor craft, can cause coastal erosion, which may take the form
of long-term losses of sediment and rocks, or merely the
temporary redistribution of coastal sediments; erosion in one
location may result in accretion
nearby.

Statement 3 and 4: They protect the coast from erosion because


they bear the impact of waves and tidal action.

67. Consider the following Black carbon.

1) It is formed through the incomplete combustion of fossil fuels.


2) Its particles absorb heat but are extremely short-lived in the
atmosphere in comparison to carbon dioxide.
3) If deposited on snow, it could accelerate the heating of snow.

Select the correct answer using the codes below.

a) 1 and 2 only
b) 2 and 3 only
c) 1 and 3 only
d) 1, 2 and 3
Correct Answer : C

Answer Justification :

Justification: Statement 1: It consists of pure carbon in several


linked forms formed through the incomplete combustion of fossil
fuels, biofuel, and biomass, and is emitted in both anthropogenic
and naturally occurring soot.

Statement 2: As BC particles absorb heat, they warm the


surrounding air, become lighter and rise to greater heights by a
process called self-lift and persist for longer time in the air.
Statement 3: Because BC particles strongly absorb solar and
terrestrial radiation and heats up the atmosphere it can upset the
monsoon system. If deposited on snow, it could accelerate the
heating
of snow and quicken the melting of glaciers.

68. The largest human source of carbon dioxide emissions is from

a) Industrial manufacturing
b) Combustion of fossil fuels
c) Agricultural fields
d) Clearing of forests
Correct Answer : B

Answer Justification :

Learning: Nearly 87 percent of all human-produced carbon dioxide


emissions come from the burning of fossil fuels like coal, natural gas
and oil.

The remainder results from the clearing of forests and other land use
changes (9%), as well as some industrial processes such as cement
manufacturing (4%)

Coal is the most carbon intensive fossil fuel. For every tonne of coal
burned, approximately 2.5 tonnes of CO2e are produced

69. Sunderbans delta is facing potential threats due to

1) Global warming induced sea level rise leading to coastal erosion


2) Freshwater flows from Brahmaputra and Meghna

Which of the above is/are correct?

a) 1 only
b) 2 only
c) Both 1 and 2
d) None

Correct Answer : A

Justification: Sundarben delta hosts a large population of mangorves


(that grow in salt water) – the largest tidal halophytic mangrove forest in
the world.
Statement 1: A study highlights a time series of the erosion of many
mangrove forested islands of the Indian Sunderbans from 1986 to 2012.
These islands include Sagar, Gosaba, Dulibhasani, Dalhousie, Bhangaduni
and Jambudwip.
The mean sea level rise is considered a driving factor for coastal erosion,
coastal flooding,
increase in the number of tidal creeks and losing land.
Statement 2: A critical minimal inflow of freshwater is necessary for the
luxuriant growth of
mangroves. Decrease in freshwater inflow results change in mangrove
succession where freshwater loving species of mangroves are replaced by
salt-water loving ones. So, 2 is wrong. The loss land, including mangrove
forest is mainly due to decrease in fresh water flow and sediment supply in
the western (Indian) part of the delta, and the rate of sea level rise is
higher than sediment supply. However, the eastern (Bangladesh) side of
the Sunderbans delta is gaining land because of the huge amount of
sediment and water flow from the Brahmaputra and Meghna rivers (which
is good for the Mangroves). This immediate impact of salinity will also
have negative impact on the fishing community, as commercially sought
fish species will be replaced by fish that does not have as much market
value
70. The Centre has banned the manufacture and import of
Polychlorinated Biphenyls (PCBs). This is because

a) It is a toxic pollutant.
b) It is responsible for coral bleaches across the eastern coast of
India.
c) It was being oversupplied in the market leading to price crashes
and poor producer margins.
d) It was heavily used by farmers as an additive in their NPK
fertilizers.
Correct Answer : A

Answer Justification :

Learning: PCBs are synthetic organic chemicals used in many


different products including electrical equipment, inks,
adhesives, flame-retardants, and paints.

Its production and use are now severely restricted in many


countries because of possible impacts on human health and the
environment.

The use of PCBs in any form shall be completely prohibited by


2025.
The government has directed users not to drain or discharge PCBs
directly or indirectly on land, in surface water or from effluent
treatment plants.
The government's decision is in line with the Stockholm
Convention, under which signatory countries are to prohibit or
take necessary legal and administrative measures to eliminate the
production and use of PCBs.

70. Consider the following statements.

1) Arsenic contamination of groundwater is found in West Bengal.


2) High concentrations of fluoride in ground water are common in some of
the semi-arid areas of Rajasthan.
3) The sulphate content in groundwater is high in regions like Haridwar and
Dehradun.

Select the correct answer using the codes below.

a) 1 and 2 only
b) 3 only
c) 1 and 3 only
d) 1, 2 and 3

Correct Answer : D
Answer Justification :

Justification: Arsenic pollution of ground water in West Bengal was first


reported in the early eighties. The occurrence of arsenic is mainly due to
two reasons: natural and anthropogenic.
High concentrations of fluoride in ground water are common in some of
the semi-arid areas of Rajasthan, southern Punjab, Gujarat, Karnataka,
Tamil Nadu, Madhya Pradesh, and southern Haryana. Groundwater in at
least 387 districts has high nitrate levels.

HEALTH ISSUES

71. The Global Alliance to Eliminate Lead Paint is a voluntary partnership


formed by UN Environment and the World Health Organization to
prevent exposure to lead, while promoting the phase-out of paints
containing lead. What can be the adverse effects of lead based paints?

1) Childhood lead poisoning during pregnancy can have lifelong health


impacts including learning disabilities
2) In adults, it can lead to depression, loss of appetite and even muscle pain

Which of the above is/are correct?


a) 1 only
b) 2 only
c) Both 1 and 2
d) None

Correct Answer : C
Answer Justification :

Justification: Statement 1: Paint containing lead additives poses risks to


health from poisoning and environmental contamination. Lead can have
permanent health effects on children, but also causes harm in adults.
Childhood lead poisoning, also during pregnancy, can have lifelong health
impacts including: learning disabilities, anemia, and disorders in
coordination, visual, spatial and language skills. There is no known level of
lead exposure that is considered safe for adults or children.
Statement 2: Symptoms may be different in adults and children; the main
symptoms in adults are headache, abdominal pain, memory loss, kidney
failure, male reproductive problems, and weakness, pain, or tingling in the
extremities.
Early symptoms of lead poisoning in adults are commonly nonspecific and
include depression, loss of appetite, intermittent abdominal pain, nausea,
diarrhoea, constipation, and muscle pain.

72. Why the sale of Oxytocin, despite being a natural hormone, is heavily
regulated?

1) It was used as a depressant by a large segment of population.


2) Its indiscriminate use by dairy farmers was causing irreversible hormone
damage in milch animals.
3) Consumption of food laced with Oxytocin can cause cardiac problems in
humans.
4) It is abused to speed up child birth in women.

Select the correct answer using the codes below.


a) 1 and 3 only
b) 2, 3 and 4 only
c) 2 and 4 only
d) 1, 2, 3 and 4

Correct Answer : B

Answer Justification :

Justification: Statement 1: Oxytocin has also been dubbed the love or the bliss
hormone due to its effects on behaviour. It isn’t a depressant. So, 1 is wrong.
Statement 2: The drug Oxytocin is banned in India under Prevention of Cruelty to
Animals Act and Food and Drug Adulteration Prevention Act, 1960. In
mammals, Oxytocin stimulates ejection of milk from the mammary glands. But, its
use can cause reproductive damage in animals and even their death.

Statement 3: Overdose of the drug in humans can lead to haemorrhage,


palpitation, low blood pressure and in some cases even death.

Statement 4: The release of oxytocin by the pituitary gland acts to regulate two
female reproductive functions: Childbirth and Breast-feeding. So, it is used to
speed up child birth which is wrong and should be shunned.

So, after being banned in retail markets, the Indian Drug Controller will
soon restrict manufacturing of controversial hormone drug Oxytocin to public
sector undertakings (PSUs).

EFFORTS TO CONTROL POLLUTION

73. Bioremediation may NOT be most ideal for the removal of which of the
following from a site?

a) Cadmium
b) Lead
c) Sodium Chloride
d) All of the above are toxic and may pose difficulty in degradation to
microbes.

Correct Answer : D
Answer Justification :

Learning: In a non-polluted environment, bacteria, fungi, protists, and


other microorganisms are constantly at work breaking down organic
matter, which we call as bioremediation.
If an organic pollutant such as oil contaminated this environment, some of
the microorganisms would die, while others capable of eating the organic
pollution would survive.
Bioremediation works by providing these pollution-eating organisms with
fertilizer, oxygen, and other conditions that encourage their rapid growth.
However, bioremediation may not provide a feasible strategy at sites with
high concentrations of chemicals that are toxic to most microorganisms.
These chemicals include some toxic heavy metals such as cadmium or
lead, and salts such as sodium chloride.
74. Consider the following statements about fine particulate matter (PM2.5).

1) They are visible to the naked eye causing haze in the traffic.
2) Their levels are most likely to be elevated on days with little or no wind or
air mixing.
3) They can bypass the nose and throat and can easily enter the circulatory
system.
4) In Particulate matter, the National Air Quality Index (AQI) monitors only
PM10 particles.

Select the correct answer using the codes below.

a) 1, 3 and 4 only
b) 2 and 3 only
c) 2, 3 and 4 only
d) 1, 2 and 4 only

Correct Answer : B
Answer Justification :

Justification: Statement 1: PM 2.5 is an atmospheric particulate matter


of diameter of fewer than

2.5 micrometres, which is around 3 per cent the diameter of a human hair.
It causes respiratory problems and reduces visibility. PM 2.5 particles can
only be detected with the help of an electron microscope because they are
so small.

Statement 2: This prevents their dispersion in the atmosphere and


increases concentration.

Statement 3: The circulatory system, also called the cardiovascular system


is an organ system that permits blood to circulate and transport nutrients
(such as amino acids and electrolytes), oxygen, carbon dioxide, hormones,
and blood cells to and from the cells in the body to provide nourishment.

Due to their smaller size, the PM 2.5 particles can easily bypass the nose
and throat and can easily enter the circulatory system. The particles can
also lead to cause chronic diseases such as asthma, heart attack, bronchitis
and other respiratory problems.

Statement 4: AQI disseminates information on air quality in an easily


understandable form for the general public. The measurement of air
quality is based on eight pollutants, namely,

Particulate Matter (size less than 10 µm) or (PM10),

Particulate Matter (size less than 2.5 µm) or (PM2.5),


Nitrogen Dioxide (NO2),

Sulphur Dioxide (SO2),

Carbon Monoxide (CO),

Ozone (O3),

Ammonia (NH3), and

Lead (Pb)

75. Which of the following statements about Central Pollution Control Board
(CPCB) is correct?

1) It is a statutory body.
2) It functions under the Ministry of Urban Development.
3) It is responsible for maintaining national standards under a variety of
environmental laws.
4) The ambient air quality monitoring systems in cities have been setup by
CPCB with the help of public sector undertakings.

Select the correct answer using the codes below.

a) 1, 2 and 3 only
b) 1, 2, 3 and 4
c) 1, 3 and 4 only
d) 2 and 4 only

Correct Answer : C
Answer Justification :

Justification: It is a statutory organisation under the Ministry of


Environment and Forests (MoEF).
It co-ordinates the activities of the State Pollution Control Boards by
providing technical assistance, guidance and resolving disputes among
them.
It is the apex organisation in country in the field of pollution control, as a
technical wing of MoEF.
It is responsible for maintaining national standards under a variety of
environmental laws, in consultation with zonal offices, tribal, and local
governments.
It has responsibilities to conduct monitoring of water and air quality, and
maintains monitoring data.
76. Which of the following are reason(s) for adoption of bio-toilets in Indian
Railways at a large scale?

1) They prevent erosion of railways tracks that was caused due to open
defecation from the coaches.
2) Bio-toilets do not require water and can operate in a variety of climatic
situations.

Which of the above is/are correct?

a) 1 only
b) 2 only
c) Both 1 and 2
d) None

Correct Answer : C
Answer Justification :

Justification: Statement 1: The railways currently uses flush toilets in


trains, in which human waste is dumped directly on the track. This makes
the environment unhygienic and railway stations
an eyesore, apart from the fact that the faecal matter corrodes the tracks.
Statement 2: They can be used in tropical, temperate, Himalayan Regions,
glaciers and even sea ports.
Bio-toilets are generally maintenance free and have a life of upto 50 years
at once. They do not need to be flushed. This is why they are being
installed at large scale in railways.

77. Global Urban Ambient Air Pollution Database is released by

a) International Meteorological Organization (IMO)


b) United Nations Environment Programme (UNEP)
c) UN-HABITAT
d) World Health Organization (WHO)

Correct Answer : D
Answer Justification :

Justification: WHO recently released global air pollution database in


Geneva. WHO monitored 4,300 world cities for their air pollution levels in
terms of PM 2.5 levels in the year 2016.
The PM2.5 includes pollutants like sulfate, nitrate and black carbon,
which pose the greatest risk to human health.
More than 80% of people living in urban areas that monitor air pollution
are exposed to air quality levels that exceed the World Health
Organization (WHO) limits. While all regions of the world are affected,
populations in low-income cities are the most impacted.
According to the latest urban air quality database, 98% of cities in low-
and middle income countries with more than 100 000 inhabitants do not
meet WHO air quality guidelines. However, in high-income countries, that
percentage decreases to 56%.
In the past two years, the database – now covering 3000 cities in 103
countries – has nearly doubled, with more cities measuring air pollution
levels and recognizing the associated health impacts.
As urban air quality declines, the risk of stroke, heart disease, lung cancer,
and chronic and acute respiratory diseases, including asthma, increases
for the people who live in them.
Learning: As per the database, 14 out of 15 most polluted cities in the
world are from India and the top 14 cities are from India only.
Kanpur is the most polluted city which came on top with PM 2.5
concentration of 173 micrograms per cubic metre.
Other Indian cities that registered very high levels of PM2.5 pollutants
were Kanpur, Faridabad, Gaya, Patna, Agra, Muzaffarpur, Srinagar,
Gurgaon, Jaipur, Patiala and Jodhpur followed by Ali Subah Al-Salem in
Kuwait and a few cities in China and Mongolia.
The national Capital climbed down from the fourth spot, where it
appeared in WHO 2015 data, to the sixth spot only.

RENEWABLE ENERGY AND NON RENEWABLE ENERGY

78. Consider the following statements.

1) Natural gas is often found with petroleum deposits.


2) Russia is a major producer of natural gas.
3) In India, natural gas is found only in Western and Southern
India.

Select the correct answer using the codes below.


a) 1 and 2 only
b) 2 and 3 only
c) 3 only
d) 1, 2 and 3
Correct Answer : A

Answer Justification :

Justification: Statement 1: It can be released when crude oil is


brought to the surface. It can be used as a domestic and
industrial fuel.

Statement 2: Russia, Norway, UK and the Netherlands are the major


producers of natural gas. But, very few countries in the world have
sufficient natural gas reserves of their own.
Statement 3: In India Jaisalmer, Krishna Godavari delta, Tripura and
some areas off shore in Mumbai have natural gas resources.

79. Consider the following statements. Hydrogen cyanide (HCN)

1) is a colorless and non-flammable liquid


2) is used in manufacture of polymers and pharmaceuticals
3) is a strong base with a high pH value
4) has been detected in the interstellar medium

Select the correct answer using the codes below.


a) 2 and 4 only
b) 1 and 3 only
c) 1, 2 and 4 only
d) 2, 3 and 4 only
Correct Answer : A

Answer Justification :

Justification: Statement 1: Sometimes called prussic acid, is a


chemical compound with the chemical, it is a colorless, extremely
poisonous and flammable liquid that boils slightly above room
temperature.

Statement 2: HCN is produced on an industrial scale and is a


highly valuable precursor to many chemical compounds ranging
from polymers to pharmaceuticals.

Statement 3: Hydrogen cyanide is a weak acidic that partially


ionizes in water solution to give the cyanide anion.

A solution of hydrogen cyanide in water, represented as HCN, is


called hydrocyanic acid.

HCN is obtainable from fruits that have a pit, such as cherries,


apricots, apples, and bitter almonds, from which almond oil and
flavoring are made.

Statement 4: HCN has been detected in the interstellar medium


and in the atmospheres of carbon stars.

Since then, extensive studies have probed formation and


destruction pathways of HCN in various environments and
examined its use as a tracer for a variety of astronomical species
and processes.

HCN can be observed from ground-based telescopes through a


number of atmospheric windows.

80.Which of these is the lightest?

a) Compressed Natural Gas (CNG)


b) Diesel
c) Air
d) Liquefied petroleum gases (LPGs)
Correct Answer : A

Answer Justification :

Justification: Option D: Liquefied petroleum gases are all


heavier than air and will collect in low places when not confined.

Option A: CNG is considered safer than other fuels in the event of


a spill, because natural gas is lighter than air and disperses
quickly when released.

Learning: CNG is made by compressing natural gas (which is


mainly composed of methane, CH4), to less than 1 percent of the
volume it occupies at standard atmospheric pressure.

The cost and placement of fuel storage tanks is the major barrier to
wider/quicker adoption of CNG as a fuel.

81. Tata Motors has recently unveiled India’s first Bio-CNG (bio-methane)
bus. Chemically, bio-methane is identical to natural gas, however,
natural gas is classified as a fossil fuel, whereas bio-methane is called
as a renewable source of energy. This is because

1) Nature of production process of bio-methane reduces its emissions of


greenhouse gases into the air.
2) Bio-methane is produced from fresh organic matter, unlike natural gas
which is obtained from decomposition of fossils.

Which of the above is/are correct?


a) 1 only
b) 2 only
c) Both 1 and 2
d) None
Correct Answer : C

Answer Justification :

Justification: Statement 1: Biomethane is produced by ‘anaerobic’


digestion of organic matter such as dead animal and plant material.

This gas when produced out of natural degradation process, escapes into
the atmosphere unused.

But, if produced under controlled conditions, the impact on


environment can be significantly reduced.

Statement 2: Methane is produced from thousands or millions of years


old fossil remains of organic matter that lies buried deep in the ground.

Production of fossil fuel derived methane, therefore, depends


exclusively on its natural reserves which vary greatly from one country
to another and are not available in limitless amounts.

Biomethane, on the other hand, is produced from “fresh” organic


matter which makes it a renewable source of energy that can be
produced worldwide.

Since biomethane is chemically identical to natural gas, it can be used


for the same applications as natural gas.

82. With reference to “flammable ice”, recently seen in news,


consider the following statements:

1) It consists of methane trapped within water crystals.


2) It is another name for permafrost.
3) All of its reserves are trapped in plateaus at higher latitudes.
4) India is considering it as an alternative energy source.

Select the correct answer using the codes below.


a) 1, 3 and 4 only
b) 2 and 3 only
c) 1 and 4 only
d) only
Correct Answer : A

Answer Justification :
Justification: Statement 1: Flammable ice (also known as
methane hydrate or methane clathrates) consists of methane
trapped within water crystals. It is the world’s largest natural gas
resource trapped beneath permafrost and ocean sediment where
low temperature and moderate pressure combine to trap
methane in this specific way.

Statement 2: Permafrost appears on top of it. Permafrost is frozen


chunks of ice that often contain carbon and volatile gases.

Statement 3: China has successfully produced natural gas from


methane hydrate, also known as “flammable ice”, in an
experimental project in the South China Sea (SCS).

Most of its reserves are located deep in ocean floors, especially


continental shelves.

Statement 4: India, USA, China and Canada all are looking at it


as an alternative energy source.

Learning: The methane hydrate is highly flammable and


energy-intensive fuel as one cubic metre of the compound can
releases about 160 cubic metres of gas.

It can break down into water and methane after temperature is


raised or pressure is lowered.

83. The Government is implementing an Ethanol Blended Petrol


(EBP) Programme in India. Consider the following with reference
to it.

1) The Government has introduced free market pricing mechanism


for procurement of ethanol under EBP Programme.
2) The Government has allowed procurement of ethanol produced
from cellulosic and lignocellulosic feedstock.

Which of the above is/are correct?


a) 1 only
b) 2 only
c) Both 1 and 2
d) None
Correct Answer : B

Answer Justification :
Background: For the development of Biofuels, a National Policy
on Biofuels has been announced in the year 2009. So, under the
EBP Oil Marketing Companies sell ethanol blended petrol with
percentage of ethanol up to 10% as per BIS specifications,
depending upon its availability.

Justification: Statement 1: The prices are administered so that


the availability of ethanol can be improved by adequately
incentivizing the producers.

Statement 2: This is an eco-friendly way to convert ethanol from


bio feed stock.

Learning: In order to encourage production of Biodiesel in the


country, the Government had also announced the Biodiesel
Purchase Policy in October, 2005.

The Government has also permitted the direct sale of Biodiesel


(B-100) for blending with High Speed Diesel to all consumers, in
accordance with the specified blending limits and the standards
specified by the BIS.

84. Methanol is a promising fuel for waterways in India because

1) It does not have any smog based emissions.


2) It is cheaper than most fossil fuels.
3) India manufactures all its requirement of methanol by domestic
production.
Select the correct answer using the codes below.
a) 1 and 2 only
b) 2 and 3 only
c) 2 only
d) 3 only
Correct Answer : C

Answer Justification :

Justification: Across the world, methanol is emerging as a


clean, sustainable transportation fuel of the future.

Methanol can be blended with gasoline in low-quantities and used


in existing road vehicles, or it can be used in high-proportion
blends.
Statement 1: Methanol is a clean-burning fuel that produces
fewer smog-causing emissions such as sulphur oxides (SOx),
nitrogen oxides (NOx) and particulate matter. This can improve
air quality and related human health issues.

Statement 2 and 3: It is clean, cheaper than fossil fuels and a good


substitute for heavy fuels. India imports methanol from Saudi
Arabia and Iran at present. Technology is also being
commercialized to use methanol as a diesel substitute.

Learning: Methanol is most commonly produced on a


commercial scale from natural gas. It can also be produced from
renewable sources such as biomass and recycled carbon dioxide.

As a high-octane vehicle fuel, methanol offers excellent


acceleration and power. It also improves vehicle efficiency.

The government has asked the think-tank NITI Aayog to study the
automobile standards developed in China to use methanol as an
alternative fuel.

Experts believe that Methanol economy will help India use its
vast reserves of coal while driving import substitution.

ENVIRONMENTAL IMPACT ASSESSMENT

85. Discount rate tool is usually employed in environmental analysis to

a) Evaluate the carbon sequestration derived from a particular initiative


b) Understand the interconnection between ecological services
c) Calculate inflation by using ecological GDP deflator
d) Determine the present value of future benefits

Correct Answer : D
Answer Justification :

Justification: When weighing the benefits and costs of coastal


restoration projects and other environmental management programs, the
selection of a discount rate is a key consideration and often a source of
controversy.
The discount rate is the rate at which society as a whole is willing to trade
off present for future benefits. Because a rupee received today is
considered more valuable than one received in the future.
When weighing the decision to undertake a project with long-term
benefits (e.g., wetland protection programs) versus one with short-term
benefits and long-term costs (e.g., logging forests near aquatic
ecosystems), the discount rate plays an extremely important role in
determining the outcome of the analysis.
For example, if the present value of all discounted future benefits of a
restoration project is equal to 30 million rupees and the discounted
present value of project costs totals 20 million, the benefit- cost ratio
would be 1.5 (30 million / 20 million), and the net benefit would be 10
million (30 million
—20million)
Learning: Indeed, a number of reasonable decision measures (e.g., net
present value, benefit-cost ratio, internal rate of return, return on
investment) depend critically on the chosen discount rate.
However considering nations or societies with time horizons in the
thousands of years, discounting the future at all is highly questionable.
This is one of the most heavily debated issues in ecological economics.
Discount rates of even 1–2 percent per year shift the costs of
environmental degradation to later generations, and reduce incentives for
long-term environmentally favourable projects.

86. Many development and industrial projects such as erection of dams,


mining, and construction of industries or roads require diversion of forest
land. Consider the following in this regards.

1) Any project proponent, government or private, must apply for forest


clearance from Ministry of Environment, Forests and Climate Change
(MoEFCC) through the concerned forest department of the state
government.
2) As per the Forest (Conservation) Act, 1980, the project proponent
recommends and approves the compensation for the lost forest land.

Which of the above is/are correct?

a) 1 only
b) 2 only
c) Both 1 and 2
d) None

Correct Answer : A
Answer Justification :

Justification: Statement 1 and 2: This proposal is to be submitted


through the concerned forest department of the state government. If
clearance is given, then compensation for the lost forest land is also to be
decided by the ministry and the regulators.
Due to certain discrepancies in the implementation of compensatory
afforestation, some NGOs had approached The Hon’ble Supreme Court for
relief.
The Supreme Court in 2009 issued orders that there will be a
Compensatory Afforestation Fund Management and Planning Authority
(CAMPA) as National Advisory Council under the chairmanship of the
Union Minister of Environment & Forests for monitoring, technical
assistance and evaluation of compensatory afforestation activities.
Learning: Objectives of CAMPA
Compensatory Afforestation Fund Management and Planning Authority
(CAMPA) are meant to promote afforestation and regeneration activities
as a way of compensating for forest land diverted to non-forest uses
National CAMPA Advisory Council has the following mandate

 Lay down broad guidelines for State CAMPA.


 Facilitate scientific, technological and other assistance that may be
required by State CAMPA
 Make recommendations to State CAMPA based on a review of their plans
and programmes
 Provide a mechanism to State CAMPA to resolve issues of an inter-state or
Centre-State character.

87. Which of the following items are relevant to properly conduct an


Environmental Impact Assessment (EIA) at a locality for an upcoming
project?

1) Landscape, including the details of drainage and natural water structures


2) Pattern of permanent wind flow in the locality
3) Availability of eateries in the locality
4) Position of sanitary fills in the locality
5) Calorie intake of the population

Select the correct answer using the codes below.

a) 1, 4 and 5 only
b) 1, 2 and 3 only
c) 1, 2 and 4 only
d) 2, 3, 4 and 5 only

Correct Answer : C
Answer Justification :

Justification: Statement 1, 2 and 3: Suppose a thermal power plant is to


be setup in the area. The pattern of predominant wind flow would affect
the distribution of emissions from the plant in the locality.
Also, the wastewater of waste products emitted by the plant should not be
very close to local water storage structures. Also, sanitary fills should not
be destroyed by the plant as it will affect waste management in the area.
One can take many examples and justify these statements. So, 1, 2 and 3
are correct.
Statement 3 and 5: Availability of eateries in the locality and eateries are
remote considerations and trivial matters when it comes to an EIA.
BIODIVERSITY

88.The Global 200 is the list of priority ecoregions identified by

a) World Wildlife Fund (WWF)


b) United Nations Environment Programme (UNEP)
c) International Union for Conservation of Nature (IUCN)
d) Food and Agriculture Organization (FAO)

Correct Answer : A
Answer Justification :

Background: The WWF has identified 867 terrestrial ecoregions across


the Earth's land surface, as well as freshwater and marine ecoregions. The
goal of this classification system is to ensure that the
full range of ecosystems will be represented in regional conservation and
development strategies.
Of these ecoregions, the WWF selected the Global 200 as the ecoregions
most crucial to the conservation of global biodiversity. The Global 200 list
actually contains 238 ecoregions, made up of 142 terrestrial, 53
freshwater, and 43 marine ecoregions.
Learning: These regions arepriorities for conservation. According to
WWF, an ecoregion is defined as a "relatively large unit of land or water
containing a characteristic set of natural communities that share a large
majority of their species dynamics, and environmental conditions".
The WWF assigns a conservation status to each ecoregion in the Global
200: critical or endangered; vulnerable; and relatively stable or intact.
Over half of the ecoregions in the Global 200 are rated endangered.

89. Consider the following statements.

1) Wetlands in India occupy more area than the net sown area.
2) A greater area is under Inland wetlands than coastal wetlands in India.

Which of the above is/are correct?

a) 1 only
b) 2 only
c) Both 1 and 2
d) None

Correct Answer : B
Answer Justification :

Justification: Statement 1: - India's total geographical area is 329


million hectares. Out of this, 195 million hectare is gross cropped area and
141 million hectare is net sown area.
On the other hand, wetlands in India account for 4.7% of the total
geographical area of the country (around 15-16 mHa only).
Statement 2: Out of this, area under inland wetlands accounts for 69%,
coastal wetlands 27%, and other wetlands (smaller than 2.25 ha) 4% (SAC,
2011).
In terms of average area under each type of wetland, natural coastal
wetlands have the largest area.
These wetlands provide numerous ecosystem goods and services, but are
under stress.
Reasons for wetlands loss in India are urbanization, land use changes and
pollution.

90. Ecological balance may be disturbed due to the

1) Introduction of new species


2) Natural hazards
3) Anthropogenic pollution
Select the correct answer using the codes below.
a) 1 only
b) 2 only
c) 2 and 3 only
d) 1, 2 and 3
Correct Answer : D

Answer Justification :

Justification: We are giving examples to illustrate the above


statements.

Statement 1: Exotic species may disturb the local food chain or


create scarcity of competitive resources.

Statement 2: Natural hazards like tsunami or cyclone can uproot


coastal mangroves.

Statement 3: In the plants, any disturbance in the native forests


such as clearing the forest causes a change in the species
distribution.

This change is due to competition where the secondary forest


species such as grasses, bamboos or pines overtakes the native
species changing the original forest structure.
91. Which of these is NOT an Asian biodiversity hotspot?

a) Eastern Arc mountains


b) Peninsular Malaysia
c) Sinharaja forest
d) Eastern Himalayas
Correct Answer : A

Answer Justification :

Justification: Eastern Arc mountains is in Tanzania. A hotspot


is a region where a lot of endangered and endemic species are
found.

92. Consider the following statements. Assertion (A): The more the
variety of species in an ecosystem, the more stable the ecosystem
is likely to be. Reason (R): Genetic diversity is essential for a
healthy breeding of and provision of a variety of services between
a population of species.

In the context of the above, which of these is correct?


a) A is correct, and R is an appropriate explanation of A.
b) A is correct, but R is not an appropriate explanation of A.
c) A is correct, but R is incorrect.
d) Both A and R are incorrect.
Correct Answer : A

Answer Justification :

Justification: Genetic biodiversity refers to the variation of genes


within species.

Species capture and store energy, produce and decompose


organic materials, help to cycle water and nutrients throughout
the ecosystem, fix atmospheric gases and help regulate the clima
93. Which of the following best defines the Biosphere?

a) It is the upper region of lithosphere.


b) It is the is the narrow zone where land, water and air come together to
contain life forms.
c) It is the lower atmospheric region near the hydrosphere.
d) It is the abiotic component of all forms on earth.
Correct Answer : B

Answer Justification :

Learning: The solid portion of the earth on which we live is called the
Lithosphere.

The gaseous layers that surround the earth, is the Atmosphere, where
oxygen, nitrogen, carbon dioxide and other gases are found.

Water covers a very big area of the earth’s surface and this area is
called the Hydrosphere.

The Hydrosphere comprises water in all its forms, that is, ice, water
and water vapour.

The Biosphere is the narrow zone where we find land, water and air
together, which contains all forms of life.

ANIMAL BIODIVERSITY

94. This bird species is endemic to India, where it occurs in two isolated
populations, one in Western India and the other in Eastern Ghats. The
latest Indian Birding Fair at Rajasthan has been dedicated to this species.
It is

a) White Naped Tit bird


b) Blue-toothed Shrew
c) Siberian crane
d) Great Indian bustard

Correct Answer : A
Answer Justification :
Justification: You can solve such questions by elimination as well.

Option D is not found in Southern India in Eastern Ghats. Option B –


Shrew – is an animal, not a bird.

Learning: White Naped Tit bird species is very patchily distributed and
has been considered to be vulnerable to extinction especially because of
the scarcity of suitable habitats particularly nest cavities made by
woodpeckers.

Two patches are: one in central and southern Rajasthan, Kutch and
northern Gujarat in the north-west, and the other in the Eastern Ghats of
southern Andhra Pradesh, northern Tamil Nadu and Karnataka in the
south.

It inhabits tropical, dry thorn-scrub forests, particularly those dominated


by Acacia species, from the lowlands and foothills up to at least 750 m,
occasionally straying into gardens, vegetated streambeds and irrigated
crop fields during drought years.

The 21st Indian Birding Fair is happening at the Man Sagar Lake,
Rajasthan and this year, this fair is dedicated to the White Naped Tit
bird, which is quite rare in Jaipur and are at the verge of extinction.

It is a rare species, with a maximum of 165 records pertaining to 250


individuals over the last 150 years. It appears to have declined
substantially.

95. Consider the following about Chiru, which is also known as the Tibetan
antelope.

1) It is hunted for its underfur, Shahtoosh.


2) At present, international trade in their products is strictly prohibited.
3) It has recently been ascribed the status of ‘Critically Endangered’ by IUCN.

Select the correct answer using the codes below.

a) 1 and 2 only
b) 1 only
c) 2 and 3 only
d) 1 and 3 only

Correct Answer : A
Answer Justification :

Justification: At present, international trade in their products is strictly


prohibited.
The Tibetan antelope inhabits open alpine and cold steppe environments
between 3,250 and 5,500 m elevation. They prefer flat, open terrain, with
sparse vegetation cover.
They are found almost entirely in China, where they inhabit Tibet,
southern Xinjiang, and western Qinghai; a few are also found across the
border in Ladakh.
Its wool is renowned for its quality and has traditionally been transported
to Srinagar, where it is woven into an extremely fine fabric used to make
shawls.
In 2017, the Chiru goat species was assessed as ‘near threatened’ by the
International Union for Conservation of Nature (IUCN) because their
current population can only be maintained with continued high levels of
protection in its natural range and strict controls on trade of the shawls
made from its underfur

96. Natural habitat of Mouse Deer, in India, is

a) Salt marshes
b) Deciduous and evergreen forests
c) Arid regions
d) Mangroves

Correct Answer : B

Answer Justification :

Learning: Also called as ‘spotted Chevrotain’, it is one of the endangered


species usually found in deciduous and evergreen forests of the
country.These animals are nocturnal in habitat and because of their
small size they are called ‘smallest ungulates’ too.Their number has been
decreasing at an alarming rate in recent years because of destruction
of habitat and poaching.

So, in a novel and unique initiative attempted anywhere in the country,


the Telangana State Forest Department has re-introduced the endangered
‘mouse deer’ into the forests of Nallamalla in Amrabad Tiger Reserve.

97. Which of the following are migratory birds arriving in India?

1) Black-bellied Tern
2) Blue Jay
3) Amur Falcon

Select the correct answer using the codes below.

a) 1 and 3 only
b) 1 and 2 only
c) 1, 2 and 3
d) 2 and 3 only

Correct Answer : A
Answer Justification :

Justification: You can find the full list here – it need not be
remembered, but you may glance at it a couple of times to become familiar
with the names so that you can mark the correct answer in the exam.
http://bnhsenvis.nic.in/Database/MigratorybirdsIndia_835.aspx
The number of migratory birds arriving in the country depends on various
factors including habitat quality and preference, human disturbances
along their migratory route, pollution, hunting and other biotic factors etc.
However, as per the latest ‘Asian Waterbird Census’ coordinated by the
Wetlands International, the population of threatened migratory birds in
the entire flyway region which includes Central Siberia, Mongolia, Central
Asian republics, Iran, Afghanistan, Gulf States and the Indian sub-
continent, are
either decreasing or stable.
The Government of India is providing financial and technical assistance to
specialized institutions like Salim Ali Centre for Ornithology and
Natural History (SACON), Wildlife Institute of India (WII),
Bombay Natural History Society (BNHS) etc. for conducting
various studies for monitoring populations of important bird species and
their habitat.

98. With reference to the Snow Leopard, consider the following statements.

1) They are found only in Northern Himalayas.


2) They are known for their mighty roar.
3) The Government of India has identified the snow leopard as a flagship
species for the high altitude Himalayas.

Select the correct answer using the codes below.

a) 1 and 2 only
b) 3 only
c) 2 and 3 only
d) 1, 2 and 3

Correct Answer : B
Answer Justification :

Justification: Statement 1: Snow leopards live in the mountainous


regions of central and southern Asia. They are found at elevations of
3,000-5,000 metres or higher in the Himalayas.
In India, their geographical range encompasses a large part of the western
Himalayas including the states of Jammu and Kashmir, Himachal
Pradesh, Uttarakhand and Sikkim and Arunachal Pradesh in the eastern
Himalayas.
Snow leopards prefer steep, rugged terrains with rocky outcrops and
ravines. This type of habitat provides good cover and clear view to help
them sneak up on their prey.
The last three states form part of the Eastern Himalayas – a priority global
region of WWF andthe Living Himalayas Network Initiative.
Statement 2: Snow leopards do not roar.
Statement 3: GoI has developed a centrally-supported programme called
Project Snow Leopard for the conservation of the species and its habitats.
They are listed in Schedule I of Wildlife (Protection) Act 1972, in Appendix
I of CITES and as Vulnerable on IUCN Red List.

99. The olive ridley turtle is the smallest and most abundantly found sea
turtle in the world found all along the Indian coastline. It is facing which
of the following threats in India?

1) Absence of sites for mass nesting


2) International trade in these species is not covered under CITES
3) Accidental killing of adult turtles through entanglement in trawl nets

Select the correct answer using the codes below.

a) 1 only
b) 1 and 3 only
c) 3 only
d) 2 and 3 only

Correct Answer : C
Answer Justification :

Justification: Statement 1: The Olive ridley turtles are the smallest and
most abundant of all sea turtles found in the world, inhabiting warm
waters of the Pacific, Atlantic and Indian oceans. These turtles, along with
their cousin the Kemps ridley turtle, are best known for their unique mass
nesting called Arribada, where thousands of females come together on the
same beach to lay eggs.
Though found in abundance, their numbers have been declining over the
past few years, and the species is recognized as Vulnerable by the IUCN
Red list.
Olive-ridleys face serious threats across their migratory route, habitat and
nesting beaches, due to human activities such as turtle unfriendly fishing
practices, development and exploitation of nesting beaches for ports, and
tourist centres.
Statement 2: Though international trade in these turtles and their
products is banned under CITES Appendix I, they are still extensively
poached for their meat, shell and leather, and their eggs, though illegal to
harvest, have a significantly large market around the coastal regions.
Statement 3: However, the most severe threat they face is the accidental
killing of adult turtles through entanglement in trawl nets and gill nets due
to uncontrolled fishing during their mating season around nesting
beaches.
100. With reference to Indus Dolphins, consider the following statements.

1) They are amphibians.


2) They are a blind species that communicate through echo like bats do.
3) They are found only in India and Pakistan.

Select the correct answer using the codes below.

a) 1 and 2 only
b) 2 only
c) 2 and 3 only
d) 1 and 3 only

Correct Answer : C
Answer Justification :

Justification: Statement 1: It is a mammal.


Mammals are warm-blooded, but amphibians are cold-blooded. Mammals
have hairs on skin, whereas amphibians have a bare and moistened skin.
Mammals have mammary glands to feed the young but amphibian
newborns are not breast-fed
Statement 2 and 3: It is a blind species that communicates through echo
like bats do, Indus dolphins are one of the seven freshwater dolphins
found across the world.
They are found only in India and Pakistan. In Punjab, they are confined to
only a 185 km stretch between Talwara and Harike Barrage in India’s Beas
river in Punjab.
The most flourishing population of the Indus dolphin, platanista gangetica
minor, is found across Pakistan where their numbers are estimated to be
around 1,800 over a stretch of 1,500 km of the Indus river.
Learning: It is listed by the IUCN as endangered on its red list of
threatened species. IUCN suspects the population size of the Indus river
dolphins has reduced by more than 50% since 1944.
For the conservation of Indus dolphins – one of the world’s rarest
mammals – the Punjab government along with WWF-India are
conducting the first organised census on their population.

101. With reference to the Greater One Horned Rhino (Indian Rhino),
consider the following statements.

1) They prefer alluvial flood plains and areas containing tall grasslands.
2) Indian rhinos cannot swim despite being amphibious.
3) The Indian rhino has a single horn, which is present in both sexes.

Select the correct answer using the codes below.

a) 1 only
b) 1 and 2 only
c) 1 and 3 only
d) 2 and 3 only
Correct Answer : C
Answer Justification :

Justification: Statement 1: The preferred habitat of an Indian


rhinoceros is alluvial flood plains and areas containing tall grasslands
along the foothills of the Himalayas.
Formerly, extensively distributed in the Gangetic plains, today the species
is restricted to small habitats in Indo-Nepal terai and northern West
Bengal and Assam.
In India, rhinos are mainly found in Kaziranga NP, Pobitara WLS, Orang
NP, Manas NP in Assam, Jaldapara NP and Garumara NP in West Bengal
and Dudhwa TR in Uttar Pradesh.
Statement 2 and 3: The Indian rhino has a single horn, which is present in
both sexes. It is the largest of all the Asian rhinos. Considered to be the
most amphibious of all the rhino species, the Indian rhino is an excellent
swimmer.
It can run at speeds of up to 55km/hr for short periods of time. Though it
has an excellent sense of hearing and smell, its eyesight is relatively poor.

102. Changthangi goat of Northern Himalayan region is known for yielding

a) Pashmina wool
b) Shahtoosh
c) Himayalan Yew
d) Pitas and Humas
Correct Answer : A

Answer Justification :

Learning: They are a breed of goat inhabiting the plateaus in Tibet and
neighbouring areas
They are best known for the pashmina wools

103. A variety of dolphin locally called Susu is naturally found in India in

a) Sambhar lake
b) Ganga and Brahmaputra rivers
c) Chilka lake
d) Vembanad river
Correct Answer : B

Answer Justification :

Learning: The Ganges River dolphin or Susu, lives in one of the


most densely populated regions of the world in the fresh water of
Ganga and Brahmaputra.
One of the main threats to the species is loss of habitat due in
large part to the creation of dams and irrigation projects.

The presence of Susu is an indication of the health of the river.


The untreated industrial and urban wastes with high amount of
chemicals are killing this species.

PLANT BIODIVERSITY

104. Scientists from Botanical Survey of India (BSI) have discovered a new
species of parasitic flowering plant named Gleadovia konyakianorum, that
was named in honour of Konyak tribe of Nagas. Consider the following
about it.
1) The plant was discovered in dry deciduous forests.
2) The plant is different from others because it has no chlorophyll.

Which of the above is/are correct?

a) 1 only
b) 2 only
c) Both 1 and 2
d) None

Correct Answer : B
Answer Justification :

Justification: Statement 1: This could have been made out because the
Naga tribes do not live in deciduous forests and instead inhabit semi-
evergreen and evergreen forests. So, this option could have been eliminated
easily.
It was found in a forest at an altitude of 1 ,500-1,600 metres during an
exploration near Tobu town of Mon district in eastern Nagaland.
Statement 2: Gleadovia konyakianorum is a holoparasite (complete
parasite) plant that derives its entire nutritional requirement from the host
plant, which is a Strobilanthes species.
It has no chlorophyll (which helps a plant make its own food using
sunlight).
Though it has no chlorophyll, the plant has a vascular system and extracts
(its nutrition from the host plant with the help of a haustorium.
Haustorium is specialised structure with which plant parasites attach
themselves to host plants and derive nutrition.
105. A new species of wild banana that grows to height of nine metres, bears
an edible sweet-and-sour tasting fruit which is boat-shaped and has
numerous bulb-shaped seeds, has been recently found in

a) Lakshadweep Islands
b) Western Ghats
c) Central India
d) Andaman and Nicobar Islands

Correct Answer : D
Answer Justification :

Learning: It was named Musa paramjitiana in honour of Paramjit Singh,


director of the Botanical Survey of India (BSI).
Musa paramjitiana species was discovered in North Andaman's
Krishnapuri forest, 6 kilometres from any human habitation.
The fruit of Musa paramjitiana is part of diet of local tribes.
Its fruits and seeds have ethno-medicinal importance. Pseudo-stem and
leaves of these species are also used during religious and cultural
ceremonies.
Its conservation status has been declared as 'Critically Endangered' as it
has so far been spotted in only two locations on islands, each with 6 to 18
plants in a clump.
106. Consider the following statements.

1) Eucalyptus plantations supply moisture to the soil making them suitable


for arid and semi-arid plantation agriculture.
2) When grown using wastewater, eucalyptus plantations can remove toxic
metals.

Which of the above is/are correct?

a) 1 only
b) 2 only
c) Both 1 and 2
d) None

Correct Answer : B
Answer Justification :

Justification: It has been suggested that we should plant eucalyptus


trees for arresting ecological degradation.
Justification: Statement 2: Growing eucalyptus along sewage ponds
absorbs all surplus
wastewater. It is said that these trees are thirsty of water and guzzle a lot
of water.
Statement 2: However, where water is sufficient or there is a lot of waste
water, it is utilized for tree growth which then could yield fuelwood,
helping in many ways.
Eucalyptus plantations are known to sequester, tolerate and accumulate
high levels of various heavy metals.

107. With reference to Himalayan Yew, consider the following


statements.

1) It is an endemic and exclusive species of Indo-Burma biodiversity


hotspot region.
2) Taxol extracted from the plant has been successfully used to treat
some cancers
3) It is currently classified as endangered by the IUCN.

Select the correct answer using the codes below.


a) 1 and 2 only
b) 2 and 3 only
c) 3 only
d) 1 and 3 only
Correct Answer : B

Answer Justification :
Justification: The Himalayan Yew is a medicinal plant found in
various parts of Himachal Pradesh and Arunachal Pradesh.

A chemical compound called ‘taxol’ is extracted from the bark,


needles, twigs and roots of this tree, and it has been successfully
used to treat some cancers – the drug is now the biggest selling
anti- cancer drug in the world.

The species is under great threat due to over-exploitation.

108. Mosses is a bioindicator. What do you understand by this statement?

1) It reflects changes to ecosystems such as pollution.


2) It cannot be cloned or genetically engineered.
3) It does not contain mRNA.

Select the correct answer using the codes below.


a) 1 only
b) 2 and 3 only
c) 3 only
d) 1 and 2 only
Correct Answer : A
Answer Justification :

Justification: Mosses are a common flowerless plant found in all cities


especially in damp (humid) or shady locations. It generally absorbs water
and nutrients from their immediate environments.

So, according to scientists, delicate mosses found on rocks and trees in


cities around the world can be used as low-cost bioindicator to monitor
urban pollution and to measure the impact of atmospheric change.

As a bioindicator, mosses respond to pollution or drought- stress by


changing its shape, density or disappearing. This characteristic will
allow scientists to calculate atmospheric alterations and air pollution.

109. Bromeliads are able to live in a vast array of environmental


conditions due to their many adaptations.
Which of these are such adaptations?

1) Opening their stomates at night rather than during the day


2) Forming a tightly bound structure with their leaves that helps to
capture water and nutrients
3) The most well-developed root system amongst monocot flowering
plants

Select the correct answer using the codes below.


a) 1 and 2 only
b) 3 only
c) 2 and 3 only
d) 2 only
Correct Answer : A

Answer Justification :

Justification: Statement 1: Trichomes, in the form of scales or


hairs, allow bromeliads to capture water in cloud forests and help
to reflect sunlight in desert environments.

Bromeliads also use crassulacean acid metabolism (CAM)


photosynthesis to create sugars.
This adaptation allows bromeliads in hot or dry climates to open
their stomates at night rather than during the day, which reduces
water loss through transpiration.

Statement 2 and 3: Some bromeliads have also developed an


adaptation known as the tank habit, which involves them
forming a tightly bound structure with their leaves that helps to
capture water and nutrients in the absence of a well-developed
root system.

110. What is/are Castor seed plants generally known for?

1) They are a bio-indicator of pollution because they do not grow in areas


where groundwater is polluted.
2) They are an exotic species to India, and were introduced by the French.
3) They form components of many cosmetics.
4) High percentage of ricinoleic acid in its residues has anti-microbial
properties.

Select the correct answer using the codes below.


a) 2, 3 and 4 only
b) 3 and 4 only
c) 1, 2 and 4 only
d) 2 and 3 only
Correct Answer : B

Answer Justification :

Justification: Statement 1: It is one of the sturdiest plants that can


grow in areas where the soil is highly polluted, including in areas
where mining is carried out.

Statement 2: Castor is indigenous to the southeastern Mediterranean


Basin, Eastern Africa, and India.
Statement 3: It is a popular medicine as well as used in cosmetics.

Statement 4: Castor seed is the source of castor oil, which has a wide
variety of uses. The seeds contain between 40% and 60% oil that is rich
in triglycerides, mainly ricinolein. The acid and its derivatives, inhibits
many microbes, whether viral, bacterial or fungal.

Learning: The use of castor bean oil ("eranda") in India has been
documented since 2000 BC in lamps and in local medicine as a
laxative, purgative, and cathartic in Unani, Ayurvedic, siddha and
other ethnomedical systems.

111. Consider the following statements.


1) Zenkeria Sebastinei is a critically endangered grass in
Agasthiyamalai peak.
2) Humbodtia decurrens Bedd is a highly rare endemic insect of the
Southern Western Ghats.

Which of the above is/are correct?


a) 1 only
b) 2 only
c) Both 1 and 2
d) None
Correct Answer : A

Answer Justification :

Justification: Statement 1: is a species of grass in the family


Poaceae found in Pothigai Hills, Kerala. It is classified as an
angiosperm, i.e. a flowering plant.

Statement 2: It is a species of legume in the Fabaceae family.

It is found only in India in the southern part of the Western Ghats,


ranging from the Anamalai Hills to the Travancore range.

MARINE ORGANISM\

112. The reduction in population of Phytoplankton may reduce

1) The amount of carbon dioxide drawn out of the atmosphere


2) Amount of carbon transferred to the deep ocean for long-term storage
3) Number of fish in the sea

Select the correct answer using the codes below.

a) 1 only
b) 1 and 2 only
c) 2 and 3 only
d) 1, 2 and 3

Correct Answer : D
Answer Justification :

Justification: Statement 1: Phytoplankton need carbon dioxide for


photosynthesis, just like trees.
Carbon dioxide (CO2) from the atmosphere dissolves in cold ocean water.
During a phytoplankton bloom, which can span hundreds of miles and be
seen from space, the tiny organisms take up the dissolved CO2 and
convert it to organic carbon – a form that animals can use as food to grow,
the essential base of the marine food web.
Then when the phytoplankton cell dies, it sinks to the ocean floor, taking
with it the carbon in its body.
Statement 2: Diatoms, the largest type of phytoplankton algae, have
declined more than 1 percent per year from 1998 to 2012 globally, with
significant losses occurring in the North Pacific, North Indian and
Equatorial Indian oceans.
The reduction in population may reduce the amount of carbon dioxide
drawn out of the atmosphere and transferred to the deep ocean for long-
term storage.
Because they are larger than other types of phytoplankton, diatoms can
sink more quickly than smaller types when they die.
A portion will circulate back to the surface because of ocean currents, and,
like fertilizer, fuel another phytoplankton bloom. But the rest will settle on
the sea floor miles below, where they will accumulate in sediment.
Statement 3: Phytoplankton Population has dropped 40 Percent Since
1950. Researchers find trouble among phytoplankton, the base of the food
chain, which has implications for the marine food web and the world's
carbon cycle. The scientists believe that rising sea surface temperatures
are causing this.
113. The Gharial is under increasing pressure for survival due to factors like

1) Sand-mining and riverside agriculture


2) Construction of dams and barrages obstructing dispersal and migration of
fish
3) Pollution and siltation of rivers

Select the correct answer using the codes below.

a) 1 only
b) 2 and 3 only
c) 1 and 3 only
d) 1, 2 and 3

Correct Answer : D
Answer Justification :

Justification: These are a combination of factors such as:


Habitat alteration and destruction: A combination of land-use changes
and exploitation such as sand-mining, riverside agriculture, livestock
grazing, and hydrological modifications such as building of dams for water
diversion.
Prey depletion: Over harvesting of fish stock- Construction of dams and
barrages obstructing dispersal and migration of fish.

Direct mortality: Drowning of Gharial in fishing nets. Its nest destruction


and local egg collection.
Pollution and siltation: Pollution and siltation of rivers damage fish
stocks, and are also believed to be the direct cause of the catastrophic
dieoff of 2007-2008 in the Chambal.

 Hunting: In the past, Gharial was hunted for skin, trophies and use in
indigenous medicine

Learning: The gharial once thrived in all the major river systems of the
Indian Subcontinent, spanning the rivers of its northern part from the
Indus River in Pakistan across the Gangetic floodplain to the Irrawaddy
River in Myanmar.
Today, it is extinct in the Indus River, in the Brahmaputra of Bhutan and
Bangladesh, and in the Irrawaddy River.
In India, small populations are present and increasing in the rivers of the
National Chambal Sanctuary, Katarniaghat Wildlife Sanctuary, and Son
River Sanctuary.
Another small population exists in the rainforest biome of Mahanadi in
Satkosia Gorge Sanctuary, Odisha, where they apparently do not breed.
As the population has declined drastically in the past 70 years, the gharial
is listed as Critically Endangered on the IUCN Red List
114. Scientists from Zoological Survey of India (ZSI) have discovered three
new species of eel along northern Bay of Bengal coast. Eels are found
mostly at

a) Bottom of rivers and seas


b) Shorelines
c) Ocean surface during upwelling
d) Shallow ocean reefs

Correct Answer : A
Answer Justification :

Learning: There are about 1,000 species of eels identified so far across
the world. In India, there are around 125 species of eels identified.

Eel species belonging to Muraenidae family, referred commonly as Moray


eels, recorded about 200 species of which more than 30 species are found
in India.

With the three new discoveries, Bay of Bengal coast has yielded at least 5
new species of eel. In 2016, Gymnothorax indicus, an edible species of eel
was discovered. In 2015, a short brown unpatterned moray eel, named
Gymnothorax mishrai (Bengal moray eel) was discovered.

Japan consumes more than 70% of the global eel catch. European Eeel is
critically endangered.

115. Highest concentration of plankton is found at

a) Tropics
b) Sub-tropics
c) Higher temperate latitudes
d) Polar regions

Correct Answer : C
Answer Justification :

Justification: Highest concentration of plankton is found at higher


latitudes, tropics and subtropics have lower concentrations.
It is due to nutrient limitation brought about by strong, year-round
thermocline (temperature gradients) and pycnocline (salinity gradients).
Pycnocline refers to the rapid change in density with depth, and
thermocline, a rapid change in temperature with depth. Both of these limit
circulation of nutrients to the surface where planktons grow.
However, in areas where ocean upwelling occurs, nutrients are cycled to
the surface and rich plankton grounds can be found.
116. Phytoplankton are generally found in upper layers of ocean water due to

1) Absence of nutrients in deep ocean water


2) Sunlight abundance in top layers
3) Calm top layers of water due to absence of mixing between top and bottom
layers

Select the correct answer using the codes below.

a) 1 only
b) 1 and 2 only
c) 2 only
d) 2 and 3 only

Correct Answer : C
Answer Justification :

Justification: Phytoplankton in the open ocean need both sunlight and


nutrients (such as nitrate and phosphate) to be able to photosynthesize.
Sunlight is only available in the uppermost layers.
During photosynthesis, the nutrients are quickly used up by
phytoplankton so they are not available for long periods in the upper
layers under normal circumstances.
This is indeed the case in tropical waters, and as a result they are very
unproductive.
To escape this problem the seawater needs to be mixed regularly to bring
the nutrient rich deep waters up to the sunlight zone where the
phytoplankton can grow.
This is one of the reasons why areas where cold and warm currents meet
are very productive; the collision of currents causes mixing.

PROTECTED AREA NETWORK

117. What is an Eco-sensitive Zone (ESZ) as notified by the Ministry of


Environment and Forests (MoEF) frequently?

a) It is the area around a national park or sanctuary where developmental


activities are regulated
b) It is a zone of high endemism of species
c) It is the “Core zone” inside a protected area where no activities are allowed
d) It is an ecological hotspot which is internationally recognized as a part of
protected areas network

Correct Answer : A
Answer Justification :
Learning: As per MOEF 2011 guidelines, it is the area around a national
park or sanctuary upto 10 kms where developmental activities are
regulated (not prohibited) to:
Minimize impact of ecologically harmful developmental activities (e.g.
mining) on fragile ecosystem encompassing the area
Protects biodiversity by avoiding fragmentation of habitat
Act as a transition zone between areas of high protection (inside) and low
protection outside
Act as a "shock absorber" for the protected areas

118. Key Biodiversity Areas (KBAs) are nationally identified sites of global
significance. They include

1) Alliance for Zero Extinction (AZE) sites


2) Important Plant Areas (IPAs)
3) Important Sites for Freshwater Biodiversity (ISFB)
4) Important Bird and Biodiversity Areas (IBAs)

Select the correct answer using the codes below.

a) 1 and 3 only
b) 2, 3 and 4 only
c) 1 and 4 only
d) 1, 2, 3 and 4

Correct Answer : D
Answer Justification :

Justification: The identification of KBAs is an important approach to


address biodiversity conservation at the site scale i.e. at the level of
individual protected areas, concessions and land management units.
KBAs are identified using globally standardised criteria and thresholds,
and have clearly defined boundaries. There is no maximum or minimum
size of sites, because appropriate size varies according to the socio-
economic criteria, such as land use and tenure.
KBAs are seen as an ‘umbrella’ designation, which includes globally
important sites for different taxa and realms as described in the Q
Statement.
118. Natura 2000 is

a) the largest coordinated network of protected areas in the world


b) a GHG emission standard of European Union (EU) recently admitted by
the Board of Emissions in UNEP
c) a list of most vulnerable species as compiled by the IUCN
d) a non-profit organization that helps conserve small islands in LDCs

Correct Answer : A
Answer Justification :

Learning: Natura 2000 is a network of core breeding and resting sites


for rare and threatened species, and some rare natural habitat types which
are protected in their own right.
It stretches across all 28 EU countries, both on land and at sea.
The aim of the network is to ensure the long-term survival of Europe's
most valuable and threatened species and habitats, listed under both the
Birds Directive and the Habitats Directive.
Natura 2000 is not a system of strict nature reserves from which all
human activities would be excluded. While it includes strictly protected
nature reserves, most of the land remains privately owned.
The approach to conservation and sustainable use of the Natura 2000
areas is much wider, largely centered on people working with nature
rather than against it.
119. Most of the states with less than 10 per cent of the forest area lie in which
part of the country?

a) North and north-western regions


b) Eastern belts
c) Southern and south-western belt
d) South-eastern region

Correct Answer : A
Answer Justification :

Justification and Learning: These are Rajasthan, Gujarat, Punjab,


Haryana and Delhi.
There is a lot of variation in actual forest cover, which ranges from 9.56
per cent in Jammu and Kashmir to 84.01 per cent in Andaman and
Nicobar Islands.
According to state records, the forest area covers 23.28 per cent of the
total land area of the country.
It is important to note that the forest area and the actual forest cover are
not the same.
The forest area is the area notified and recorded as the forest land
irrespective of the existence of trees, while the actual forest cover is the
area occupied by forests with canopy.
The former is based on the records of the State Revenue Department,
while the latter is based on aerial photographs and satellite imageries.
Most of the forests in Punjab and Haryana have been cleared for
cultivation. States with 10-20 per cent forest area are Tamil Nadu and
West Bengal. In Peninsular India, excluding Tamil Nadu, Dadra and
Nagar Haveli and Goa, the area under forest cover is 20-30 per cent. The
north-eastern states have more than 30 per cent of the land under forest.
Hilly topography and heavy rainfall are good for forest growth.
CONSERVATION EFFORT

120. e-Green Watch is a portal for

a) online monitoring of various afforestation works being carried out


using CAMPA funds
b) evaluating the changes by local authorities in the net sown area
c) keeping a track on endangered and vulnerable species
d) compendium of the bio-resources attached with the GIS for surveying
several blocks of a district

Correct Answer : A
Answer Justification :

Learning: Since the primary function of Compensatory Afforestation


Fund Management and Planning Authority (CAMPA) is the regeneration
of vegetation cover and promoting afforestation as a way of compensating
for forest land which is diverted to non-forest uses.
It, therefore, calls for an information system so as to collect and present
information to monitor and track how well CAMPA funds are achieving
this end.

In view of the above background, e-Green Watch is being developed as an


integrated & online system that will be completely transparent, reliable
and accountable.

It will also present the data in real time and shall be accessible to all
stakeholders and public at large
121. Ramsar Sites are those that are

a) Potentially at risk as a result of technological developments, pollution


or other human interference only
b) Major wetlands of international importance
c) Wetlands that yield high biological and non-biological products for the
local community
d) Forest patches that can potentially be disconnected from major buffer
and core zones

Correct Answer : B
Answer Justification :

Learning: The Convention on Wetlands, known as the Ramsar


Convention, is an intergovernmental environmental treaty established in
1971 by UNESCO, and coming into force in 1975.

It provides for national action and international cooperation regarding the


conservation of wetlands, and wise sustainable use of their resources.

Ramsar identifies wetlands of international importance, especially those


providing waterfowl habitat.

When a country accedes to the Ramsar Convention, it must designate at


least one wetland site as a Wetland of International Importance.
Information on this first Ramsar Site is sent with the documents of
accession to UNESCO, the Convention’s depositary.

Many internationally important wetlands extend as one ecologically


coherent whole across national borders.

In these cases, Contracting Parties can agree to establish Ramsar Sites on


their territory as

parts of a bigger Transboundary Ramsar Site.

The authorities on both or all sides of the border agree to collaborate in


the management of the Transboundary Site, and notify the Secretariat of
their intent.

The record of Ramsar sites where changes in ecological character have


occurred, are occurring, or are likely to occur due to technology, or human
interference are covered in the Montreux Record.
122. The Caribbean Challenge Initiative (CCI) is a historic initiative for

a) Revival of cultural roots in the Caribbean communities


b) Conservation of coastal environments in marine protected areas
c) Revitalization of trade and economic links amongst the member states
d) Ending poverty, injustice and ensuring high economic growth before
the fulfillment of SGDs by 2030

Correct Answer : B
Answer Justification :

Learning: IT brings together for the first time governments, companies


and partners to accelerate action on conservation in the Caribbean.

Together, the 9 participating countries and territories have committed to


conserve at least 20% of nearshore and coastal environments in marine
protected areas by 2020 with the help of private and public sector
partners.

The Caribbean Biodiversity Fund has been established to support


achievement of the CCI commitments by 2020.

For e.g. the Bahamas established three marine protected areas, including
the largest one in the region. The Dominican Republic established 30 new
protected areas, surpassing their goal of conserving at least 20% of
nearshore and coastal environments, and Jamaica too has set up eight no-
take marine zones.

The Caribbean Biodiversity Fund is currently supported by the German


Development Bank, Global Environment Facility, and The Nature
Conservancy.

123. Alliance for Zero Extinction sites (AZE) is a

a) Agency of the World Wildlife Fund (WWF) that focuses on critical


regions and critical endemic species across the world to protect them
from mass extinction
b) Global list of sites containing ninty five per cent or more of the
remaining population of one or more species listed as endangered or
critically endangered on the IUCN Red List of Threatened Species
c) A group of like-minded international environmental NGOs that are
working towards in-situ conservation of major species that face the
threat of extinction
d) Observatory located at the ecological sites where the threat of
extinction of the top predators is highest

Correct Answer : B
Answer Justification :
Learning: Alliance for Zero Extinction (AZE) sites contain the entire
population of one or more species listed as Endangered or Critically
Endangered on the IUCN Red List of Threatened Species.

The Alliance itself is formed of 93 biodiversity conservation institutions


from 37 countries, and its goal is to prevent the extinction of species.

Following identification, the Alliance aims to act together to eliminate


threats and restore habitat at these sites to allow species populations to
rebound.

The focus of AZEs is on species that face extinction either because their
last remaining habitat is being degraded at a local level, or because their
restricted global range makes them especially vulnerable to external
threats.

AZEs are included under the 'umbrella' classification of Key Biodiversity


Areas (KBAs).
124. The Bio-Bridge Initiative is an overarching programme focused on

a) Supporting organic initiatives like the living root bridges of Meghalaya


b) catalyzing and facilitating technical and scientific cooperation among
Parties to the Convention on Biological Diversity and its protocols
c) Establishing migratory and buffer routes in IUCN Class-II Protected
Areas
d) Creating a comprehensive knowledge compendium of ecological hotspots
to make societies ware of their importance

Correct Answer : B
Answer Justification :

Learning: The Protocols include the one on biosafety (Cartagena


Protocol) and on access and benefit-sharing (Nagoya Protocol). The Bio-
Bridge Initiative facilitates technical and scientific cooperation among
Parties by:

linking Parties that have specific technical and scientific needs with
Parties or institutions that are able to provide the necessary technical
support and resources to meet those needs through mutual partnerships;
and

creating a space for countries and institutions to share knowledge, good


practices and lessons learned with each other.

125. In-situ initiatives beyond protected areas may include

1) On-farm agricultural biodiversity conservation targeted at traditional crop


varieties and crop wild relatives
2) Genetic sanctuaries in natural wild surroundings
3) Preserving species in Botanical gardens

Select the correct answer using the codes below.

a) 1 only
b) 2 and 3 only
c) 1 and 2 only
d) 1, 2 and 3

Correct Answer : C
Answer Justification :

Justification: Statement 3: This is an example of ex-situ conservation


because a botanical garden is not the natural habitat of those species.
In-situ measures are perceived as more holistic in their approach and
allow the conservation of processes or habitats which can’t be protected
through ex-situ measures (e.g. soil microbial processes, evolutionary
processes, specific ecosystems such as coral reefs or species with highly
specialised needs).
Other examples of in situ (natural site) conservation are:
Habitat restoration, recovery or rehabilitation;
Strategies for the sustainable use and management of biological resources;
Recovery programmes for nationally or sub-nationally threatened or
endangered wild species;
Genetic reserve conservation, ie. monitoring of genetic diversity in natural
wild populations within a delineated area (known as genetic sanctuaries or
gene management zones)
Control of threats to biodiversity such as invasive alien species, living
modified organisms or over exploitation;
Preservation and maintenance of traditional knowledge and practices; and
Implementation of the regulatory, legislation, management or other
frameworks needed to deliver the protection of species or habitats.
The Convention on Biological Diversity (CBD) recognizes the
complementary role of both conservation strategies (in-situ and ex-situ).

126. Indian Coral Reefs Monitoring Network (ICRMN) is a program of

a) International Union for Conservation of Nature (IUCN)


b) World Wildlife Fund (WWF)
c) Ministry of Environment, Forests and Climate Change (MoEFCC)
d) UNFCCC - India

Correct Answer : C
Answer Justification :

Learning: In India, the National Committee on Mangroves and Coral


Reefs of the Ministry of Environment and Forests (MOEF) is responsible
for developing strategies for conservation and management of coral reefs
and financially supporting their implementation.
Besides the support to regional research and development activities, the
MOEF has already formulated the Indian Coral Reef Monitoring Network
(ICRMN), a national coordinated programme to be implemented from
1999 onwards.
The objectives of the to monitor the biophysical, environmental and socio-
economic changes that natural causes and patterns of resource utilisation
entail and use this knowledge for a sustainable management of the reefs.
Learning: Coral Reefs are among the most diverse ecosystems in the
world - the "Rainforests of the Seas”.
It is estimated that one-third of all the world's fish species depend on coral
for their existence. They:
Play an important role in coastal protection by slowing down potentially
damaging tidal waves;
Act as sensitive indicators of water quality.
Are Considered ‘medicine chests’ of the future. Scientists believe that their
organisms could well hold the key to cures for cancer, and coral skeletons
are already being used as bone substitutes in reconstructive bone surgery.
127. Consider the following statements. WWF-India’s Small Grants
Innovation Program

1) offers eligible individuals a one-time grant for undertaking conservation


research
2) adopts one village in every major critical habitat of India to recreate it as
an eco-model village

Which of the above is/are correct?

a) 1 only
b) 2 only
c) Both 1 and 2
d) None

Correct Answer : A
Answer Justification :

Justification: With the aim of encouraging young Indians to respond


innovatively and independently to the conservation issues which affect the
country, WWF-India is offering eligible individuals a one-time grant of
upto INR 400,000 over a maximum period of 2 years for undertaking
conservation research/ action research.
The grants will be used to fund research and initiatives which especially
demonstrate innovation that is replicable, scalable and sustainable.
This can include (but not limited to):
Improving local livelihoods through conservation and natural resource
management or promoting livelihoods that reduce impacts on biodiversity
Aspects of trade involving wildlife species
Increasing understanding on the status of lesser known or lesser studied
species of wildlife

128. Montreux Record under the Ramsar Convention is a register of wetland


sites on the List of Wetlands of International Importance where changes
in ecological character have occurred, are occurring, or are likely to occur.
In India, it includes

1) Chilka Lake
2) Bhitarkanika Mangroves
3) Keoladeo National Park
4) Loktak Lake

Select the correct answer using the codes below.

a) 1 and 2 only
b) 1 and 4 only
c) 3 and 4 only
d) 1, 2, 3 and 4
Correct Answer : C

Answer Justification :

Justification: Montreux Record includes such sites where these changes


are likely to occur as a result of technological developments, pollution or
other human interference.

It is a voluntary mechanism to highlight specific wetlands of international


importance that are facing immediate challenges.

It is maintained as part of the List of Ramsar wetlands of international


importance.

In India, only 2 sites – Keoladeo National park and Loktak Lake are
covered under the record.Globally around 48 sites are listed.

Statement 1: A brackish water lagoon, spread over the Puri, Khurda and
Ganjam districts of Odisha state on the eastern coast of India, It is the
largest wintering ground for migratory birds on the Indian sub-
continent. It is the largest coastal lagoon in India.

Statement 2: The Bhitarkanika Mangroves cover an area of 650 km2 in the


river delta of the Brahmani and Baitarani rivers. It is also famous for its
salt water crocodiles and Olive ridley sea turtle.

Statement 3:Keoladeo Ghana National Park is a man-made and man-


managed wetland and one of the national parks of India. It is a famous
avifauna sanctuary that hosts thousands of birds, especially during the
winter season.

Statement 4:Loktak Lake is the largest freshwater lake in Northeast India


and is famous for the phumdis (heterogeneous mass of vegetation, soil
and organic matter at various stages of decomposition) floating over it.

Located on this phumdi, KeibulLamjao National Park is the only floating


national park in the world. The park is the last natural refuge of the
endangered Sangai (state animal).

129. Which of the following states has recently come up with a unique way of
preserving trees by encouraging people to forge a relationship of
brotherhood or sisterhood with trees through a practice?

a) Uttarakhand
b) Madhya Pradesh
c) Sikkim
d) Andhra Pradesh
Correct Answer : C
Answer Justification :

Learning: Under Sikkim Forest Tree (Amity & Reverence) Rules 2017,
the Government of Sikkim has come up with locally known as Mith/Mit
or Mitini which shows the parent-adoptee relationship between a tree and
a person.
As per the Sikkim Forest Tree (Amity & Reverence) Rules 2017:
Under the rules, State government shall allow any person to associate with
trees standing on his or her private land or on any public land by entering
into a Mith/Mit or Mitini relationship.
A person can also adopt a tree “as if it was his or her own child in which
case the tree shall be called an adopted tree.” Another way to forge a
relationship with a tree is by “preserving a tree in remembrance of a
departed relative in which case the tree shall be called a smriti tree.”
With this, Sikkim is keen to not just preserve its forest cover but also forge
amity between people and trees

130. The only global intergovernmental mechanism directly addressing the


connectivity between terrestrial, freshwater, coastal and marine
ecosystems is

a) Partnerships in Environmental Management for the Seas of East Asia


(PEMSEA)
b) United Nations Program for Disaster Risk Reduction (UNPDR)
c) UNEP Global Programme of Action
d) Sustainable Energy for All (SE4ALL)

Correct Answer : C
Answer Justification :

Justification: The UNEP Global Programme of Action (UNEP/GPA)


aims at preventing the degradation of the marine environment from land-
based activities by facilitating the realization of the duty of States to
preserve and protect the marine environment.
It is unique in that it is the only global initiative directly addressing the
connectivity between terrestrial, freshwater, coastal and marine
ecosystems.
It was created in 1995.
It aims to be a source of conceptual and practical guidance to be drawn
upon by national and/or regional authorities for devising and
implementing sustained action to prevent, reduce, control and/or
eliminate marine degradation from land-based activities.
Intergovernmental Review Meetings are organized every 5 years to review
the progress made by countries in the implementation of the GPA through
their respective National Action Plans.
131. The National Wildlife Action Plan (NWAP) for 2017-31 adopts a
“landscape approach” to conserve wildlife. What does this approach
mean?

1) To conserve all the wildlife rather than the areas where specific wildlife
occurs
2) Focussing on conservation of wildlife by establishing smaller National
parks and Sanctuaries
3) Doing away with the system of Reserved Forests

Select the correct answer using the codes below.

a) 1 only
b) 2 and 3 only
c) 1 and 2 only
d) 1, 2 and 3

Correct Answer : A
Answer Justification :

Justification: The plan adopts a “landscape approach” in conservation


of all wildlife – uncultivated flora and fauna – that have an ecological
value to the ecosystem and to mankind irrespective of where they occur. It
gives special emphasis to recovery of threatened species of wildlife while
conserving their habitats.

This is a key strategic change in the plan.


This means that while till now programmes and plans related to wildlife
were focused on and around national parks and sanctuaries, now the
strategies would be based on the landscape of the region that may not be
limited to a reserve forest system alone.

132. The Ecosystem-Based Management (EBM) Tools Network is focussing


on tools for

1) Climate change vulnerability assessment and adaptation planning


2) Ecosystem-based coastal and marine spatial planning

Which of the above is/are correct?

a) 1 only
b) 2 only
c) Both 1 and 2
d) None

Correct Answer : C
Answer Justification :
Justification: You will face such questions in Prelims, some are meant to
be bouncers, some to be answered smartly. This was an easy question, if
you have solved many such similar ones in the past. Both options seem
correct in the first go.
Ecosystem-Based Management tools are methods and software that help
practitioners incorporate scientific and socioeconomic information into
decision making.
EBM tools can help: develop models of ecosystems; generate scenarios
illustrating the consequences of different management decisions on
natural resources and the economy; and facilitate stakeholder involvement
in planning processes.
The network is currently focusing on tools for integrated land-sea
planning to minimize the impacts of land use on coastal and marine
environments, apart from S1 and S2.

RED BOOK

133. Red panda is native to

a) Eastern Himalayas and south-western China


b) West Asia
c) Mediterranean Region
d) Siberia and Central Asia

Correct Answer : A
Answer Justification :

Justification: Distribution of the red panda is disjointed, with two


extant subspecies:
Western red panda lives in the western part of its range, in Nepal, Assam,
Sikkim, and Bhutan.
Styan's red panda lives in the east-north-eastern part of its range, in
southern China and northern Burma.
It is also covered under CITES. Habitat loss, poaching and hunting are the
biggest threats to red panda.
It is a solitary animal, mainly active from dusk to dawn, and is largely
sedentary during the day.

134. Sangai deer is listed on Schedule-1 of Wildlife (Protection) Act, 1972, and
as Endangered on IUCN Red List. Consider the following statements
about it.

1) It is the state animal of Manipur and found in its natural habitat only in
Manipur.
2) Introduction of Paragrass in Sangai reserves by Government has helped
recuperate their population.

Select the correct answer using the codes below.


a) 1 only
b) 2 only
c) Both 1 and 2
d) None

Correct Answer : A
Answer Justification :

Justification: Statement 1: The brow-antlered deer or the dancing deer


or Sangai is found in its
natural habitat only at Keibul Lamjao National Park over the floating
biomass locally called "phumdi" in the south-eastern part of Loktak Lake
(Manipur).
Statement 2: Sangai faces threat from steadily degenerating habitat of
phumdi as a result of continuous inundation and flooding caused due to
artificial reservoir. Water quality of the reservoir is degrading due to
pollution and stoppage of nutrient supply. There is also invasion of non-
native plants like Paragrass. There has been decrease in area of phumdi

135. Endangered species of mammals of that are found in Southern India


include

a) Bonnet macaque, Asian palm civet and Indian gray mongoose


b) Indian giant squirrel, Indian leopard and Indian pangolin
c) Golden jackal, leopard cat, jungle cat and spotted deer
d) Indian elephant, Asiatic wild dog, Nilgiri tahr and Lion-tailed macaque

Correct Answer : D
Answer Justification :

Learning: These are all the species found in the Annamalai reserve.
The endangered ones include Bengal tiger, Indian elephant, dhole (Asiatic
wild dog), Nilgiri tahr and lion-tailed macaque.
The vulnerable one are brown mongoose, gaur, Malabar spiny dormouse,
Nilgiri langur, rusty- spotted cat, sambar deer, sloth bear and smooth-
coated otter.

The near threatened ones are Indian giant squirrel, Indian leopard and
Indian pangolin. The rest mentioned in the options are species of least
concern

136. With reference to Lion-tailed Macquae, consider the following


statements.

1) It is endemic to the Western Ghats of South India.


2) It is an endangered species as per IUCN classification.
3) It is ranked among the rarest and most threatened primates.
Which species the above statements refer to?

a) 1 only
b) 2 and 3 only
c) 1 and 3 onl
d) 1, 2 and 3

Correct Answer : D
Answer Justification :

Learning: The lion-tailed macaque or the wanderoo, is an Old World


monkey endemic to the Western Ghats of South India.
It is a good climber and spends majority of its life in the upper canopy of
tropical moist evergreen forests. Unlike other macaques, it avoids
humans.
Their range has become increasingly isolated and fragmented by the
spread of agriculture and tea, coffee, teak and cinchona, construction of
water reservoirs for irrigation and power generation, and human
settlements to support such activities.
They do not live, feed or travel through plantations. Destruction of their
habitat and their avoidance of human proximity have led to the drastic
decrease of their population

137. Consider the following about the Red panda.

1) It is native to the eastern Himalayas and south-western China.


2) It has been classified as endangered by the IUCN.
3) It is highly active during the day.

Select the correct answer using the codes below.

a) 1 only
b) 1 and 3 only
c) 2 and 3 only
d) 1 and 2 only

Correct Answer : D
Answer Justification :

Justification: Statement 1: Distribution of the red panda is disjointed,


with two extant subspecies:
Western red panda lives in the western part of its range, in Nepal, Assam,
Sikkim, and Bhutan.
Styan's red panda lives in the east-north-eastern part of its range, in
southern China and northern Burma.
Statement 2: It is also covered under CITES. Habitat loss, poaching and
hunting are the biggest threats to red panda.
Statement 3: It is a solitary animal, mainly active from dusk to dawn, and
is largely sedentary during the day.
138. The International Union for Conservation of Nature (ILICN) has recently
downgraded the conservation status of this animal from "endangered" to
"vulnerable". Designated as the National Heritage Animal of Pakistan and
Afghanistan, and found in India, it is?

a) Snow leopard
b) Kashmir Stag
c) Blue Sheep
d) Himalayan Tahr

Correct Answer : A
Answer Justification :

Justification: Snow leopard is native to the mountain ranges of Central


and South Asia- including Himalayas, and Russia's remote Altai
mountains.
It is threatened by poaching for their fur, habitat destruction by
infrastructure developments
and climate change.The change in status comes 45 years after snow
leopard was first declared endangered in
1972. However, experts have warned that snow leopard species still face
serious threats from poaching and habitat destruction.
Learning: Endangered Species: According to ILICN, species are
considered 'endangered,' if they are fewer than 2,500 and experiencing
high rate of decline.

Vulnerable Species: Species are fewer than 10,000 and its population has
declined at least 10% over three generations.

139. IUCN does NOT publish a red list of

1) Threatened species
2) Vulnerable Ecosystems
3) Outdated environmental laws
4) Fragile landscapes

Select the correct answer using the codes below.

a) 2, 3 and 4 only
b) 1 and 3 only
c) 3 and 4 only
d) 2 only

Correct Answer : C
Answer Justification :

Justification: Statement 1: The IUCN Red List of Threatened Species™


is the world's most comprehensive inventory of the global conservation
status of plant and animal species. It uses a set of criteria to evaluate the
extinction risk of thousands of species and subspecies.
Statement 2: The IUCN Red List of Ecosystems is a global standard for
how we assess the conservation status of ecosystems. It is applicable at
local, national, regional and global levels.
It is based on a set of rules, or criteria, for performing evidence-based,
scientific assessments of the risk of ecosystem collapse, as measured by
reductions in geographical distribution or degradation of the key
processes and components of ecosystems.

140. Which of the following is/are critically endangered species in India?

1) Indian vulture
2) Golden Mahseer
3) Ganges river dolphin

Select the correct answer using the codes below.

a) 1 only
b) 2 only
c) 1 and 3 only
d) 2 and 3 only

Correct Answer : A
Answer Justification :

Justification: Statement 1: The species has been covered in some of the


previous tests. Statement 2: It has been declared as endangered by IUCN.
It is the longest-living freshwater sigh, and native to mountain and sub-
mountain regions. The Pong Dam reservoir, not very far from Shimla
supports an ample population of the golden mahseer.
It migrates upstream for spawning during the southwest floods. After
spawning, it returns to the original feeding grounds. It is purely
carnivorous.
The range of these fish is from Malaysia, Indonesia, across southern Asia
including the Indian Peninsula and Pakistan.
They are commercially important game fish, as well as highly esteemed
food fish. Mahseer fetches a high market price.
Statement 3: This is endangered.
The Ganges river dolphin is primarily found in the Ganges and
Brahmaputra Rivers and their tributaries in Bangladesh, India and Nepal,
while the Indus river dolphin is found in the Indus River in Pakistan and
its Beas and Sutlej tributaries.
The Ganges river dolphin has been recognized by the government of India
as its National Aquatic Animal. The Indus river dolphin has been named
as the National Mammal of Pakistan. Further, the Ganges river dolphin is
the official animal of the Indian city of Guwahati
CLIMATE CHANGE

141. During winter the vehicles with closed doors and windows remain
warmer than the temperature outside. This may be due to

a) Greenhouse effect
b) Heat Conduction by metallic frame of vehicle
c) Ionizing radiation effect
d) High pollution outside the vehicle

Correct Answer : A
Answer Justification :

Justification: Option C: Ionizing radiation is radiation that carries


enough energy to free electrons from atoms or molecules, thereby ionizing
them. It is not relevant here; hence (c) is also wrong.
Option D: High pollution outside may actually keep vehicle surroundings
warm rather than the opposite. So, (d) is also wrong.
Learning: A greenhouse is made up of glass. The glass which is
transparent to incoming short wave solar radiation is opaque to outgoing
long wave radiation. The glass, therefore, allows in more radiation and
prevents the long wave radiation going outside the glass house, causing
the temperature inside the glasshouse structure warmer than outside.
When you enter a car or a bus, during summers, where windows are
closed, you feel more heat than outside. Likewise during winter the
vehicles with closed doors and windows remain warmer than the
temperature outside.

142. Oil extraction in Arctic may lead to release of

1) Black carbon
2) Methane
3) Carbon Monoxide

Select the correct answer using the codes below.

a) and 3 only
b) 2 only
c) 1 and 2 only
d) 1 only

Correct Answer : C
Answer Justification :

Justification: This is a lateral explanation.


Oil extraction in Arctic may release black carbon and methane leading to
global warming and thawing of Arctic ice. It subsequently causes climate
change. Some studies estimate the cost involved as $60 trillion to the
world economy.
Apart from the usual emission issues from the Arctic Permafrost (a
concept covered in earlier tests), many of the boundary lines in the Arctic
remain to be fully demarcated as under EEZ. Therefore, disputes can arise
on resource extraction in the Arctic.
The short growing season and the low diversity of living organisms make
the Arctic ecosystem vulnerable to global warming, climate change,
exploratory activities etc.

143. As global temperature increases

1) Increased frequency of extreme events like drought will reduce crop


production.
2) Pest attacks may increase on crops.
3) Acidification of oceans will disturb the marine food chain

Select the correct answer using the codes below.

a) 1 and 2 only
b) 2 and 3 only
c) 1 and 3 only
d) 1, 2 and 3

Correct Answer : D
Answer Justification :

Justification: Statement 1 and 2: Climate change is expected to make


water resources scarcer and encourage weeds and pests. Increased
temperatures provide a better breeding ground for many pests. Moreover,
as climates “migrate”, insects and fungi are moving toward the poles too
affecting crops in temperate regions.
Statement 3: Seafood is a large part of many people's diets around the
world. And many marine species can't survive in the more acidic
environment, for e.g. planktons which form the basis of food chain.
You can read this link for better understanding

144. A greenhouse is made up of glass. What is the property of this glass?

a) It is transparent to incoming short wave solar radiation and opaque to


outgoing long wave radiation.
b) It is opaque to incoming short wave solar radiation and transparent to
outgoing long wave radiation.
c) It is transparent to incoming long wave solar radiation and opaque to
outgoing short wave radiation.
d) It is opaque to incoming long wave solar radiation and transparent to
outgoing short wave radiation.
Correct Answer : A
Answer Justification :

Learning: The glass allows in more radiation and prevents the long wave
radiation going outside the glass house, causing the temperature inside
the glasshouse structure warmer than outside.
When you enter a car or a bus, during summers, where windows are
closed, you feel more heat than outside.
Likewise during winter the vehicles with closed doors and windows
remain warmer than the temperature outside.

145. Consider the following statements with reference to the carbon budget in
India.

1) The forestry sector is a net carbon absorber.


2) The energy sector is the highest carbon emitter followed by agriculture.

Which of the above is/are correct?

a) 1 only
b) 2 only
c) Both 1 and 2
d) None

Correct Answer : C
Answer Justification :

Justification: Statement 1: The energy sector accounts for two-thirds the


total emissions, followed by agriculture (20 per cent), industrial processes
(5.8 per cent) and waste (3.4 per cent)
Among gases, CO2 is the largest contributor to GHG emissions at 67.25%,
followed by methane with a contribution of 26.73% and nitrous oxide, 5%.
Statement 2: The forestry sector was a net absorber; carbon absorption by
forests amount to 222,567 million tonnes of CO2 equivalent.

146. The total direct radiative greenhouse effect of Methane is smaller than
that of Carbon Dioxide due to

1) Lower concentration of methane in the atmosphere as compared to CO2


2) Lower direct radiative effect of a fixed mass of methane as compared to
CO2

Which of the above is/are correct?

a) 1 only
b) 2 only
c) Both 1 and 2
d) None
Correct Answer : A
Answer Justification :

Background: The most abundant greenhouse gases in Earth's


atmosphere are: Water vapor, Carbon dioxide, Methane, Nitrous oxide,
Ozone, Chlorofluorocarbons (CFCs).
The contribution of each gas to the greenhouse effect is affected by the
characteristics of that gas, its abundance, and any indirect effects it may
cause.
Justification: The direct radiative effect of a mass of methane is about
72 times stronger than the same mass of carbon dioxide over a 20-year
time frame but it is present in much smaller concentrations so that its
total direct radiative effect is smaller, in part due to its shorter
atmospheric lifetime.
On the other hand, in addition to its direct radiative impact, methane has
a large, indirect radiative effect because it contributes to ozone formation.
Ozone itself is a GHG.

OCEAN ACIDIFICATION

OZONE DEPLETION

147. Peak ground-level ozone concentrations are measured usually in which


part of the day?

a) Early morning
b) Afternoon
c) Late evening
d) Midnight

Correct Answer : B
Answer Justification :

Justification: Ozone (O3) is a colourless, reactive oxidant gas that is a


major constituent of atmospheric smog. Many factors impact ground-level
ozone development, including temperature, wind speed and direction,
time of day, and driving patterns.
Tropospheric, or ground level ozone, is not emitted directly into the air,
but is created by chemical reactions between oxides of nitrogen (NOx) and
volatile organic compounds (VOC).
This happens when pollutants emitted by cars, power plants, industrial
boilers, refineries, chemical plants, and other sources chemically react in
the presence of sunlight. Sunlight is the brightest in the afternoon, hence
B is the correct answer.
Learning: Ozone at ground level is a harmful air pollutant, because of its
effects on people and the environment, and it is the main ingredient in
“smog.”
Ozone can also be transported long distances by wind, so even rural areas
can experience high ozone levels.
Ground level ozone is formed by the reaction of NOx and VOCs under the
influence of sunlight hundreds of kilometers from the source of emissions.
Ozone concentrations are influenced by the intensity of solar radiation,
the absolute concentrations of NOx and VOCs, and the ratio of NOx and
VOCs.
Anaerobic biological processes, lightning, and volcanic activity are the
main natural contributors to atmospheric NOx, occasionally accounting
for as much as 90% of all NOx emissions

148. Consider the following statements. The ozone hole is

1) Caused due to reaction of chlorine monoxide and diatomic oxygen.


2) Produced when Chloro-fluoro Carbons (CFCs) are hit by ultraviolet light

Which of the above is/are correct?

a) 1 only
b) 2 only
c) Both 1 and 2
d) None

Correct Answer : C
Answer Justification :

Justification: Although CFC molecules are several times heavier than


air, winds mix the atmosphere to altitudes far above the top of the
stratosphere much faster than molecules can settle according to their
weight.
CFCs are insoluble in water and relatively unreactive in the lower
atmosphere but are quickly mixed and reach the stratosphere regardless of
their weight.
When UV radiation hits a CFC molecule it causes one chlorine atom to
break away. The chlorine atom then hits an ozone molecule consisting of
three oxygen atoms and takes one of the oxygen molecules, destroying the
ozone molecule and turning it into oxygen.
When an oxygen molecule hits the molecule of chlorine monoxide, the two
oxygen atoms join and form an oxygen molecule. When this happens, the
chlorine atom is free and can continue to destroy ozone.
Naturally occurring chlorine has the same effect in the ozone layer, but
has a shorter life span.
Learning: Lower temperatures favor the formation of polar stratospheric
clouds (PSCs) which serve as platforms for catalytic ozone breakdown.
Unlike the Arctic, the circulation over the Antarctic is more persistent and
vortex-like as a consequence of having less land. So, Antarctica is colder
than the Arctic circle.
Therefore ozone depletion over the South Pole is greater than ozone
depletion over the North Pole.
IMPACT OF CLIMATE CHANGE

149. Melting permafrost would

1) Increase carbon content in the atmosphere


2) Raise sea levels
3) Reduce soil erosion at higher latitudes

Select the correct answer using the codes below.

a) 1 and 2 only
b) 2 and 3 only
c) 2 only
d) 1 and 3 only

Correct Answer : A
Answer Justification :

Concept and Justification: Permafrost is ground, including rock or


soil, at or below the freezing point of water for two or more years.
Most permafrost is located in high latitudes. It also occurs subsea on the
continental shelves of the continents surrounding the Arctic Ocean,
portions of which were exposed during the last glacial period, with global
weather implications
Permafrost can also store carbon, both as peat and as methane.
The amount of carbon sequestered in permafrost is roughly four times the
carbon that has been released to the atmosphere due to human activities
in modern time.
Melting permafrost would raise water levels in the Earth's oceans and
increase erosion. Erosion happens when permafrost melts because soil
and sediment are easily washed away without the ice binding them
together.

150. Consider the following about Microplastics.

1) They are small plastic pieces less than a nanometer long.


2) Bureau of Indian Standards (BIS) has classified them as unsafe for
consumer products.
3) Microbeads are a type of microplastic that are sometimes found in
toothpastes.
4) If dumped in oceans, these pose a potential threat to aquatic life.

Select the correct answer using the codes below.

a) 1 and 4 only
b) 2, 3 and 4 only
c) 1 and 3 only
d) 2 and 4 only

Correct Answer : B

Answer Justification :

Justification: Statement 1 and 4: These are plastic pieces less than


five millimeters long which if engulfed by marine life can pose grave
danger to them. Possibilities are even more since they easily pass
through water filtration systems.

Microplastics come from a variety of sources, including from larger


plastic debris that degrades into smaller and smaller pieces

Statement 3: Microbeads have emerged as a new form of threat. First


patented in 1972 for use in cleansers.

Due to abrasive nature, microbeads began to replace natural material


like ground almonds, oatmeal and sea salt in the area of cosmetics, e.g.
as exfoliating agents in facewash or in toothpaste.

MITIGATION STRATEGIES

151. Under Joint Implementation, countries with commitments are eligible to


transfer and/or acquire emission reduction units (ERUs) and use them to
meet part of their emission reduction target. This has been provided for
under the

a) Paris Climate Accord


b) Montreal Protocol
c) Kyoto Protocol
d) Convention on Long-Range Transboundary Air Pollution
(LRTAP)

Correct Answer : C
Answer Justification :

Learning: The mechanism known as “joint implementation,” defined in


Article 6 of the Kyoto Protocol, allows a country with an emission
reduction or limitation commitment under the Kyoto Protocol (Annex B
Party) to earn emission reduction units (ERUs) from an emission-
reduction or emission removal project in another Annex B Party, each
equivalent to one tonne of CO2, which can be counted towards meeting its
Kyoto target.
Joint implementation offers Parties a flexible and cost-efficient means of
fulfilling a part of their Kyoto commitments, while the host Party benefits
from foreign investment and technology transfer.

A JI project must provide a reduction in emissions by sources, or an


enhancement of removals by sinks, that is additional to what would
otherwise have occurred. Projects must have approval of the host Party
and participants have to be authorized to participate by a Party involved in
the project.

152. Compensatory Afforestation Fund Management and Planning Authority


(CAMPA) was established

a) on the directions of the Supreme Court in 2009


b) by a UNEP Project in India in 2012
c) with India’s accession to the Convention on Biological Diversity (CBD)
d) after the ratification of the Cartagena and Nagoya protocol

Correct Answer : A
Answer Justification :

Background: Many development and industrial projects such as


erection of dams, mining, and construction of industries or roads require
diversion of forest land. Any project proponent, government or private
must apply for forest clearance from Ministry of Environment and Forests
(MoEF), before the conversion of land take place.
This proposal is to be submitted through the concerned forest department
of the state government. If clearance is given, then compensation for the
lost forest land is also to be decided by the ministry and the regulators.
Due to certain discrepancies in the implementation of compensatory
afforestation, some NGOs had approached the SC for relief.
The SC in 2009 issued orders that there will be a Compensatory
Afforestation Fund Management
and Planning Authority (CAMPA) as National Advisory Council under the
chairmanship of the Union Minister of Environment & Forests for
monitoring, technical assistance and evaluation of compensatory
afforestation activities.
Learning: National CAMPA Advisory Council has been established as per
orders of The Hon’ble Supreme Court with the following mandate:
Lay down broad guidelines for State CAMPA.
Facilitate scientific, technological and other assistance that may be
required by State CAMPA.
Make recommendations to State CAMPA based on a review of their plans
and programmes.
Provide a mechanism to State CAMPA to resolve issues of an inter-state or
Centre-State character.
The State CAMPA would presently receive funds collected from user
agencies towards compensatory afforestation, additional compensatory
afforestation, penal compensatory afforestation, Net Present Value (NPV)
and all other amounts recovered from such agencies under the Forest
(Conservation) Act, 1980 and presently lying with the Adhoc CAMPA.

153. What is/are the qualification for a habitat to be called a Green Habitat?
concept:

1) Should be located near a region denoted as forest or classified forest by the


government
2) Should not consume groundwater for its water usage
3) Should use natural sunlight and wind to maximum effectiveness

Select the correct answer using the codes below.

a) 1 and 2 only
b) 1 and 2 only
c) 3 only
d) 1 and 3 only

Correct Answer : B
Answer Justification :

Justification: The buildings, under the concept, are planned to be


designed in such a way that natural sunlight and wind would be used to
maximum and the usage of electricity reduces to the minimum.
The Kerala government is considering promoting a ‘green habitat’ concept
giving thrust to eco- friendly and reusable building materials, natural
water storage and solid waste management. The
objective of the concept is to introduce maximum eco-friendly and
reusable materials in the construction.
Statement 1: There is no such requirement, a green habitat can be within a
city as well. Statement 2: The habitat can use groundwater provided the
use is sustainable.
However, A rainwater harvesting or natural water storage system would
be a sure feature of such ‘green’ buildings, where even kitchen and
drainage water would be recycled and reused for other household
purposes.
154. Consider the following statements.

1) A certified emission reduction (CER) is generated from a clean


development mechanism (CDM) project activity.
2) CERs can be purchased only from the country that generated the
emissions reduction potential.

Which of the above is/are correct?

a) 1 only
b) 2 only
c) Both 1 and 2
d) None

Correct Answer : A
Answer Justification :

Justification: Statement 1: The Clean Development Mechanism (CDM),


defined in Article 12 of the Protocol, allows a country with an emission-
reduction or emission-limitation commitment under the Kyoto Protocol
(Annex B Party) to implement an emission-reduction project in
developing countries. Such projects can earn saleable certified emission
reduction (CER) credits, each equivalent to one tonne of CO2, which can
be counted towards meeting Kyoto targets.
A CDM project activity might involve, for example, a rural electrification
project using solar panels
or the installation of more energy-efficient boilers.
Statement 2: CERs can be purchased from the primary market (purchased
from an original party that makes the reduction) or secondary market
(resold from a marketplace).
At present, most of the approved CERs are recorded in CDM Registry
accounts only. It is only when the CER is actually sitting in an operator's
trading account that its value can be monetized through being traded. The
UNFCCC's International Transaction Log has already validated and
transferred CERs into the accounts of some national climate registries
155. In climate change negotiations, the principle of Common but
Differentiated Responsibilities (CBDR) is NOT based on?

a) Historical emission responsibility of developed nations


b) Need of sustainable development in developing countries
c) Principle of Equity in international affairs
d) It is based on all of the above

Correct Answer : D
Answer Justification :

Justification: Option A: The Incremental impact of a ton of GHG on


climate change is independent of where in the world it is emitted. These
emissions impose a cost on both the present and future generations, which
are not fully recouped from the emitters of these emitters, which
historically were developed nations.
Option B and C- Developed nations must take greater share of the
responsibility because developing cannot sacrifice economic development
focussing solely on environmental conservation. There needs to be a
balance based on the principles of sustainable development.
This formed the starting point for a globally coordinated policy action and
the need for an International climate change negotiation regime as in
UNFCCC.

156. The Global Carbon Project (GCP) seeks to

1) Quantify global carbon emissions and their causes


2) Sequester global carbon generation using geo-engineering

Which of the above is/are correct?

a) 1 only
b) 2 only
c) Both 1 and 2
d) None

Correct Answer : A
Answer Justification :

Justification: It was established in 2001. The scientific goal of the


Global Carbon Project is to develop a complete picture of the global
carbon cycle, including both its biophysical and human dimensions
together with the interactions and feedbacks between them.
This will be:
Patterns and Variability: What are the current geographical and temporal
distributions of the major pools and fluxes in the global carbon cycle?
Processes and Interactions: What are the control and feedback
mechanisms - both anthropogenic and non-anthropogenic - that
determine the dynamics of the carbon cycle?
Carbon Management: What are the dynamics of the carbon-climate-
human system into the future, and what points of intervention and
windows of opportunity exist for human societies to manage this system?

Learning: The Global Carbon Project is responding to this challenge


through a shared partnership between the International Geosphere-
Biosphere Programme (IGBP), the International Human Dimensions
Programme on Global Environmental Change (IHDP), the World Climate
Research Programme (WCRP) and Diversitas. This partnership
constitutes the Earth Systems Science Partnership (ESSP

157. Fly ash is becoming a popular construction material. What is/are the
environmental benefits of fly ash utilization in concrete?

1) Reduction in amount of coal combustion products that must be disposed


in landfills
2) Net reduction in energy use and greenhouse gas emissions when fly ash is
used to partly replace or displace manufactured cement
Which of the above is/are correct?

a) 1 only
b) 2 only
c) Both 1 and 2
d) None

Correct Answer : C
Answer Justification :

Justification: Fly ash is the finely divided residue that results from the
combustion of pulverized coal and is transported from the combustion
chamber by exhaust gases.

Fly ash is produced by coal-fired electric and steam generating plants.


Even though its production causes a lot of environmental hazards, its
correct use can yield for e.g. fly ash utilization, especially in concrete, has
significant environmental benefits.

These lead to: (1) increase in the life of concrete roads and structures by
improving concrete durability, (2) net reduction in energy use and
greenhouse gas and other adverse air emissions when fly ash is used to
replace or displace manufactured cement, (3) reduction in amount of coal
combustion products that must be disposed in landfills, and (4)
conservation of other natural resources and materials.

158. The scope of World Climate Services Programme (WCSP) includes

1) Climate data and analysis


2) Climate adaptation and risk management
3) Climate mitigation finance
4) Climate monitoring and prediction

Select the correct answer using the codes below.

a) 1 and 2 only
b) 1 and 4 only
c) 1, 2 and 4 only
d) 1, 3 and 4 only

Correct Answer : C
Answer Justification :

Justification: WCSP contributes to improve the availability and access


to reliable data, advancement of the knowledge in the area of climate data
management and climate analysis, definition of the technical and scientific
standards, and development of activities to support them in countries.
Climate data management will include data rescue, development and
coordination of a global climate data management system compatible with
the WMO Information System (WIS).
The scope of WCSP spans across four inter-related areas: Climate data and
analysis; Climate monitoring, watch and prediction; Climate system
operation and infrastructure; Climate adaptation and risk management;
thereby serving as the Climate Services Information System and a part of
the User Interface Platform components of the GFCS.
The programme would establish and maintain research operations
linkages to expedite the implementation of research advances in
operational climate services and to ensure ongoing improvement to the
operational practices and outcomes.

159. Halting deforestation and increasing green cover is a cost-effective action


that has a clear impact in reducing global GHG emissions. Why?

1) When forests are cleared or degraded, they can become a source of GHG
emissions by releasing the stored carbon in biomass or soil.
2) Farms such as rice fields soak atmospheric methane and reduce the
overall stock of CO2 in the atmosphere.

Which of the above is/are correct?

a) 1 only
b) 2 only
c) Both 1 and 2
d) None

Correct Answer : A
Answer Justification :

Justification: Statement 1: Forests play a fundamental role they play in


climate change mitigation, by removing CO2 from the atmosphere and
storing it in biomass and soils.
This also means that when forests are cleared or degraded, they can
become a source of greenhouse gas (GHG) emissions by releasing that
stored carbon.
It is estimated that globally, deforestation and forest degradation account
for around 11 percent of CO2 emissions. Halting deforestation is a cost-
effective action that has a clear impact in reducing global GHG emissions.
FAO’s support also helps countries during the later implementation phase
of REDD .
Statement 2: Rice fields actually emit methane, and more rice farms lead
to an increase in the methane GHG emissions, even if they soak some of
the CO2
160. Carbon Capture Utilization Storage (CCUS) is the process of

a) Capturing waste carbon dioxide from large point sources and reusing it
b) Redistributing captured carbon from storage in the atmosphere to
mitigate de-calcification in marine sources
c) Generating carbon certificates from carbon-intensive activities and issuing
them based on the carbon footprint associated with each such activity
d) Destroying atmospheric CO2 by way of electrostatic precipitators and
storing the residue in specialized storages

Correct Answer : A
Answer Justification :

Learning: As part of its climate change commitments in the aftermath of


signing the Paris agreement India is now exploring the possibility of
carbon capture utilization storage or CCUS.
Carbon capture utilization storage is the process of capturing waste carbon
dioxide (CO2) from large point sources, such as fossil fuel power plants,
and either transporting it to a storage site where it will not enter the
atmosphere, normally an underground geological formation , or reusing it.
In Carbon Capture and Storage (CCS), emissions are forced into
underground rocks at great cost and no economic benefit while CCUS
aims at using CO2 emissions by exploiting the resource itself and creating
new markets around it.

161. Which of the following bacterium may be used for bio-remediation


operations?

1) Haemophilus
2) Mycobacterium
3) Rhodococcus
4) Ralstonia

Select the correct answer using the codes below.

a) 1 and 2 only
b) 2, 3 and 4 only
c) 1 and 3 only
d) 1, 2, 3 and 4

Correct Answer : D
Answer Justification :

Learning: Bacteria are widely diverse organisms, and thus make


excellent players in biodegradation and bioremediation. There are few
universal toxins to bacteria, so there is likely an organism able to break
down any given substrate, when provided with the right conditions
(anaerobic versus aerobic environment, sufficient electron donors or
acceptors, etc.).
Fast development of molecular microbiological tool has enabled the
identification of many un-
culturable microbes and therefore extended the list of microbial species
with petroleum hydrocarbon degrading abilities.
Species of Pseudomonas, Mycobacterium, Haemophilus, Rhodococcus,
Paenibacillus and Ralstonia, are some of the most extensively studied
bacteria for their bioremediation capability

CLIMATE CHANGE ORGANISATION

162. Consider the following statements.

1) The Special Climate Change Fund (SCCF) was established under the
UNFCCC to finance projects relating to adaptation, technology transfer
and capacity building.
2) Developed countries are contributing to the ‘fast-start finance’ (FSF) as a
step towards mobilizing USD 1 trillion per year from public and private
sources by 2020.

Which of the above is/are correct?

a) 1 only
b) 2 only
c) Both 1 and 2
d) None

Correct Answer : A
Answer Justification :

Justification: Statement 1: The Global Environment Facility (GEF), as


an operating entity of the Financial Mechanism, has been entrusted to
operate the SCCF.

It was established in 2001 to finance projects relating to: adaptation;


technology transfer and capacity building; energy, transport, industry,
agriculture, forestry and waste management; and economic
diversification.

This fund should complement other funding mechanisms for the


implementation of the Convention.

Statement 2: The amount is USD 100 billion, which was a major demand
from developing countries for many COP meetings. Gradually it has been
acceded in the form of new and additional ‘fast-start finance’ (FSF)
between 2010-2012 to developing countries and other funds.
163. With reference to the United Nations Framework Convention on Climate
Change (UNFCCC), consider the following statements.

1) It is the parent treaty of the 1997 Kyoto Protocol.


2) The framework sets binding limits on greenhouse gas emissions for
individual countries.
3) It contains no enforcement mechanisms.

Select the correct answer using the codes below.

a) 1 only
b) 2 and 3 only
c) 1 and 3 only
d) 1, 2 and 3

Correct Answer : C
Answer Justification :

Justification: With 197 Parties, the United Nations Framework


Convention on Climate Change (UNFCCC) has near universal membership
and is the parent treaty of the 1997 Kyoto Protocol. The Kyoto Protocol has
been ratified by 192 of the UNFCCC Parties.
The ultimate objective of both treaties is to stabilize greenhouse gas
concentrations in the atmosphere at a level that will prevent dangerous
human interference with the climate system.
It is an international environmental treaty adopted in 1992 at the Earth
Summit in Rio de Janeiro.
Statement 2: The framework sets non binding limits on greenhouse gas
emissions for individual countries and contains no enforcement
mechanisms. Instead, the framework outlines how specific international
treaties (called "protocols" or "Agreements") may be negotiated to specify
further action towards the objective of the UNFCCC.
One of the first tasks set by the UNFCCC was for signatory nations to
establish national greenhouse gas inventories of greenhouse gas (GHG)
emissions and removals, which were used to create the 1990 benchmark
levels for accession of Annex I countries to the Kyoto Protocol and for the
commitment of those countries to GHG reductions.
Updated inventories must be submitted annually by Annex I countries.
164. Parties to the UNFCCC are classified as Annex I, Annex II and Annex B
countries. Consider the following about it.

1) Annex I countries are industrially developed countries.


2) Annex II countries are required to provide financial and technical support
to developing countries for climate change mitigation.
3) Annex B countries are Least Developed Countries (LDCs) or developing
island economies.

Select the correct answer using the codes below.

a) 1 and 2 only
b) 2 and 3 only
c) 1 and 3 only
d) 1, 2 and 3

Correct Answer : A
Answer Justification :

Justification: Statement 1: Annex I: There are over 40 Parties to the


UNFCCC listed in Annex I of the Convention, including the European
Union. These Parties are classified as industrialized (developed) countries
and "economies in transition" (EITs).
The EITs are the former centrally-planned (Soviet) economies of Russia
and Eastern Europe.
Statement 2: Annex II: Of the Parties listed in Annex I of the Convention,
over 20 are also listed in Annex II of the Convention, including the
European Union.
These Parties are made up of members of the Organization for Economic
Cooperation and Development (OECD). Annex II Parties are required to
provide financial and technical support to the EITs and developing
countries to assist them in reducing their greenhouse gas emissions
(climate change mitigation) and manage the impacts of climate change
(climate change adaptation).
Statement 3: Parties listed in Annex B of the Kyoto Protocol are Annex I
Parties with first- or second-round Kyoto greenhouse gas emissions
targets. The first-round targets apply over the years 2008–2012.
165. Consider the following statements.

1) Green Initiatives for Future transport (GIFT) aims to develop and


demonstrate hydrogen powered IC engine and fuel cell based vehicles.
2) Green Initiative for Power Generation (GIP) envisages developing fusion
reactors for supplying on- grid power to the metropolitan areas.

Which of the above is/are correct?

a) 1 only
b) 2 only
c) Both 1 and 2
d) None

Correct Answer : A
Answer Justification :

Justification: Hydrogen is suitable for meeting decentralised energy


needs of the country’s population. It can be used for telecom tower for
providing back up power in rural areas, vehicular transportation and
Industrial applications
In this light, the National Hydrogen Energy Road Map (2006) is an
industry driven planning process that offers long-term energy solutions to
the growing energy needs of India, while ensuring energy security for the
country.
The Road Map has identified two major initiatives; namely:
Green Initiatives for Future transport (GIFT) - It aims to develop and
demonstrate hydrogen powered IC engine and fuel cell based vehicles
ranging from small two/three wheelers to heavy
vehicles through different phases of development.
Green Initiative for Power Generation (GIP) - It envisages developing and
demonstrating hydrogen powered IC engine/turbine and fuel cell based
decentralized power generating systems ranging from small watt capacity
to MW size systems.

166. Operation IceBridge is a 2009–2018 NASA mission that aims to

1) Monitor changes in polar ice from a satellite


2) Create physical underground networks for clutter free movement on
frozen surfaces

Which of the above is/are correct?

a) 1 only
b) 2 only
c) Both 1 and 2
d) None

Correct Answer : D
Answer Justification :

Justification: It was to monitor polar ice changes, but it is not being


done through a satellite, but instead a fixed-wing aircraft.
Since 2003 NASA has used a satellite, ICESat (Ice, Cloud, and land
Elevation Satellite), for observing polar ice. ICESat was retired in 2009
due to a technical malfunction, leaving NASA without a satellite dedicated
to ice observance.
NASA therefore introduced the IceBridge program which utilizes an
aircraft to make similar measurements.
Learning: IceBridge flights began in 2009. There are tradeoffs to using
an aircraft instead of a satellite.
One drawback is that a satellite can observe a far wider area. Also,
satellites take measurements full-time, while IceBridge aircraft
measurements are limited to annual campaigns that are several weeks
long.
Aircraft, however, have the advantages of being able to carry more
instruments and target and focus on scientifically interesting areas instead
of just flying a fixed path.
167. Montreal Protocol is an international treaty designed to protect ozone
layer by phasing out production of numerous Ozone Depleting Substances
(ODSs) that are responsible for ozone depletion. Under it, apart from
CFCs, production and consumption of which of these key ODSs are
banned?

1) Methyl Chloroform
2) CTC halons
3) Methyl Bromide

Select the correct answer using the codes below.

a) 1 only
b) 2 and 3 only
c) 1 and 2 only
d) 1, 2 and 3

Correct Answer : D
Answer Justification :

Justification & Learning: Due to its universality, Montreal Protocol


has helped in recovering the ozone hole in Antarctica.
All the ODSs mentioned above have been phased-out globally.
Vienna Convention for Protection of the Ozone Layer acts as a framework
for the international efforts to protect the ozone layer. These are laid out
in the accompanying Montreal Protocol.
However, it does not include legally binding reduction goals for the use of
CFCs, the main chemical agents causing ozone depletion.

INDIA AND CLIMATE CHANGE

168. With reference to the ‘Biodiversity Conservation and Rural Livelihood


Improvement (BCRLI) Project’, consider the following statements.

1) It is a joint venture indigenously developed by a consortium of WWF-


India and IUCN-India.
2) It is supported by Global Environment Facility (GEF) and International
Development Agency (IDA).
3) It aims to develop and promote new models of conservation at the
landscape scale.

Select the correct answer using the codes below.

a) 1 and 2 only
b) 2 and 3 only
c) 1 and 3 only
d) 1, 2 and 3
Correct Answer : B
Answer Justification :

Justification: The BCRLIP is a blended Global Environment Facility


(GEF) and International Development Agency (IDA) activity using a
Specific Investment Loan instrument aimed at strengthening biodiversity
conservation and improving rural livelihoods at landscape sites.

The Project was launched in 2011 for a duration of six years.

This externally aided project is being implemented as a Centrally


Sponsored Scheme.

New models of conservation are to be built by enhanced capacity and


institution building for mainstreaming biodiversity conservation
outcomes.

The project has four landscape located at Askot, Uttarakhand; Little Rann
of Kachchh, Gujarat, Satpura, Madhya Pardesh and Maharashtra and
Agasthyamalai, Tamil Nadu and Kerala; and three field learning centre at
Gir, Gujarat; KalakadMundanthurai Tiger Reserve, Tamil Nadu and
Periyar Tiger Reserve, Kerala.

A capacity building centre for the project is at Wildlife Institute of India,


Dehradun.

169. Consider the following statements. Under the scheme ‘Perform Achieve
and Trade (PAT)’

1) Identified industries are required to improve their specific energy


consumption (SEC) within the specified period or face penalty provisions
2) Energy efficient industries are facilitated to trade their additional certified
energy savings with other designated consumers

Which of the above is/are correct?

a) 1 only
b) 2 only
c) Both 1 and 2
d) None

Correct Answer : C
Answer Justification :

Justification: It is a scheme for trading energy-efficiency certificates in


large energy-intensive industries under the National Mission for
Enhanced Energy Efficiency (NMEEE) which is one of the eight missions
under the National Action Plan on Climate Change (NAPCC).
Statement 2: These designated consumers could use these certificates to
comply with their SEC- reduction targets.
Other details under the PAT scheme are technical and need not be
covered.
Lateral Learning: Similarly, the RPO (Renewable Purchase Obligation)
is creating domestic markets for renewable energy through regulatory
interventions at state level.
In RPO there is a minimum level of renewable energy (out of total
consumption) the obligated entities (DISCOMs, Captive Power Plants, and
Open Access Consumers) are entitled to purchase in the area of a
distribution licensee.
The obligation is mandated by the State Electricity Regulatory
Commission (SERC). Since the renewable energy sources are not evenly
spread across India, SERCs cannot specify a linear level of RPOs for all
states.
Renewable Energy Certificates (RECs) under the RPO mechanism
is an instrument that enables the obligated entities to meet their
Renewable Purchase Obligation by trading surplus or deficit RECs among
themselves with the owner of the REC being able to claim to have
purchased renewable energy.

170. “Green Good Deeds” campaign, recently in news, has been launched by

a) WWF-India
b) Union Ministry of Environment, Forests and Climate Change
c) Greenpeace India
d) Conservation International – India Chapter

Correct Answer : B
Answer Justification :

Learning: The campaign has been launched by the Environment


Ministry to sensitise the people and students, in particular, about climate
change and global warming.

The objective of the campaign is to restore and return the clean and green
environment to the next generation.

The plan is to broad-base it with the involvement of teachers, students and


other voluntary organisations by emphasizing the idea of “Green Social
Responsibility” (GSR), similar to Corporate Social Responsibility (CSR).
171. What is/are the major difference(s) in the standards between the existing
BS-IV and new BS-VI auto fuel norms?

1) Reduced sulphur content in BS-VI


2) BS-VI includes Particulate Matter (PM) emissions from Petrol vehicles
which were not included under BS-IV
3) CO2 emissions, which were ignored under BS-IV, have been included in
BS-VI.

Select the correct answer using the codes below.

a) 1 only
b) 1 and 3 only
c) 1, 2 and 3
d) 2 and 3 only

Correct Answer : A
Answer Justification :

Background: BS norms introduced in 2000 are emission control


standards introduced by the government, based on European emission
standards, to keep a check and control over release of air pollutants from
equipment using internal combustion engines including vehicles.
Oil retailers introduced the BS-IV grade auto fuel across the country in
April 2017 and planned to leapfrog to BS-VI grade fuel by April 2020.
Justification: Statement 1: BS-IV fuels contain 50 parts per million
(ppm) sulphur, while BS-V and BS-VI grade fuel will have 10 ppm
sulphur. Thus, newly introduced BS VI fuel is estimated to reduce amount
of sulphur released by 80%.
It is also estimated to bring down the emission of NOx (nitrogen oxides)
from diesel cars by nearly
70 % and 25% from cars with petrol engines. It will also bring down cancer
causing particulate matter emissions from diesel engine cars by an
estimated 80%.
Statement 2: They are included in both norms.
Statement 3: India's auto sector accounts for about 18% of the total CO2
emissions in the country. Relative CO2 emissions from transport have
risen rapidly in recent years, but like the EU, currently there are no
standards for CO2 emission limits for pollution from vehicles.
There is also no provision to make the CO2 emissions labelling mandatory
on cars in the country
172. Consider the following statements. The Oil and Gas Climate Initiative
(OGCI) is

1) An inter-governmental initiative as a part of the post-Paris agenda


2) Under the auspices of the United nations Environment Programme
(UNEP)
3) Serving as a platform to collaboratively advance technological solutions
for meaningful action on climate change

Select the correct answer using the codes below.

a) 1 Only
b) 3 only
c) 1 and 3 only
d) 1, 2 and 3

Correct Answer : B
Answer Justification :

Justification and learning: The OGCI is comprised of 10 oil and gas


companies (BP, CNPC, Eni, Pemex, Petrobras, Repsol, Saudi Aramco,
Shell, Statoil, and Total). Together, they represent more than one fifth of
the global oil and gas production. CEOs are personally invested in steering
and leading the initiative.
It is a bottom-up, voluntary, industry-driven initiative, which will enable
the Oil and Gas industry to work collaboratively to address climate
concerns.
It is an unprecedented and unique collaboration. The initiative serves as a
platform to collaboratively advance technological solutions and to catalyse
meaningful action and coordination on climate change.
This is the only initiative to provide a full spectrum on what the sector is
currently doing about climate change, and what it is prepared to do,
collaboratively, going forward.

173. The project SECURE Himalaya has been launched by

a) Union Environment ministry in association with the United Nations


Development Programme (UNDP)
b) World Wildlife Fund (WWF) in collaboration with UNESCO
c) International Union for Conservation of Nature (IUCN)
d) United Nations Environment Programme (UNEP) jointly with IUCN

Correct Answer : A
Answer Justification :

Learning: The Centre has launched a six-year project- called SECURE


Himalaya– to ensure conservation of locally and globally significant
biodiversity, land and forest resources in the high Himalayan ecosystem
spread over four states in India.
The SECURE – securing livelihoods, conservation, sustainable use and
restoration of high range Himalayan ecosystems – is meant for specific
landscapes.
It includes Changthang (Jammu and Kasmir), Lahaul – Pangi and
Kinnaur (Himachal Pradesh), Gangotri – Govind and Darma – Byans
Valley in Pithoragarh (Uttarakhand) and Kanchenjunga
– Upper Teesta Valley (Sikkim).
Benefits: Protection of snow leopard and other endangered species and
their habitats is one of the key components of the project which will also
focus on securing livelihoods of the people in the region and enhancing
enforcement to reduce wildlife crime.
Enhanced enforcement efforts and monitoring under the project will also
curb illegal trade in some medicinal and aromatic plants which are among
the most threatened species in these landscapes.

174. Consider the following about The Partnership for Land Use Science
(Forest-Plus).

1) It is a joint programme by the UNEP and IUCN for developing countries.


2) It works to strengthen capacity for REDD (Reducing Emissions from
Deforestation and Forest Degradation) implementation in India.

Which of the above is/are correct?

a) 1 only
b) 2 only
c) Both 1 and 2
d) None

Correct Answer : B
Answer Justification :

Justification: It is a joint programme by the United States Agency for


International Development (USAID) and Ministry of Environment, Forest
and Climate Change (MoEF&CC) to strengthen capacity for REDD.
The programme brings together experts from India and the United States
to develop technologies, tools and methods of forest management to meet
the technical challenges of managing forests for the health of ecosystem,
carbon stocks, biodiversity and livelihood

175. WWF-India’s Small Grants Innovation Program (SGIP) grants to


individuals for activities and/or research that addresses issues such as

1) Enabling communities and other stakeholders to address local


environmental concerns
2) Aspects of trade involving wildlife species
3) Development of Entrepreneurship based Self-help groups

Select the correct answer using the codes below.


a) 1 only
b) 1 and 2 only
c) 2 and 3 only
d) 1, 2 and 3

Correct Answer : B
Answer Justification :

Justification: With the aim of encouraging young Indians to respond


innovatively and independently to the conservation issues which affect the
country, WWF-India is offering eligible individuals a one-time grant of
upto INR 400,000 over a maximum period of 2 years for undertaking
conservation research/ action research.
The grants will be used to fund research and initiatives which especially
demonstrate innovation that is replicable, scalable and sustainable.
Around the globe, conservationists are employing the latest technological
advances to make a difference for people, wildlife, oceans, forests and
clean water.
The SGIP will provide grants to individuals for activities and/or research
that address issues and off- er solutions or insights towards:
Species and habitat related problems and concerns – with a focus on
immediate threats and issues
Enabling communities and other stakeholders to address local
environmental concerns
Improving local livelihoods through conservation and natural resource
management or promoting livelihoods that reduce impacts on biodiversity
Aspects of trade involving wildlife species
Increasing understanding on the status of lesser known or lesser studied
species of wildlife
Innovative approaches to awareness raising regarding environmental
concerns
Demonstrating individual or collective action towards conservation
outcomes

176. Consider the following statements.

1) BioCarbon Fund Initiative for Sustainable Forest Landscapes (ISFL)


promotes reducing GHG emissions from the land sector by helping tackle
forest degradation.
2) The Carbon Initiative for Development (Ci-Dev), managed by WWF, helps
the world’s poorest countries access carbon finance, mainly in the area of
energy access.

Which of the above is/are correct?

a) 1 only
b) 2 only
c) Both 1 and 2
d) None
Correct Answer : A
Answer Justification :

Justification: Statement 1: The BioCarbon Fund Initiative for


Sustainable Forest Landscapes (ISFL) is a multilateral fund, supported by
donor governments and managed by the World Bank.
It promotes reducing greenhouse gas emissions from the land sector, from
deforestation and forest degradation in developing countries (REDD ),
and from sustainable agriculture, as well as smarter land-use planning,
policies and practices.
ISFL can transform rural areas by protecting forests, restoring degraded
lands, enhancing agricultural productivity, and by improving livelihoods
and local environments.
Statement 2: The Carbon Initiative for Development (Ci-Dev) was
launched in 2011 by World Bank to build capacity and develop tools and
methodologies to help the world’s poorest countries access carbon finance,
mainly in the area of energy access.
It is set up to use performance payments based on reduced emissions to
support projects that use clean and efficient technologies in low-income
countries.
The Ci-Dev will build on the infrastructure that has been created by the
Clean Development Mechanism (CDM)

177. The Union Government signed the Global Environment Facility (GEF)
Grant Agreement of the World Bank for Ecosystem Service Improvement
Project. It has which of the following objectives?

1) Improve the livelihoods of forest dependent communities


2) Strengthen the institutional capacity of the Community Organisations and
Departments of Forestry to enhance forest ecosystem services

Which of the above is/are correct?

a) 1 only
b) 2 only
c) Both 1 and 2
d) None

Correct Answer : C
Answer Justification :

Justification: The Project will be entirely financed by the World Bank


out of its GEF Trust Fund. The project’s duration is of five years.
The project will be implemented by the Union Ministry of Environment,
Forest and Climate Change (MoEF&CC) in Chhattisgarh and Madhya
Pradesh through Indian Council of Forestry Research & Education
(ICFRE) under the National Green India Mission (GIM).
The objective is to strengthen the institutional capacity of the Community
Organisations and Departments of Forestry to enhance forest ecosystem
services and improve the livelihoods of forest dependent communities in
Central Indian Highlands.
The Project will support the Government of India’s Green India Mission’s
(GIM) goal of protecting, restoring and enhancing India’s forest cover and
responding to climate change.

178. SECURE Himalaya initiatives of the Union environment ministry in


association with the United Nations Development Programme (UNDP)
focuses on

1) Protecting snow leopard and other endangered species and their habitats
2) Securing livelihoods of the people in the region
3) Enhancing enforcement to reduce wildlife crime

Select the correct answer using the codes below.

a) 1 only
b) 2 and 3 only
c) 1 and 3 only
d) 1, 2 and 3

Correct Answer : D
Answer Justification :

Justification: The Centre has launched a six-year project- called


SECURE Himalaya– to ensure conservation of locally and globally
significant biodiversity, land and forest resources in the high Himalayan
ecosystem spread over four states in India.
The SECURE – securing livelihoods, conservation, sustainable use and
restoration of high range Himalayan ecosystems – is meant for specific
landscapes.
It includes:
Changthang (Jammu and Kasmir),
Lahaul – Pangi and Kinnaur (Himachal Pradesh),
Gangotri – Govind and Darma – Byans Valley in Pithoragarh
(Uttarakhand) and
Kanchenjunga – Upper Teesta Valley (Sikkim)
Protection of snow leopard and other endangered species and their
habitats is one of the key components of the project
It will also focus on securing livelihoods of the people in the region and
enhancing enforcement to reduce wildlife crime.

179. Under National Adaptation Fund for Climate Change (NAFCC)

1) Cent per cent central grant is given to the State Governments for
implementing climate change adaptation projects
2) United Nations Environment Programme (UNEP) and associated
agencies have committed to contributing fifty per cent of the budget

Which of the above is/are correct?


a) 1 only
b) 2 only
c) Both 1 and 2
d) None
Correct Answer : A

Answer Justification :

Justification: Statement 1: Under this scheme, Union Government


encourages States to come up with innovative and scalable projects to
develop resilience against climate change and mainstream it in the
planning processes.

The objective of the fund is to assist states/UTs that are particularly


vulnerable to the adverse effects of climate change in meeting the cost
of adaptation.

Statement 2: There is no such provision.

Learning: It is a flagship Scheme of Union Government launched in


2015.

The Scheme has been designed to fulfill the objectives of National


Action Plan on Climate Change (NAPCC) and operationalize the State
Action Plans on Climate Change (SAPCCs).

NABARD is the National Implementing Entity (NIE) responsible for


implementation of adaptation projects under the NAFCC.

AGRICULTURE

180. Agro-forestry is

a) Altering the agriculture land use category to forest category


b) Clearing forests for slash and burn agriculture
c) Raising of trees and agriculture crops on the same land inclusive of the
waste patches
d) Harnessing the practices of local tribal communities in raising crops
inside a forest
Correct Answer : C
Answer Justification :

Learning: It combines forestry with agriculture, thus, altering the


simultaneous production of food, fodder, fuel, timber and fruit.
Community forestry involves the raising of trees on public or community
land such as the village pasture and temple land, roadside, canal bank,
strips along railway lines, and schools etc.
It maintains or increases total yields by combining food crops (annuals)
with tree crops (perennials) and/or livestock on the same unit of land,
either alternately or at the same time, using management practices that suit
the social and cultural characteristics of the local people and the economic
and
ecological conditions of the area.

181. With reference to Contour bunding, consider the following statements.

1) It is the practice of plowing or planting across a slope following its


elevation contour lines.
2) Bunds area can be constructed along field boundaries without reference to
contour.
3) The most suitable soil for building Contour bunds are deep black soils.
4) Coutour bunding is not practiced in Peninsular and Southern India owing
to the hard terrain structure.

Select the correct answer using the codes below.

a) 1 only
b) 1 and 2 only
c) 1, 2 and 3 only
d) 2, 3 and 4 only

Correct Answer : B
Answer Justification :

Justification: Contour bunds can save soils from erosion to the


extent of hundreds of tones/ hectare annually. It maintains soil fertility
and increases water infiltration into the soil considerably,
Statement 1: It consists of building earthen embankments across the slope
of the land, following the contour as closely as possible. A series of such
bunds divide the area into strips and act as barriers to t5he flow of water,
thus reducing the amount and velocity of the runoff.
Statement 2: Bunds area also constructed along field boundaries without
reference to contour. These bunds are called peripheral bunds. They serve
as fences, and give protection from water and wind erosion in low rainfall
areas. They are not suitable in heavy rainfall areas.
No cultivation is allowed on the earthen embankments of contour bunds.
Therefore, under contour bunds an area of about 5 percent is lost under
the bunds and is not available for cultivation.
Statement 3: Contour bunds in deep black soils have been a failure
because of the nature of soil, which cracks during hot weather and cakes
during the monsoon. So they are not stable in black
soils. Further the poor drainage properties of deep black soils gives raise
to long stagnation of water against contour bunds and make it unstable.
Contour bunds are also not successful in very shallow soils having a depth
loss than 7.5cm.
Statement 4: Counter Bunding are carried out in many parts in India-
notably in Maharashtra, Gujarat, Tamilnadu, Karnataka and Andhra
Pradesh.

182. As defined by FAO, Globally Important Agricultural Heritage Systems


(GIAHS) are “remarkable land use systems and landscapes which are rich
in globally significant biological diversity evolving from the co-adaptation
of a community with its environment and its needs and aspirations for
sustainable development”. GIAHS sites are expected to fulfil which of the
following criteria?

1) Food and Livelihood Security


2) Cultures, Value systems and Social Organisations
3) Intensive integration of technology into traditional farming

Select the correct answer using the codes below.

a) 1 only
b) 1 and 2 only
c) 2 and 3 only
d) 1, 2 and 3

Correct Answer : B
Answer Justification :

Justification: Statement 1: Over centuries, generations of farmers, fisher


folks and herders have developed complex, diverse and locally adapted
agricultural systems with time-tested technologies.
Building on generations of accumulated knowledge and experience by
smallholders, family farming and indigenous communities, these systems
have been adapted to ever changing environment and climate conditions
which finally have acquired resilience and robustness so as to ensure food
and livelihood security in the local communities and reduce risks.
Statement 2: GIAHS sites are expected to fulfil the following criteria which
well demonstrate the characteristics of GIAHS which focus agricultural
production as a basis and has both tangible and intangible effects.
Food and Livelihood Security
Agro-biodiversity
Local and Traditional Knowledge systems
Cultures, Value systems and Social Organisations
Landscapes and Seascapes Features
The overall goal of the global initiative is to identify and safeguard
Globally Important Agricultural Heritage Systems and their associated
landscapes, agricultural biodiversity and knowledge systems

183. Consider the following about the Guar bean.

1) It lives in symbiosis with nitrogen-fixing bacteria.


2) Its demand is rising rapidly due to industrial use of guar gum in hydraulic
fracturing of oil shale gas.
3) It requires extensive irrigation facility for survival and maturity.
4) Most of the Guar bean production is from India and Pakistan.

Select the correct answer using the codes below.

a) 1, 2 and 4 only
b) 2 and 4 only
c) 1 and 3 only
d) 1, 2 and 3 only

Correct Answer : A
Answer Justification :

Justification: Guar bean is the source of Guar gum.


This legume is a very valuable plant within a crop rotation cycle, as it lives
in symbiosis with nitrogen-fixing bacteria.
Statement 3: Frequent drought periods can lead to delayed maturation.
On the contrary, too much moisture during early phase of growth and
after maturation lead to lower seed quality.
In fact, agriculturists in semi-arid regions of Rajasthan follow crop-
rotation and use guar as a source to replenish the soil with essential
fertilizers and nitrogen fixation, before the next crop
Demand is rising rapidly due to industrial use of guar gum in hydraulic
fracturing (oil shale gas).
About 80% of world production occurs in India and Pakistan
Guar is very drought-tolerant and sun-loving, but it is very susceptible to
frost.
Even though it can cope with little but regular rainfall, it requires
sufficient soil moisture before planting and during maturation of seeds.
Guar is also produced near to coastal areas in the Gandhidham region of
Kutch, Gujarat.
Derivatives of guar gum that has been further reacted is also used in
industrial applications, such as the paper and textile industry, ore
flotation, the manufacture of explosives and hydraulic fracturing
(fracking) of oil and gas formations.
184. Contour Bunding is the farming practice of planting across a slope
following its elevation contour lines. This has the advantage of

1) Reducing the formation of rills and gullies during times of heavy water
run-off
2) Allowing no time for the water to settle into the soil

Which of the above is/are correct?

a) 1 only
b) 2 only
c) Both 1 and 2
d) None

Correct Answer : A
Answer Justification :

Justification: Statement 1: These contour lines create a water break


which reduces the formation of rills during times of water run-off; which
is a major cause of soil erosion.
Statement 2: The water break lets water settle more easily into the soil,
allowing for higher productivity.
Soil erosion prevention practices such as this can drastically decrease
negative effects associated with soil erosion such as reduced crop
productivity, worsened water quality, lower effective reservoir water
levels, flooding, and habitat destruction.
Contour farming is considered an active form of sustainable agriculture.
185. Which of the following microorganisms can bring about nutrient
enrichment of soil?
1) Anabaena azollae
2) Nostoc
3) Azolla pinnata

Select the correct answer using the codes below.

a) 1 only
b) 2 only
c) 1 and 3 only
d) 1, 2 and 3

Correct Answer : D
Answer Justification :

Justification: Bio-fertilizers are specific types of living organisms like


symbiotic bacteria, Cyanobacteria (also called as blue green algae), sea
weeds etc. that can bring about nutrient enrichment of soil in many
different ways. Anabaena azollae, Anabaena cycadae, Azolla pinnata and
Nostoc are different plants that enhance the productivity Of soil when
added to it.
Bacteria like Rhizobium fix nitrogen for plants and Nostoc, Azolla, and
Cyanobacteria that are great nitrogen fixers, are used as bio- fertilizers in
crop fields, most frequently.
Bio-fertilizers are useful in the Conservation and Management Of soil in
following ways —
These are helpful in the replenishment and enhancement of soil fertility.
These fertilizers improve water holding capacity, aeration, porosity and
drainage of soil.
These fertilizers are helpful in reclamation of unproductive soils.
These fertilizers are helpful in preventing soil erosion.

186. Terrace Farming offers which of the following advantage(s)?


1) Increased water run-off
2) Help retain soil moisture
3) Cut down soil erosion

Select the correct answer using the codes below.

a) 1 and 2 only
b) 2 and 3 only
c) 3 only
d) 1 and 3 only

Correct Answer : B
Answer Justification :
Justification: Statement 1: Terrace farming is done on the steep slopes
so that flat surfaces are available to grow crops. They can reduce surface
run-off and soil erosion.
Statement 2: The water break also allows more time for the water to settle
into the soil.
Statement 3: It logically follows that as the velocity of water over the
slopes is reduced, it lowers soil erosion.

187. National Innovations on Climate Resilient Agriculture (NICRA) is a


network project of the Indian Council of Agricultural Research (ICAR)
launched in 2011. The project aims to

1) Demonstrate site specific technology packages on farmers’ fields for


adapting to current climate risks
2) Enhance the capacity building of scientists and other stakeholders in
climate resilient agricultural research and its application

Which of the above is/are correct?

a) 1 only
b) 2 only
c) Both 1 and 2
d) None

Correct Answer : C
Answer Justification :

Background: The impacts of climate change are global, but countries


like India are more vulnerable in view of the high population depending
on agriculture.
In India, significant negative impacts have been implied with medium-
term (2010-2039) climate change, predicted to reduce yields by 4.5 to 9
percent, depending on the magnitude and distribution of warming.
Since agriculture makes up roughly 16 percent of India’s GDP, a 4.5 to 9%
negative impact on production implies a cost of climate change to be
roughly up to 1.5 percent of GDP per year.
Justification: The project consists of four components viz. Strategic
Research, Technology Demonstration, Capacity Building and
Sponsored/Competitive Grants.
The Prime Minister’s National Action Plan on climate change has
identified Agriculture as one of the eight national missions.
With this background, the ICAR has launched a major Project entitled,
National Initiative on Climate Resilient Agriculture (NICRA) during 2010-
11 with an outlay of Rs.350 crores for the XI Plan with the following
objectives.
To enhance the resilience of Indian agriculture covering crops, livestock
and fisheries to climatic variability and climate change through
development and application of improved production and risk
management technologies
To demonstrate site specific technology packages on farmers’ fields for
adapting to current climate risks
To enhance the capacity building of scientists and other stakeholders in
climate resilient agricultural research and its application

188. Biochemical pesticides are

a) Naturally occurring substances that control pests by non-toxic


mechanisms
b) Artificial chemicals that directly kill or inactivate the pest
c) Bio-fertilizers that also act as a pesticide
d) Symbiotic microorganisms residing in plant roots that release certain
chemicals hindering growth of foreign microbes and entities

Correct Answer : A
Answer Justification :

Learning: Conventional pesticides are generally synthetic materials that


directly kill or inactivate the pest.
Biochemical pesticides include substances that interfere with mating, such
as insect sex pheromones, as well as various scented plant extracts that
attract insect pests to traps.
Biopesticides are certain types of pesticides derived from such natural
materials as animals, plants, bacteria, and certain minerals. For example,
canola oil and baking soda have pesticidal applications and are considered
biopesticides.
Microbial pesticides consist of a microorganism (e.g., a bacterium, fungus,
virus or protozoan) as the active ingredient.
Microbial pesticides can control many different kinds of pests, although
each separate active ingredient is relatively specific for its target pest[s].
For example, there are fungi that control certain weeds and other fungi
that kill specific insects.
The most widely used microbial pesticides are subspecies and strains of
Bacillus thuringiensis, or Bt. Each strain of this bacterium produces a
different mix of proteins and specifically kills one or a few related species
of insect larvae.

189. Which of the following are the benefits of Agro-forestry?

1) Increased organic matter on the field and soil


2) More efficient recycling of nutrients by deep rooted trees on the site
3) Reduction of surface run-off and nutrient leaching

Select the correct answer using the codes below.

a) 1, 2 and 3
b) 3 only
c) 2 and 3 only
d) 1 and 3 only
Correct Answer : A
Answer Justification :

Justification: Agroforestry is a land use management system in which


trees or shrubs are grown around or among crops or pastureland.
Environmental benefits
Reduction of pressure on natural forests.
More efficient recycling of nutrients by deep rooted trees on the site as
crop roots do not reach as deep in the soil as that of trees, and often the
top soil is bereft of nutrients.
Better protection of ecological systems
Reduction of surface run-off, nutrient leaching and soil erosion through
impeding effect of tree roots and stems on these processes
Improvement of microclimate, such as lowering of soil surface
temperature and reduction of evaporation of soil moisture through a
combination of mulching and shading
Increment in soil nutrients through addition and decomposition of
litterfall. Improvement of soil structure through the constant addition of
organic matter from decomposed litter. So, 3 is incorrect.

190. Contour ploughing is done in order to

1) Let water freely flow along a slope of hill


2) Create furrows that reduces soil erosion

Which of the above is/are correct?

a) 1 only
b) 2 only
c) Both 1 and 2
d) None

Correct Answer : B

Answer Justification :

Justification: It is the farming practice of plowing and/or planting


across a slope following its elevation contour lines. These contour lines
create a water break which reduces the formation of rills and gullies
during times of heavy water run-off; which is a major cause of soil erosion.
The water break also allows more time for the water to settle into the soil.
In contour plowing, the ruts made by the plow run perpendicular rather
than parallel to slopes, generally resulting in furrows that curve around
the land and are level.
This method is also known for preventing tillage erosion
191. In India, the safety aspects of genetically modified crops are
assessed by which of these bodies?

1) Institutional Biosafety Committees (IBSCs)


2) Review Committee on Genetic Manipulation (RCGM)
3) Genetic Engineering Appraisal Committee (GEAC)
4) National Biopiracy authority (NBA)

Select the correct answer using the codes below.


a) 1, 2 and 3 only
b) 2 and 4 only
c) 1 and 3 only
d) 2, 3 and 4 only
Correct Answer : A

Answer Justification :

Background: GEAC is the apex body constituted in the


Ministry of Environment and Forests.

It was set up under 'Rules for Manufacture, Use, Import, Export


and Storage of Hazardous Microorganisms/Genetically
Engineered Organisms or Cells 1989', under the Environment
Protection Act, 1986.

The Rules of 1989 also define five competent authorities i.e. IBSC,
RCGM, GEAC, State Biotechnology Coordination Committee
(SBCC) and District Level Committee (DLC) for
handling of various aspects of the rules.

Justification: Statement 1: Its main function is to note, examine


and approve proposals involving r- DNA work; to ensure
adherence of r-DNA Safety Guidelines- 1990 of Government and
inspection of containment facilities at R&D and production units.
Statement 2: Main functions of RGCM are:

To bring out manuals of guidelines specifying producers for


regulatory process on GMOs in research, use and applications
including in industry with a view to ensure environmental safety.

To review all on going r-DNA projects involving high risk category


and controlled field experiments

To lay down producers for restriction or prohibition, production,


sale, import & use of GMOs both for research and applications.

To authorize imports of GMOs/ transgenes/ transgenic seeds for


research purposes.

192. Ahar-Pyne system, recently seen in news, is a

a) Traditional floodwater harvesting system in South Bihar


b) Plant extract used in addressing anemia among tribals
c) System of rock mining in earthquake prone regions
d) Free national meal scheme launched by an NGO
Correct Answer : A

Answer Justification :

Justification: Ahar-pyne system is an indigenous irrigation


technology. This system has evolved from an understanding of the
particular agro-climatic conditions of the South Bihar plains of India.
Ahars are reservoirs and consist of a major embankment across the line of
the drainage with two side embankments running backwards up to the
line of the drainage gradually losing their heights because of the gradient
of the surface.

Thus, an ahar resembles a rectangular catchment basin with only three


embankments, and the fourth side left open for the drainage water to enter
the catchment basin following the natural gradient of the country.
Learning: A model of water conservation adopted successfully by the
authorities in Nalanda district of south central Bihar, has been selected
for the national award for excellence in the Mahatma Gandhi national
rural employment guarantee programme (MGNREGP), by the ministry
of rural development.

The award for excellence will be conferred on ‘Project Jal Sanchay’, the
water conservation model.

‘Project jal sanchay’ was launched under MGRNREGP, to offer farmers a


wide spectrum of solutions to water crisis.

193. Mulch is simply a protective layer of a material that is spread on


top of the soil. Consider the following with reference to it.

1) Mulch provides organic matter which helps keep the soil loose.
2) Mulch improves root growth and increase the water-holding
capacity of the soil.

Which of the above is/are correct?


a) 1 only
b) 2 only
c) Both 1 and 2
d) None
Correct Answer : C

Answer Justification :

Justification: Statement 1: Organic mulches also improve the


condition of the soil. As these mulches slowly decompose, they
provide organic matter which helps keep the soil loose.

This improves root growth, increases the infiltration of water, and


also improves the water-holding capacity of the soil.

Organic matter is a source of plant nutrients and provides an ideal


environment for earthworms and other beneficial soil organisms.

Learning: Both organic and inorganic mulches can be used.

While inorganic mulches have their place in certain landscapes,


they lack the soil improving properties of organic mulches.
Inorganic mulches, because of their permanence, may be
difficult to remove if crop plans need to be changed.

194. Organic Farming does NOT deal with which of the following
problems of farms?

1) Nutrient management
2) Weed management
3) Insect pest management
4) Disease management in plants

Select the correct answer using the codes below.


a) 1 and 2 only
b) 2 and 3 only
c) 3 and 4 only
d) It deals with all of the above using non-synthetic methods.
Correct Answer : D
Answer Justification :

Justification: Organic farming is a production system which


avoids or largely excludes the use of synthetically compounded
fertilizers, pesticides, growth regulators, genetically modified
organisms and livestock food additives.
To the maximum extent possible organic farming system rely upon
crop rotations, use of crop residues, animal manures, legumes,
green manures, off farm organic wastes etc.

It also focuses on the use of biofertilizers, mechanical


cultivation, mineral bearing rocks and aspects of biological
control to maintain soil productivity and tilth to supply plant
nutrients and to control insect, weeds and other pests.

CONSERVATION EFFORTS

NATIONAL PARKS AND SANCTUARIES


195. The protected area that represents the north-western limit of the
Bengal tiger’s distribution range

a) Ranthambore Tiger Reserve


b) Panna Tiger Reserve
c) Dibang Wildlife Sanctuary
d) Kanha Tiger Reserve

Correct Answer : A
Answer Justification :

Justification: Dibag is in Arunachal Pradesh, Kanha and Panna are in


MP. Ranthambore is in Rajasthan.
Ranthambore Tiger Reserve lies in the eastern part of Rajasthan state
It comprises of the Ranthambore National Park as well as Sawai Mansingh
and Keladevi Sanctuaries, each with varied conservation history.
Geographically, narrow corridors link the two sanctuaries to the reserve’s
core– the National Park.
The vegetation includes grasslands on plateaus and dense forests along
the seasonal streams.

196. Kanikkarans are the Original tribal Settlers in this protected area. It
includes the Indian Ecoregions of South Western Ghats moist deciduous
forests, South Western Ghats montane rain forests and Shola. It became
part of World Network of Biosphere Reserves in 2016, it is

a) Agasthyamala Biosphere Reserve


b) Nilgiri Biosphere Reserve
c) Periyar Biosphere Reserve
d) Nagarhole Biosphere Reserve

Correct Answer : A
Answer Justification :

Learning: It spans both Kerala and Tamil Nadu regions.


It is the habitat for 2,000 varieties of medicinal plants, of which at least 50
are rare and endangered species.
Rare animals include the tiger, Asian Elephant, and Nilgiri Tahr.
Kanikaran are one of the oldest surviving ancient tribes in the world. Even
though it is said their language is a mixture of Tamil and Malayalam
dialects, they have their own mother tongue, which is centuries old.

197. The highest peak in India south of the Himalayas is inside this park.
Many perennial streams criss- cross the park. A UNESCO World Heritage
Site, it is

a) Silent Valley National Park


b) Singalila National Park
c) Kudremukh National Park
d) Eravikulam National Park

Correct Answer : D
Answer Justification :

Learning: It is located along the Western Ghats in the Idukki district of


Kerala.
It also runs the nearby Mathikettan Shola National Park, Anamudi Shola
National Park, Pambadum Shola National Park, Chinnar Wildlife
Sanctuary and the Kurinjimala Sanctuary.
It consists of a high rolling hill plateau with a base elevation of about
2,000 m. The terrain consists of high altitude grasslands interspersed with
sholas.
Many perennial streams criss-cross the park. They merge to form
tributaries of the Periyar river in the west and of the Cauvery River in the
east.
Twenty six species of mammals have been recorded in the park including
the largest surviving population of Nilgiri tahr

198. The National Park is located where the Deccan Plateau meets the
Western Ghats. The park is flanked by the Kabini river in the north and
the Moyar river in the south. It is?

a) Bandipur National Park


b) Anaimudi National Park
c) Kabini Forest Reserve
d) Eravikulam National Park

Correct Answer : A
Answer Justification :

Learning: Together with the adjoining Nagarhole National Park,


Mudumalai National Park and Wayanad Wildlife Sanctuary, it is part of
the Nilgiri Biosphere Reserve
The altitude of the park ranges from over 2,000 ft to close to 4,770 ft. As a
result, the park has a variety of biomes including dry deciduous forests,
moist deciduous forests and shrublands.
Dry deciduous forest is dominant however.
The wide range of habitats help support a diverse range of organisms. The
park is flanked by the Kabini river in the north and the Moyar river in the
south. The Nugu river runs through the park.
Option C: Kabini Forest Reserve is surrounded by Wayanad Wildlife
Sanctuary, Mudumalai National Park, Bandipur National Park and
Nagarhole National Park.

199. The largest Marine Protected Area in India is

a) Mahatma Gandhi Marine National Park, Andaman


b) Gulf of Kutch Marine National Park
c) Garimatha Marine Sanctuary
d) Chilka Marine Sanctuary

Correct Answer : C
Answer Justification :

Justification: Option A: Most of the coral reefs in the park are fringing
reefs. The type and composition of vegetation varies from island to island.
Most notably there you can see a difference between the tourist islands
which suffer more anthropocentric change and others which do not.
Twin Islands are an important breeding ground for turtles within the park.
Option B: Marine National Park of Gulf of Kutchh is a fragile ecosystem.
In recent years, biodiversity of marine park has been under threat on
several scores like extraction of corals and sands by cement industries,
increased turbidity of water, oil refineries, chemical industries and
mechanised fishing boats
Option C: Located in Odisha and a very popular tourist attraction,
extending from Dhamra River mouth in the north to Brahmani river
mouth in the south, it is the one of world's most important nesting beach
for olive ridley turtles.

200. Being at the confluence of the Western Ghats and the Eastern Ghats, the
sanctuary is home to eco- systems that are unique to both the mountain
ranges. The site is also a declared a tiger reserve. It is

a) Kanger Ghati Wildlife Sanctuary


b) Gugamal Sanctuary
c) Biligiriranganatha Swamy Temple Wildlife Sanctuary
d) Kudremukh Sanctuary

Correct Answer : C
Answer Justification :

Justification: The hills are located at the easternmost edge of the


Western Ghats and support
diverse flora and fauna in view of the various habitat types supported.
The sanctuary derives its name Biligiri (white hill in Kannada) from the
white rock face that constitutes the major hill crowned with the temple of
Lord Rangaswamy or from the white mist and the silver clouds that cover
these hills for a greater part of the year.
The hills are in the Yelandur, Kollegal and Chamarajanagar talukas of
Chamarajanagar District of Karnataka. They are contiguous with hills in
Sathyamangalam Wildlife Sanctuary.
Learning: Nearby, bannerghatta national park is part of a wildlife
corridor for elephants which connects the Biligiriranga Hills and the
Sathyamangalam forest.
Bannerghatta is the first biological park in India to have a fenced forested
elephant sanctuary where elephants can roam around freely without
chains.

201. The Satpura mountain ranges cross India from west to east and this
Biosphere Reserve lies directly in its centre. There is a Plateau, borrowing
name from this biosphere reserve, which is also known as the ‘Queen of
Satpura’. The Biosphere Reserve is

a) Simlipal Biosphere Reserve


b) Nilgiri Biosphere Reserve
c) Achanakmar-Amarkantak Biosphere Reserve
d) Pachmarhi Biosphere Reserve

Correct Answer : D
Answer Justification :

Learning: The conservation area was created in 1999 by the Indian


government. UNESCO designated it a biosphere reserve in 2009.
It includes three wildlife conservation units:
Bori Sanctuary
Pachmarhi Sanctuary
Satpura National Park
The reserve is composed primarily of forest habitats, and is an important
transition zone between the forest species of western and eastern India.
The forests are dominated by teak. They include the westernmost groves
of sal, which is the dominant tree of eastern India's forests.

202. The protected area that is also famously known as ‘Top Slip’ is actually

a) Anamalai Tiger Reserve


b) Bandipur National Park
c) Gangeri National Park
d) Kanheri Tiger Reserve

Correct Answer : A
Answer Justification :
Learning: Earlier known as Indira Gandhi Wildlife Sanctuary and
National Park (IGWLS&NP) and previously as Anamalai Wildlife
Sanctuary, it is a protected area located in the Anaimalai Hills of
Coimbatore District of Tamil Nadu state.
The park is named after former Prime Minister of India Indira Gandhi.
The main tourist facilities are located in the northeast corner of the park at
"Topslip", so named because of the local 19th century practice of sliding
timber logs down the hills from here.
This tiger reserve, together with the several other contiguous protected
forest and grassland habitats, is the core of the Parambikulum-Indira
Gandhi tiger habitat landscape complex.

203. Among the following protected areas, the one that lies farthest in the
South is

a) Silent Valley National Park


b) Mudumalai Wildlife Sanctuary
c) Rajiv Gandhi National Park
d) Bandipur Tiger Reserve

Correct Answer : A

Answer Justification

Silent Valley NP is one of the last undisturbed tracts of South Western


Ghats mountain rain forests and tropical moist evergreen forest in India.
Contiguous with the proposed Karimpuzha National Park to the north and
Mukurthi National Park to the north-east, it is the core of the Nilgiri
International Biosphere Reserve, and is part of The Nilgiri Sub-Cluster,
Western Ghats World Heritage Site, recognised by UNESCO in 2007
204. Part of the Eastern Ghats, these hills were declared as a Biosphere
Reserve in 2010. Tirupati, a major Hindu pilgrimage town, is located in
these hills. It is

a) Anantagiri Hills
b) Seshachalam Hills
c) Agasthyamalai Biosphere Reserve
d) Simlipal Biosphere Reserve

Correct Answer : B
Answer Justification :

Learning: The ranges were formed during the Precambrian era (3.8
billion to 540 million years ago). Minerals contained in these hills include
sandstone and shale interbedded with limestone. The ranges are bounded
by the Rayalaseema uplands to the west and northwest, and the Nandyal
Valley to the north.
The Srivenkateshwara National Park is also located in these ranges. The
famous Natural Arch, Tirumala Hills is also a part of Seshachalam Hills.
In 2010 it was designated as a Biosphere Reserve. It has large reserves of
Red Sandal wood which is used in medicines, soaps ,spiritual rituals.

205. A species known as dugong (sea cow) is found here. The estuaries of
some South Indian rivers drain near the protected area. It is

a) Gulf of Mannar Marine National Park


b) Lakshadweep Sanctuary
c) Mahatma Gandhi Conservation Park
d) Paradip Marine Community Reserve

Correct Answer : A
Answer Justification :

Learning: The Gulf of Mannar is a large shallow bay forming part of the
Laccadive Sea in the Indian Ocean.
The Gulf of Mannar Marine National Park is a protected area of India
consisting of 21 small islands (islets) and adjacent coral reefs in the Gulf of
Mannar in the Indian Ocean.
The park has a high diversity of plants and animals in its marine,
intertidal and near shore habitats. Public access inside the Park is limited
to glass bottom boat rides.
Mangroves dominate the intertidal zones of the park islands.
206. The Kaziranga-Karbi Anglong Landscape (KKL) is a vital site classified
as a critical region by WWF- India. Consider the following about it.

1) It is situated within the Indo-Burma biodiversity hotspot.


2) The area is critical for the protection and conservation of Elephants,
Tigers and Rhinoceros.

Which of the above is/are correct?

a) 1 only
b) 2 only
c) Both 1 and 2
d) None

Correct Answer : C
Answer Justification :

Justification: Statement 1: KKL is spread over 25,000 km2 south of the


Brahmaputra River in Assam, touching the neighbouring states of
Meghalaya and Nagaland in north-eastern India.
Kaziranga National Park, the biggest protected area (PA) in this landscape
is connected with the rest of the landscape through four corridors, namely
Panbari, Haldhibari, Amguri and Kanchanjhuri, which are facing
anthropogenic pressures.
Statement 2: This landscape has a population of about 2500 elephants –
about half of Assam’s elephant population and more than 70 per cent of
Assam’s tigers.
In addition, this landscape boasts of more than 2,000 rhinos, comprising
close to 90 per cent of the rhino population of India. This makes the area
critical for protection and conservation of wildlife and their habitats.

207. Considered a wetland of international importance especially as


waterfowl refuge, it is a significant abode for the birds migrating from
across the international frontiers and one of the largest man-made
wetlands of Northern India?

a) Harike Lake
b) Ashtamudi Wetland
c) Chandra Taal Lake
d) Renuka Lake

Correct Answer : A
Answer Justification :

Justification: Option A: It is one of the largest man-made wetlands of


northern India which shares its area with the Tarntaran, Ferozpur and
Kapurthala districts of Punjab.
It came into existence in 1952 after the construction of barrage near the
confluence of rivers Sutlej and Beas. Harike is a significant abode for the
birds migrating from across the international frontiers.
Harike also harbours endangered aquatic mammalian as well as reptilian
fauna like Indus river dolphin, smooth-coated otter and seven species of
rare freshwater turtles.
Option B: It is a natural backwater in Kollam district. It forms an estuary
with Sea at Neendakara which is a famous fishing harbour in Kerala.
National Waterway 3 passes through it.
Option C: A Ramsar site, it is a high altitude lake on the upper Chandra
valley flowing to the Chandra river of the Western Himalayas near the
Kunzam pass joining the Himalayan and Pir Panjal ranges.
It supports CITES and IUCN Redlisted Snow Leopard and is a refuge for
many species like Snow Cock, Chukor, Black Ring Stilt, Kestrel, Golden
Eagle, Chough, Red Fox, Himalayan Ibex, and Blue Sheep.
Option D: It is a natural wetland with freshwater springs and inland
subterranean karst formations, fed by a small stream flowing from the
lower Himalayan out to the Giri river.
The lake has high religious significance and is named after the mother of
Hindu sage Parshuram, and is thus visited by thousands of pilgrims and
tourists.
It is a Ramsar site.

208. It is one of the largest national parks of India and the Great Indian
bustard is commonly found in the region. Despite a fragile ecosystem
there is an abundance of birdlife here. It is

a) Desert National Park


b) Hemis National Park
c) Neora Valley National Park
d) Khangchendoga National park

Correct Answer : A
Answer Justification :

Justification: The major landform of the Desert National Park consists


of craggy rocks and compact salt lake bottoms, intermedial areas and fixed
dunes.
It has a collection of fossils of animals and plants of 180 million years old.
Some fossils of dinosaurs of 6 million years old have been found in the
area
The region is a haven for migratory and resident birds of the desert. The
endangered Indian bustard is the major attraction of Desert National
Park.
Gadsisar Sagar Tank tank is among the tourist places in Jaisalmer,
Rajasthan. Thousands of migratory birds come to this place every year.
Hemis is the largest national park of India.
209. The protected area, located in Western India, is one of the richest bird
areas in the world and is known for nesting of resident birds and visiting
migratory birds including water birds. A Ramsar Site, it is

a) Betla National Park


b) Dachigam National Park
c) Keoladeo National park
d) Sanjay Gandhi National Park

Correct Answer : C
Answer Justification :

Learning:Keoladeo national park is a very important site, and has been


covered in one another question.

Every year thousands of migratory waterfowl visit the park for wintering
and breeding. It is also a World Heritage Site.

The reserve protects Bharatpur from frequent floods, provides grazing


grounds for village cattle, and earlier was primarily used as a waterfowl
hunting ground.

210. What is common between the sites of Wild Ass


Sanctuary,Kangchendzonga National Park, Namdhapha National Park
and Neora Valley National Park?

a) All of them are located in North-eastern India.


b) All of them host varied vegetation ranging from tropical to tundra.
c) They been included in the tentative list of World Heritage Site
nominations from India.
d) They are situated in either arid or semi-arid regions.

Correct Answer : C
Answer Justification :

Justification:Option A:Indian Wild Ass Sanctuary also known as the


Wild Ass Wildlife Sanctuary islocated in the Little Rann of Kutch in the
Gujarat state of India.

Option D:Namdhapha National Park and Neora Valley National Park


contain dense vegetation.

Option C: Six new natural heritage sites, viz., Bhitarkanika Conservation


area, Desert National Park, Kangchendzonga National Park, Namdhapha
National Park, Neora Valley National Park, and Wild Ass Sanctuary, Little
Rann of Kutch have been included in the tentative list of World Heritage
Site nominations from India.

211. Which of the following national park boasts the highest tiger density in
India?

a) Kaziranga
b) Jim Corbett
c) Orang
d) Bandipur

Correct Answer : C
Answer Justification :

Learning: A Wildlife Institute of India and NTCA report in 2016 titled


The Status of Tigers, Co- predators and Prey in India, said the density in
Kaziranga National Park was 12.72 per 100 sq. km., followed by Jim
Corbett National Park (11) in Uttarakhand and Bandipur National Park
(10.28) in Karnataka.
However, a recent report says that based on density of tigers for 100 sq.
km., Orang tops the list as
35.44 tigers per 100 sq. km. It has the highest density nationally.
Orang is also known as the mini Kaziranga National Park (IUCN site)
since the two parks have a similar landscape made up of marshes, streams
and grasslands.

212. Consider the following statements.

1) There is no settlement in this national park.


2) It is encompassed in the Nanda Devi Biosphere Reserve and registered
under the UNESCO World Network of Biosphere Reserves.
3) It is known for its meadows of endemic alpine flowers.

The above refer to?

a) Dudhwa National Park


b) Indravati National Park
c) Gangotri National Park
d) Valley of Flowers National Park

Correct Answer : D
Answer Justification :

Justification: Dudhwa is in U.P, whereas Joshimath is in Uttarakhand.


So, (a) is eliminated.
While Gangotri park is in Uttarakhand, Harsil is the nearest town and it is
not in the UNESCO network. So, (c) is also eliminated.
Indravati is in MP, so (d) is also wrong.
Learning: Located in West Himalayas, it is home to rare and endangered
animals, including the Asiatic black bear, snow leopard, musk deer, brown
bear, red fox and blue sheep.
Flowers like orchids, poppies, primulas, marigold, daisies and anemones
carpet the ground at the national park.

213. The Biosphere Reserve that was established the most recently is

a) Panna Biosphere Reserve


b) Panchmari Biosphere Reserve
c) Achanakmar-Amarkantak Biosphere Reserve
d) Simlipal Biosphere Reserve

Correct Answer : A
Answer Justification :

Learning: It is the 3rd Biosphere Reserve of Madhya Pradesh. The area


represents unique ecosystem within narrow belt of table top mountains of
‘Vindhyan Hill Ranges’ and part of ‘Bundelkhand’ region. This includes
the traditional agro-ecosystems, dry deciduous forests of Teak, Salai,
Kardhai, bamboo and mixed type of forests.
It falls under bio-geographic zones of Deccan Peninsula and covering the
Biotic Province of Central highlands. However, it is nearer to confluence of
Deccan Peninsula (Central Highlands), Upper Gangetic Plain and Semi-
Arid Gujarat Rajputana .
Therefore the area becomes unique, reflecting the influence of three bio-
geographic regions. From agro-climatic zone point of view, it falls under
Bundelkhand and Kymore plateau of Satpura hill ranges.

214. The Great Himalayan National Park (GHNP), a UNESCO World Heritage
Site, is situated in which of these famous and pristine river valleys?

a) Baspa
b) Tirthan
c) Chumbi
d) Doon

Correct Answer : B
Answer Justification :

Learning: This is a lateral explanation.


The park was established in 1984 and is spread over an area of 1,171 km2
at an altitude of between 1500 and 6000 m.
The GHNP is at the junction of world's two major faunal regions: the
oriental to the south and palaearctic to the north. The temperate forest
flora-fauna of GHNP represents the westernmost extension of the Sino-
Japanese Region.
The high altitude ecosystem of the Northwest Himalaya has common
plant elements with the adjacent Western and Central Asiatic region. As a
result of its 4,100 m elevation range the park has a diversity of zones with
their representative flora and fauna, such as alpine, glacial, temperate, and
sub tropical forests.
These biogeographic elements are result of geological evolution of
Himalaya which continues today from the action of plate tectonics and
continental drift.
They fauna of this park are protected under the strict guidelines of the
Wildlife Protection Act of 1972; hence any sort of hunting is not permitted.

215. Which of the following is a man-made and man-managed wetland


hosting a rich bird population that is also one of the national parks of
India and a UNESCO World Heritage Site?

a) Keibul Lamjao
b) Bhitarkanika
c) Sultanpur
d) Keoladeo Ghana

Correct Answer : D
Answer Justification

Justification: Option (a) is a floating park as an integral part of Loktak


Lake, Manipur.

Option (b) is not a man made wetland. It is the second largest mangrove
ecosystem in India and is inundated by a number of rivers - Brahmani,
Baitarni, Dhamra, Pathsala and others.

Option (c): Sultanpur is a bird sanctuary, and does not meet the
description given in the question.

Option (d): It is a famous avifauna sanctuary that hosts thousands of


birds, especially during the winter season.

The reserve protects Bharatpur from frequent floods, provides grazing


grounds for village cattle, and earlier was primarily used as a waterfowl
hunting ground.

It was also declared as a Ramsar site under the Wetland Convention


216. Consider the following matches.

The largest Marine Protected Area (MPA) in

1) India: Gulf of Mannar


2) World: Northern Ocean

Which of the above is/are correct matches?

a) 1 only
b) 2 only
c) Both 1 and 2
d) None

Correct Answer : D
Answer Justification :

Justification: Statement 1: It is in Odisha.

Statement 2: Marae Moana in Cook Islands: 1,976,000 sq. km. and Ross
Sea Region Marine Protected Area in Antarctica, 1,555,851 sq. km, both
designated in 2017 are the two largest MPAs in the world.

The Ross Sea, its shelf and slope only comprise 2% of the Southern Ocean
but they are home to 38% of the world's Adelie penguins, 30% of the
world's Antarctic petrels and around 6% of the world's population of
Antarctic minke whales
217. Which of the following national parks have Tropical Vegetation?

1) Khangchendzonga
2) Nandadevi
3) Simlipal

Select the correct answer using the codes below.

a) 1 and 2 only
b) 2 and 3 only
c) 3 only
d) 1 only

Correct Answer : C
Answer Justification :

Justification: Statement 1: Both Khangchendzonga and Nandadevi


National park do not have tropical vegetation because of the high altitude
they are situated in. The vegetation there ranges from temperate to boreal
forests.
In contrast, Namdapha National Park hosts diverse vegetation with
increasing altitude from tropical moist forests to Montane forests,
temperate forests and at the higher elevations, to Alpine meadows and
perennial snow.
Simlipal National Park largely has tropical deciduous vegetation.

218. Sharing the north-eastern boundary with Nepal, the area is a vast alluvial
floodplain traversed by numerous rivers and streams. With high density of
forests interspersed with grasslands, a characteristic of the Terai
ecosystems in India, it is, probably, the last prominent remnant of this
type of ecosystem. The protected area is

a) Bandhavgarh National Park


b) Kaziranga National Park
c) Dudhwa National Park
d) Jim Corbett National Park

Correct Answer : C
Answer Justification :

Learning: The Dudhwa Tiger Reserve is a protected area in Uttar


Pradesh that stretches mainly across the Lakhimpur Kheri and Bahraich
districts and comprises the Dudhwa National Park, Kishanpur Wildlife
Sanctuary and Katarniaghat Wildlife Sanctuary.
It covers an area of around 500 sq mi) and includes three large forest
fragments amidst the matrix dominated by agriculture. It shares the
north-eastern boundary with Nepal, which is defined to a large extent by
the Mohana River.
The area is a vast alluvial floodplain traversed by numerous rivers and
streams flowing in south- easterly direction.
Spread across with marshes, grasslands and dense forests, the area is
actually meant for tremendous counts of Swamp Deer and Tigers species.
The area of the Park is composed of a vast alluvial plain along the
tributaries of Mohana and Suheli, interspersed with numerous rivulets,
lakes and pools.
The park has some of the best forests of 'Sal' tree in the world.

219. This Tiger Reserve serves as international corridor for Asian elephant
migration between India and Bhutan with the northern boundary of this
Reserve running along the India-Bhutan international border. It is?

a) Dampa
b) Valmiki
c) Dudhwa
d) Buxa

Correct Answer : D
Answer Justification :

Justification: Option B and C: Dudhwa and Valmiki are close to India-


Nepal international borders. Dampa reserve is in Mizoram.
Buxa Tiger reserve: Notified as a tiger reserve in 1983, Buxa consists of
moist, deciduous and evergreen forests.
Northern boundary of Buxa Tiger Reserve runs along the international
border with Bhutan.
The Sinchula hill range lies all along the northern side of BTR and the
eastern boundary touches that of the Assam state.
The Phipsu Wildlife Sanctuary of Bhutan is contiguous to the north of
BTR. Manas National Park lies on east of BTR.
BTR, thus, serves as international corridor for Asian elephant migration
between India and Bhutan.

Learning: Buxa Tiger Reserve in West Bengal has been identified for the
tiger augmentation programme by the National Tiger Conservation
Authority (NTCA).

Tigers from the forest reserves of Assam, which have a similar flora and
fauna, will be introduced in Buxa.
220. You are likely to find both Mangroves and Salt Water Crocodiles in

a) Dachigam National Park


b) Dibru-Saikhowa National Park
c) Harike Wetland
d) Bhitarkanika National Park

Correct Answer : D
Answer Justification :

Learning: Bhitarkanika National Park is a national park located in


Kendrapara district of Odisha ineastern India.
Gahirmatha Beach and Marine Sanctuary lies to the east, and separates
swamp region cover with canopy of mangroves from the Bay of Bengal.
The national park is home to Saltwater crocodile (Crocodylus porosus),
Indian python, King cobra, black ibis, darters and many other species of
flora and fauna. It hosts a large number of mangrove species, and is the
second largest mangrove ecosystem in India.
The national park and wildlife sanctuary is inundated by the rivers
Brahmani, Baitarani, Dhamra, Pathsala.
Bhitarkanika has one of the largest populations of endangered saltwater
crocodile in India and is globally unique in that, 10% of the adults exceed
6 m length.

221. Which of the following protected areas is NOT located close to an


International Border?

a) Khangchendzonga National Park


b) Ranthambore National Park
c) Dudhwa National Park
d) Valmiki Tiger Reserve
Correct Answer : B
Answer Justification :

Justification: The first map shows the tiger reserves, and the
second one - National Parks

222. The first Turtle Sanctuary in India will be set up near

a) Vijayawada
b) Thiruananthpuram
c) Allahabad
d) Sundarbans

Correct Answer : C
Answer Justification :

Learning: In order to protect the rich aquatic biodiversity of river Ganga


from escalating anthropogenic pressures, development of a Turtle
sanctuary in Allahabad along with a River Biodiversity Park at Sangam
have been approved under Namami Gange programme.
The project includes:
Development of River Biodiversity Park at Sangam (confluence of Ganga,
Yamuna and mythical Sarasvati),
Establishment of a Turtle Rearing Centre and
Awareness about the importance of river Ganga and imperativeness of its
conservation.
The project will help generate awareness for reducing the impact of
human activities on critical natural resources.
The task of dissipating knowledge about river Ganga will be taken up
which is 100% centrally funded.
Rivers Ganga and Yamuna at Allahabad are home to some of the most
endangered fauna like turtles, the National Aquatic Animal – Gangetic
dolphin, the Gharial and numerous migratory and resident birds.

223. Sanctuaries for protection of Turtles have been established at which of


the following locations in India?

1) Coondapur
2) Sultanpur
3) Gahirmatha

Select the correct answer using the codes below.

a) 1 and 2 only
b) 3 only
c) 2 and 3 only
d) 1, 2 and 3

Correct Answer : B
Answer Justification :

Background: In order to protect the rich aquatic biodiversity of river


Ganga from escalating anthropogenic pressures, development of a Turtle
sanctuary in Allahabad along with a River Biodiversity Park at Sangam
have been approved under Namami Gange programme.
Justification: There are other sanctuaries as well.
The National Chambal River Sanctuary support two of the species, B.
dhongoka and B. kachuga, and riverine hatcheries protect thousands of
eggs annually from predation and return hatchlings to the river.
Karnala Bird Sanctuary in Maharashtra; Nawabganj Bird Sanctuary, a
wetland of Northern India; and Gahirmatha are others.
Turtle Survival Alliance–India is working in these sanctuaries.
ACTS AND POLICIES

224. The Wetlands (Conservation and Management) Rules, 2017

1) Stipulate setting up of State Wetlands Authority (SWA) in each State/UTs


headed by an eminent ecologist from a state NGO
2) Enable conversion of wetland for non-wetland uses to seek a balance
between growth and sustainability
3) Make it mandatory for state authorities to prepare a digital list of all
wetlands.

Select the correct answer using the codes below.

a) 1 only
b) 2 and 3 only
c) 3 only
d) 1, 2 and 3

Correct Answer : C
Answer Justification :

Justification: The rules stipulates for setting up of NWC, headed by


MoEFCC Secretary, to monitor implementation of these rules and oversee
work carried out by States.

Statement 1: The rules also stipulate setting up of SWA in each State/UTs


headed by State’s environment minister and include range of government
officials.

State government will also nominate one expert each in fields of wetland
ecology, hydrology, fisheries, landscape planning and socioeconomics.

SWA will develop comprehensive list of activities to be regulated and


permitted within

notified wetlands and their zone of influence.

It will also recommend additional prohibited activities for specific


wetlands define strategies wise use of wetlands and its conservation and
undertake measures to enhance awareness within stakeholders and local
communities on values and functions of wetlands.

In this case, wise use has been defined as principle of sustainable uses that
is compatible with conservation.

Statement 2: The rules prohibit activities like conversion of wetland for


non-wetland uses including encroachment of any kind, setting up and
expansion of industries, waste dumping and discharge of untreated wastes
and effluents from industries, cities, towns, villages and other human
settlement

225. With reference to Coastal Economic Zones (CEZs), consider the


following statements.

1) CEZs are spatial economic regions comprising of a group of coastal


districts or districts with a strong linkage to the ports in that region.
2) Coastal Economic Units (CEUs) will be specific industrial estate projects
will house the industrial clusters / projects proposed within the CEZ.

Which of the above is/are correct?

a) 1 only
b) 2 only
c) Both 1 and 2
d) None

Correct Answer : C
Answer Justification :

Justification: CEZs are envisaged to tap synergies with the planned


industrial corridor projects.
CEUs will be specific industrial estate projects with a demarcated
boundary similar to the DMIC nodes.
Each CEZ will consist of multiple CEUs and more than one industrial
cluster can be housed within a CEU.
Within each industrial cluster there can be several manufacturing units.
To accelerate the CEU development process, it is proposed that CEUs be
prioritized in locations where land parcels are available in areas close to a
deep draught port and with strong potential for manufacturing.
As part of the National Perspective Plan, 14 CEZs have been proposed
across all Maritime States & Union Territories and their perspective plans
have been prepared. The Master Plans for these CEZs will be developed in
a phased manner.

226. What is/are the difference(s) between a Reserved Forest and a protected
forest?

1) The former is notified under the provisions of Indian Forest Act, whereas
the latter is notified under the Wildlife Protection Act.
2) In the former, all activities are prohibited unless permitted, whereas in the
latter all activities are permitted unless prohibited.

Which of the above is/are correct?


a) 1 only
b) 2 only
c) Both 1 and 2
d) None
Correct Answer : B
Answer Justification :

Justification: Both are notified under the provisions of Indian Forest


Act or the State Forest Acts. But, reserved forests have full degree of
protection, protected forests have limited degree of protection.
At present, reserved forests and protected forests differ in one important
way: Rights to all activities like hunting, grazing, etc. in reserved forests
are banned unless specific orders are issued otherwise.
In protected areas, rights to activities like hunting and grazing are
sometimes given to communities living on the fringes of the forest, who
sustain their livelihood partially or wholly from forest resources or
products.
The first Reserve Forest of India was Satpura National Park.
Unlike national parks of India or wildlife sanctuaries of India, reserved
forests and protected forests are declared by the respective state
governments.

227. According to the Wildlife (Protection) Act, 1972, which of the following
animals cannot be hunted by any person except under some provisions
provided by law?

1) Gangetic Dolphin
2) India Gazelle
3) Lion-tailed Macaque
4) Pygmy Hog

Select the correct answer using the code given below:

a) 1 only
b) 2 and 3 only
c) 1, 3 and 4 only
d) 1, 2, 3 and 4 only

Correct Answer : D
Answer Justification :

Justification: All of these are protected scheduled animals under the


Wildlife Act under Schedule I. Strictest protection is provided under the
Schedule I with protection levels declining in successive schedules.
The whole list can be seen here. We will keep covering these species in
detail wherever needed.
228. Under the scheme, ‘Integrated Development of Wildlife Habitats’,
Central government provides financial and technical assistance to the
State/UT Governments for which of the following activities?

1) Recovery programmes for saving critically endangered species and


habitats
2) Support to Protected Areas
3) Protection of Wildlife Outside Protected Areas

Select the correct answer using the codes below.

a) 1 Only
b) 1 and 3 only
c) 2 and 3 only
d) 1, 2 and 3

Correct Answer : D
Answer Justification :

Justification: The Government of India provides financial and technical


assistance to the State/UT Governments for activities aimed at wildlife
conservation through the Centrally Sponsored Scheme

viz.. The scheme has three components that are covered in S1 to S3.

Support to Protected Areas: Areas falling in mountain regions, coastal


zones, deserts, or those areas which support certain selected endangered
species, are eligible for 100% central assistance for both recurring and
non-recurring items.

Protection of Wildlife Outside Protected Areas: High value biodiversity


areas outside PAs. Areas contiguous to PAs/corridors are given priority.

The Chief Wildlife Wardens prepare a Biodiversity Conservation Plan for


such selected area; Human-wildlife conflict management in and around
forests.

Recovery programme for critically endangered species and habitats:


Initially 17 species have been identified under this component. These are
Snow Leopard, Bustard (including Floricans), Dolphin, Hangul,
NilgiriTahr, Marine Turtles, Dugongs, Edible Nest Swiftlet, Asian Wild
Buffalo, Nicobar Megapode, Manipur Brow-antlered Deer, Vultures,
Malabar Civet, Indian Rhinoceros, Asiatic Lion, Swamp Deer and Jerdon’s
Courser.
229. The Atal Bhujal Yojana (ABY) will focus on

1) Recharging ground water


2) Creating sufficient water storage for agricultural purposes
3) Revival of surface water bodies

Select the correct answer using the codes below.

a) 1 only
b) 2 and 3 only
c) 1 and 3 only
d) 1, 2 and 3

Correct Answer : D
Answer Justification :

Justification: It is a Rs 6000 crore Central scheme that aims at


sustainable ground water management with emphasis on demand side
interventions with community participation in 78 districts covering states
of Gujarat, Maharashtra, Haryana, Karnataka, Rajasthan, Uttar Pradesh
and Madhya Pradesh.It is awaiting cabinet’s clearance.

The objective of scheme is to recharge ground water and create sufficient


water storage for agricultural purposes.

It also focuses on revival of surface water bodies so that ground water level
can be increased, especially in the rural areas. It will give emphasis to
recharging ground water sources and ensure efficient use of water by
involving people at local level.

230. DIVERTISAS, sometimes seen in the context of environmental


discourses, is

a) The International Programme of Biodiversity Science


b) An index to track down the net biodiversity in an ecosystem
c) A Ecological Impact Assessment (EIA) tool that helps policymakers
assess changes in an ecosystem caused by anthropogenic activities
d) A web network of Global Biosphere Reserves hosting the most
important ecological hot spots

Correct Answer : A
Answer Justification :

Learning: DIVERSITAS provides an international framework for


scientists around the world to address the questions posed by biodiversity
loss.
DIVERSITAS seeks to
Develop common international frameworks for collaborative research;
Form research networks to tackle focused scientific questions;
Promote standardised methodologies;
Guide and facilitate construction of global databases;
Facilitate efficient patterns of resource allocation, and undertake analysis,
synthesis and integration activities on particular biodiversity themes;
Promote practical application of cutting-edge science to support policy
and contributing to the Convention on Biological Diversity

231. Consider the following statements.

1) National parks and biosphere reserves are presently constituted under


Environment Protection Act, 1986.
2) Forest Conservation Act, 1980, restricts and regulates the de-reservation
of forests or use of forest land for non-forest purposes

Which of the above is/are correct?

a) 1 only
b) 2 only
c) Both 1 and 2
d) None

Correct Answer : B
Answer Justification :

Justification: Statement 1: National parks and biosphere reserves are


constituted under Wildlife
Conservation Act.
Statement 2: Forest Conservation Act 1980 was enacted to help conserve
the country's forests.
State governments cannot de-reserve forests without the prior approval of
Central Government.
To this end the Act lays down the pre-requisites for the diversion of forest
land for non-forest purposes.
The Indian Forest Act, 1927 consolidates the law relating to forests, the
transit of forest- produce and the duty leviable on timber and other forest-
produce.

232. Conservation and management of wetlands in India is primarily vested


with the

a) Central government
b) State/UT Government
c) Local Government
d) Ramsar Convention – India Chapter

Correct Answer : B
Answer Justification :
Learning: Conservation and management of wetlands is primarily vested
with the State/UTs, who are in physical possession of the area. After
identification of wetlands under the Scheme, the State/UTs are to submit
long-term comprehensive Management Action Plans (MAPs) for a period
of 3-5 years, preferably 5 years, coinciding with the Plan period.
The State Governments are advised to define objectives taking into
consideration factors responsible for degradation of the wetland.
State governments submit MAPs under the National Wetland
Conservation Programme (NWCP) and then receive funding from the
Centre.
Under the Scheme, Ministry also sponsor multidisciplinary research
projects by academic/ managerial/ research institutions on various
aspects of wetland conservation to supplement execution of MAP in more
realistic manner.

233. ECOLEX is a/an

a) Information service on environmental law


b) Global ecological patents system
c) Newly developed Phytosanitary standard of WTO
d) International NGO that protects the rights of communities who are
ecologically vulnerable

Correct Answer : A
Answer Justification :

Learning: It is a web-based platform that is operated jointly by FAO,


IUCN and UNEP.
The ECOLEX database includes information on treaties, international
soft-law and other non- binding policy and technical guidance documents,
national legislation, judicial decisions, and law and policy literature.
Its purpose is to build capacity worldwide by providing the most
comprehensive possible global
source of information on environmental law
Users have direct access to the abstracts and indexing information about
each document, as well as to the full text of most of the information
provided

234. According to Coastal Regulation Zone (CRZ) notifications, which of the


following activities cannot be allowed in any of the CRZ?

1) Letting off solid waste


2) Agriculture and public utilities
3) Exploration of hydrocarbon resources

Select the correct answer using the codes below.


a) 3 only
b) 2 and 3 only
c) 1 and 2 only
d) All of them can be allowed.

Correct Answer : D
Answer Justification :

Background: Under the Environmental Protection Act 1986, notification


was issued in 1991 for regulation of activities in the coastal area by
Ministry of Environment and Forests.
These notification known as Coastal Regulation Zone Notification defined
the Coastal Regulation Zone or CRZ as coastal land up to 500m from the
High Tide Line and a range of 100m along banks of creeks, estuaries,
backwaters and rivers subject to tidal fluctuations is CRZ.
Justification: According to Coastal Regulation Zone notifications, it is
divided into 4 zones:
CRZ I – It refers to the ecologically sensitive areas, essential in
maintaining ecosystem of the coast. These lie between the HTL and LTL.
Only exploration of natural gas and extraction of salt is permitted
CRZ II – These areas form up to the shoreline of the coast. Authorized
structures are not allowed to be constructed in this zone
CRZ III – This includes rural and urban localities. Only certain activities
relating to agriculture and public utilities allowed here
CRZ IV – This includes the aquatic area up to the territorial limit (12
nautical miles). Fishing and allied activities permitted in this zone. Solid
waste can be let off in this zone.
The CRZ notification, 1991 saw a series of reviews and amendments and
was eventually replaced with a new notification in 2011.
The draft Coastal Regulation Zone (CRZ), 2018, which was released by the
Ministry of Environment and Forests (MoEF), has the potential to change
the way coastal stretches in India are governed.
India’s coastline runs over 7,500 kilometres.

235. Consider the following statements about sub-missions under National


Mission on Enhanced Energy Efficiency (NMEEE).

1) Market Transformation for Energy Efficiency (MTEE) scheme primarily


proposes accelerating the shift from local to global energy appliances.
2) Framework for Energy Efficient Economic Development (FEEED)
provides for development of fiscal instruments to promote energy
efficiency.

Which of the above is/are correct?

a) 1 only
b) 2 only
c) Both 1 and 2
d) None
Correct Answer : B
Answer Justification :

Justification: There are these three sub-schemes listed on the home page
of BEE Net.

Perform, Achieve and Trade (PAT): This is a regulatory instrument to


reduce specific energy consumption in energy intensive industries, with an
associated market based mechanism to enhance the cost effectiveness
through certification of excess energy saving which can be traded

MTEE: This scheme proposes accelerating the shift to energy efficient


appliances in designated sectors through innovative measures to make the
products more affordable.

Energy Efficiency Financing Platform (EEFP): It is for creation


of mechanisms which would help finance, demand side management
programmes in all sectors by capturing future energy savings and for
capacity building of financial institutions and ESCOs/Industries.

FEEED is for development of fiscal instruments to promote energy


efficiency.

Learning: The other component under MTEE is a new programme called


Super-Efficient Equipment Programme (SEEP).

SEEP is a program designed to bring accelerated market transformation


for super efficient appliances by providing financial stimulus innovatively
at critical point/s of intervention.

Under this program, ceiling fan has been identifies as the first appliance to
adopted. SEEP for ceiling fans aims to leapfrog to an efficiency level which
will be about 50% more efficient than market average by providing a time
bound incentive to fan manufacturers to manufacture super efficient (SE)
fans and sell the same at a discounted price

The goal is to support the introduction and deployment of super efficient


35W ceiling fans, as against the current average ceiling fan sold in Indian
market with about 70W rating.
236. The Animal Welfare Board of India is a statutory advisory body
on Animal Welfare Laws and promotes animal welfare in the
country. It was established in 1962 under

a) Prevention of Cruelty to Animals Act


b) Wildlife Protection Act
c) Environment Protection Act
d) Forest Act
Correct Answer : A

Answer Justification :

Learning: The Animal Welfare Board of India was started under


the stewardship of Late Smt. Rukmini Devi Arundale, well known
humanitarian.
From ensuring that animal welfare laws in the country are
diligently followed, to provide grants to Animal Welfare
Organizations and advising the Government of India on animal
welfare issues, the Board has been the face of the animal welfare
movement in the country for the last 50 years.

The Board consists of 28 Members. The term of office of Members


is for a period of 3 years.

237. With reference to the governance of Indian forests, consider the


following:

1) Indian Forest Act, 1927 empowers the government to declare any area to
be a reserved forest or a protected forest.
2) Wild Life (Protection) Act, 1972 allows any area to be constituted as a
"protected area".

Which of the above is/are correct?


a) 1 only
b) 2 only
c) Both 1 and 2
d) None
Correct Answer : C

Answer Justification :

Justification: India's forests are governed by two main laws, the Indian
Forest Act, 1927 and the Wild Life (Protection) Act, 1972.
The former allows declaration of an area to be a reserved forest, protected
forest or village forest.

The latter allows any area to be constituted as a national park, wildlife


sanctuary, tiger reserve or community conservation area.

However, this has often caused problems for the tribals, to resolve which a
Forest Rights Act was constituted.

INSTITUTION AND MEASURES

238. Facility for Low Carbon Technology Deployment (FLCTD) aims to


promote innovation of low-carbon technologies and its deployment in
which of these sectors of the Indian Economy?

a) Agricultural sector
b) Hospitality and Tourism
c) Industrial and other related sectors
d) Import-Export sector

Correct Answer : C
Answer Justification :

Learning: It is jointly implemented by the Bureau of Energy Efficiency


(BEE) and the United Nations Industrial Development Organization
(UNIDO). FLCTD is supported by the Global Environment Facility (GEF).
A key agenda of the FLCTD project is to facilitate adoption of improved
low-carbon technologies in the Indian economy via deployment and
scaling up of innovative energy efficient technologies to address
technology gaps and help to reduce specific energy consumption.
This 5-year project aims to promote innovation of low-carbon
technologies and its deployment in industrial and other related sectors of
Indian economy.
The project will conduct annual 'Innovation Challenge' competitions that
will identify innovative low carbon technologies and solutions to improve
efficient end-use of energy, which in turn will help to reduce greenhouse
gas emissions in the long run. The project will promote the deployment of
winning technologies.
239. With reference to the National Clean Energy Fund (NCEF), consider the
following statements.

1) The Fund is designed as a non lapsable fund under Public Account of


India.
2) The Cabinet Committee on Economic Affairs approves all the projects
under the fund.
3) Any project or scheme relating to Innovative methods to adopt to Clean
Energy technology and R&D are eligible for funding under the NCEF.

Select the correct answer using the codes below.

a) 1 only
b) 2 only
c) 1 and 3 only
d) 2 and 3 only

Correct Answer : C
Answer Justification :

Justification: Statement 1 and 2: It is a fund created in 2010-11 using


the carbon tax - clean energy cess - for funding research and innovative
projects in clean energy technologies of public sector or private sector
entities, upto the extent of 40% of the total project cost. Assistance is
available as a loan or as a viability gap funding, as deemed fit by the Inter-
Ministerial group, which decides on the merits of such projects.
The Fund is designed as a non lapsable fund under Public Accounts and
with its secretariat in Plan Finance II Division, Department of
Expenditure, Ministry of Finance.
Statement 3: An Inter-Ministerial Group, chaired by the Finance Secretary
in Ministry of Finance (and comprising of Secretaries of other
Departments) recommends projects eligible for funding under NCEF.
Upon recommendation by NCEF, the final approval is given by the
Minister of the concerned nodal Ministry (which initially approved and
decided to take the project submitted by the public or private entity to
NCEF) if the project cost is below Rs. 150 Crore; by Minister of Finance
and the Minister of the concerned nodal Ministry if the project cost is
between Rs. 150 Crore and 300 crore; and by the Cabinet Committee on
Economic Affairs if the project cost is above Rs. 300 Crore.

240. The Animal Welfare Board of India is a statutory advisory body with an
important function of providing financial assistance to the recognised
Animal Welfare Organisations (AWOs). It was established under

a) Wildlife Protection Act, 1972


b) Prevention of Cruelty to Animals Act, 1960
c) Environment Protection Act, 1972
d) Indian Forest Act, 1927
Correct Answer : B
Answer Justification :

Learning: It was started under the stewardship of Late Smt. Rukmini


Devi Arundale, well known humanitarian.

The Board provides financial assistance to AWOs under various schemes


viz. Regular Grant, Cattle Rescue Grant, Provision of Shelter House for
looking after the Animals, Animal Birth Control (ABC) Programme,
Provision of Ambulance for the animals in distress and Natural Calamity
grant.

Some other important functions are:

To keep the law in force in India for the Prevention of Cruelty to Animals
under constant study and to advise the government on the amendments to
be undertaken in any such law from time to time.

To take all such steps as the Board may think fit to ensure that unwanted
animals are destroyed by local authorities, whenever it is necessary to do
so, either instantaneously or after being rendered insensible to pain or
suffering.

to encourage the formation of Animal Welfare Organisations in any local


area which shall work under the general supervision and guidance of the
Board.

241. Consider the following statements.

1) To undertake, aid, promote and coordinate forestry education, research


and their applications.
2) To develop and maintain a national library and information centre for
forestry and allied sciences.
3) To act as a clearing-house for research and general information related to
forests and wildlife.

The above are the objectives of the?

a) ENVIS Portal of the MoEFCC


b) Indian Council of Forestry Research and Education (ICFRE)
c) Wildlife Institute of India (WII)
d) Forest Survey of India (FSI)

Correct Answer : B
Answer Justification :

Learning: ICFRE is an apex body in the national forestry research


system, has been undertaking the holistic development of forestry
research through need based planning, promoting, conducting and
coordinating research, education and extension covering all aspects of
forestry.

The Council deals with the solution based forestry research in tune with
the emerging issues in the sector, including global concerns such as
climate change, conservation of biological diversity, combating
desertification and sustainable management and development of
resources.

Topical research by the Council enhances public confidence in the ability


of forest managers and researchers to successfully handle challenges
related to natural resource management.

Other Objectives of ICFRE

To develop forestry extension programmes and propagate the same


through mass media, audio-visual aids and extension machinery.

To provide consultancy services in the field of forestry research, education


and allied sciences.

Learning: Forest Survey of India (FSI), founded in 1976 and


headquartered at Dehradun in Uttarakhand, is a Government of India
Ministry of Environment, Forest and Climate Change organisation for
conducting forest surveys, studies and research to periodically monitor the
changing situation of land and forest resources and present the data for
national planning, conservation and sustainable management of
environmental protection as well for the implementation of social forestry
projects

242. ISO 14000 is a family of standards related to

a) Environmental management
b) Public concerts and festivals
c) Electrical appliances
d) Musical instruments

Correct Answer : A
Answer Justification :

Learning: ISO 14000 is the general term used for ISO 14000 family of
standards. ISO 14001:2004
`Environmental Management Systems – Specifications with Guidance for
use’ is a standardpublished by International Organization for
Standardization (ISO)

These standards exist to help organizations (a) minimize how their


operations (processes, etc.) negatively affect the environment (i.e., cause
adverse changes to air, water, or land); (b) comply with applicable laws,
regulations, and other environmentally oriented requirements; and (c)
continually improve in the above.

ISO 14000 is similar to ISO 9000 quality management in that both


pertain to the process of how a product is produced, rather than to the
product itself. As with ISO 9001, certification is performed by third-party
organizations rather than being awarded by ISO directly.

243. The Long Term Ecological Observatories (LTEO) programme of the


MoEFCC

1) Aims to understand the biophysical and anthropogenic drivers of


ecosystem change in the selected biomes
2) Focuses on establishing international observatories linked through a
common Eco-center to assess the impact of climate change on global
biodiversity

Which of the above is/are correct?

a) 1 only
b) 2 only
c) Both 1 and 2
d) None

Correct Answer : A
Answer Justification :

Justification: First phase of the LTEO Programme includes creating a


network of field sites to assess the health of eight different biomes of the
country namely; Western Himalaya, Eastern Himalaya, North-Western
Arid Zone, Central Indian Forests, Western Ghats, Andaman & Nicobar

Islands, Jammu & Kashmir and Sundarbans.

LTEO Programme aims to understand the biophysical and anthropogenic


drivers of ecosystem change in the selected biomes and their effects on
social- ecological responses through a network of scientific institutions.
Activities include experimental work to assess the change of structure and
function in the natural ecosystems, identification of patterns and drivers
of change in the natural ecosystems by monitoring populations of fresh
water fish, birds, mammals, herbivores & carnivores, animal movements,
soil processes in forests & grasslands, biophysical climatic variables, etc

244. The main roles and functions of the National Afforestation and Eco-
Development Board (NAEB) include

1) Financing and overseeing the compensatory afforestation work


2) Rehabilitating and resettling communities displaced by deforestation for
developmental works
3) Sponsor research and extension of research findings to disseminate new
technologies for the regeneration and development of degraded forest

Select the correct answer using the codes below.

a) 1 and 2 only
b) 3 only
c) 1 and 3 only
d) 2 and 3 only

Correct Answer : B
Answer Justification :

Justification: Set up in 1992, the detailed role and functions of the


NAEB are to:
Evolve mechanisms for ecological restoration of degraded forest areas and
adjoining lands through systematic planning and implementation, in a
cost effective manner;
Restore through natural regeneration or appropriate intervention the
forest cover in the country for ecological security and to meet the
fuelwood, fodder and other needs of the rural communities;
Restore fuelwood, fodder, timber and other forest produce on the
degraded forest and adjoining lands in order to meet the demands for
these items;
Sponsor research and extension of research findings to disseminate new
and proper technologies for the regeneration and development of
degraded forest areas and adjoining lands;
Create general awareness and help foster people's movement for
promoting afforestation and eco-development with the assistance of
voluntary agencies, non-government organisations, Panchayati Raj
institutions and others and promote participatory and
sustainable management of degraded forest areas and adjoining lands;

Coordinate and monitor the Action Plans for afforestation, tree planting,
ecological restoration and eco-development; and
Undertake all other measures necessary for promoting afforestation, tree
planting, ecological restoration and eco-development activities in the
country

245. What are the primary functions of Botanical Survey of India (BSI)?
1) Publication of National, State and District Floras
2) Survey and documentation of traditional knowledge, ethno-botany,
associated with plants
3) In-situ conservation of critically threatened species in botanical gardens
4) Identification of threatened and red list species and species rich areas
needing conservation

Select the correct answer using the codes below.

a) 1, 2 and 3 only
b) 1, 2 and 4 only
c) 2 and 4 only
d) 1 and 3 only

Correct Answer : B
Answer Justification :

Justification: Statement 3: It will be ex-situ conservation. In-situ is


done on the site, not in an external alien environment like botanical
gardens.

BSI is the apex research organization under Ministry of Environment and


Forests (MoEFCC) for carrying out taxonomic and floristic studies on wild
plant resources of country.

It was established in 1890 with objective to explore plant resources of


country and to identify plants species with economic virtues.

Apart from the roles mentioned above, it also does:

Exploration, inventorying and documentation of Phyto diversity in general


and protected areas, hotspots and fragile ecosystems in particular

Ex-situ conservation of critically threatened species in botanical gardens.

Develop National database of Indian plants, including herbarium and live


specimens, botanical paintings and illustrations, etc.
246. The principal objective of the Economics of Ecosystems and Biodiversity
(TEEB) is to

a) Mainstream the values of biodiversity and ecosystem services into


decision-making at all levels
b) Develop conservation models centred on the needs of the local
community
c) Construct an overarching economic value assessment of all ecological
services in terms of Global GDP
d) Augment the Millennium Ecosystem Assessment (MEA) framework
2005 by providing thorough scientific inputs

Correct Answer : A
Answer Justification :

Justification: is a global initiative focused on “making nature’s values


visible”. Its principal objective is to mainstream the values of biodiversity
and ecosystem services into decision-making at all levels.
It aims to achieve this goal by following a structured approach to valuation
that helps decision- makers recognize the wide range of benefits provided
by ecosystems and biodiversity, demonstrate their values in economic
terms and, where appropriate, suggest how to capture those values in
decision-making.
Learning: The TEEB study found that coral reefs generate up to $1.25
million per hectare from tourism, coastal protection, medical use and
fisheries annually.
Approximately 850 million people live within 100 km of and derive some
benefits from coral reefs, with at least 275 million depending directly on
reefs for livelihoods and sustenance.
Reef-dependence, and consequently vulnerability to reef loss, is
particularly high in small island states, in many countries in the coral
triangle, and in coastal populations in developing countries.

247. Which of these environmental/survey related organizations was


established after India’s Independence?

a) Forest Survey of India (FSI)


b) Zoological Survey of India (ZSI)
c) Botanical Survey of India (BSI)
d) Geological Survey of India (GSI)

Correct Answer : A
Answer Justification :

Learning: GSI (geology) as well as ASI (archaeology), BSI (botany), FiSI


(fisheries), FSI (forests), IIEE (ecology), NIO (oceanography), RGCCI
(population survey) and language survey), SI (cartography), and ZSI
(zoology) are key national survey organisations of India.
Option A: Forest Survey of India (FSI), is a premier national organization
under the union Ministry of Environment and Forests, responsible for
assessment and monitoring of the forest resources of the country
regularly. In addition, it is also engaged in providing the services of
training, research and extension. Established on June 1, 1981, the Forest
Survey of India succeeded the "Preinvestment Survey of Forest Resources"
(PISFR), a project initiated in 1965 by Government of India with the
sponsorship of FAO and UNDP.
Option B: ZSI was established in 1916 to promote survey, exploration and
research leading to the advancement in our knowledge of various aspects
of exceptionally rich life of the erstwhile ´ British Indian Empire´. The
survey has its genesis in the establishment of the Zoological Section of the
Indian Museum at Calcutta in 1875.
It functions as the guardian of the National Zoological Collections,
containing over a million
identified specimens from all animal groups ranging from Protozoa to
Mammals.
Option C: Botanical Survey of India (BSI), founded 13 February 1890, is
Government of India Ministry of Environment, Forest and Climate
Change's organization for survey, research and conservation of plant
resources, flora and endangered species of India, including by collecting
and maintaining germplasm and gene bank of endangered, patent and
vulnerable plant species.
Option D: The Geological Survey of India (GSI), founded in 1851, is a
Government of India Ministry of Mines organisation, one of the oldest of
such organisations in the world and the second oldest survey in India after
Survey of India (founded in 1767), for conducting geological surveys and
studies of India, and also as the prime provider of basic earth science
information to government, industry and general public, as well as the
official participant in steel, coal, metals, cement, power industries and
international geoscientific forums.

248. In a first of its kind, which of the following states is building eco-bridges
for the movement of tigers, linking tiger reserves and nearby forests?

a) Maharashtra
b) Gujarat
c) Telangana
d) Assam

Correct Answer : C
Answer Justification :

Learning: In a first of its kind, Telangana State will have eco-friendly


bridges over a canal cutting across the tiger corridor linking the Tadoba-
Andhari Tiger Reserve (TATR) in the Chandrapur district of Maharashtra
with the forests in Telangana’s Kumram Bheem Asifabad district.
The ‘eco-bridges’ will be constructed at key spots along the 72 km-long,
with laying of fertile soil to grow grass and plants over the structure, so
that fragmentation of the reserve forest is camouflaged.
National Board of Wildlife will be the nodal agency for recommending the
size and location of eco- bridge.

249. The Sustainable land and Ecosystem Management (SLEM) Programme


is a joint initiative of Government of India and Global Environmental
Facility (GEF). What is/are the objectives of SLEM?

1) To promote sustainable land management and use of biodiversity


2) To maintain the capacity of ecosystems to deliver goods and services while
taking into account climate change

Which of the above is/are correct?

a) 1 only
b) 2 only
c) Both 1 and 2
d) None

Correct Answer : C
Answer Justification :

Learning: The SLEM Programme is a joint initiative of the Government


of India and the Global Environmental Facility (GEF) under the latter’s
Country partnership Programme (CPP).
The objective of the SLEM Programmatic Approach is to promote
sustainable land management and use of biodiversity as well as maintain
the capacity of ecosystems to deliver goods and services while taking into
account climate change
It is a long-term and strategic arrangement of individual yet interlinked
projects aimed at achieving large-scale impacts on the global environment.

250. Consider the following about the Centre for Science and Environment
(CSE) in India.

1) It is a not-for-profit public interest research and advocacy organisation.


2) It provides grants to Central Universities for research in ecology and
zoology.
3) It publishes “Down to Earth” on a regular basis.

Select the correct answer using the codes below.

a) 1 and 3 only
b) 2 and 3 only
c) 3 only
d) 1 and 2 only
Correct Answer : A
Answer Justification :

Justification: Statement 1: Centre for Science and Environment (CSE) is


a not-for-profit public interest research and advocacy organisation based
in New Delhi
Statement 2: Established in 1980, CSE works as a think tank on
environment-development issues in India, poor planning, climate shifts
devastating India's Sundarbans and advocates for policy changes and
better implementation of the already existing policies.
CSE uses knowledge-based activism to create awareness about problems
and propose sustainable solutions.
The activities are carried out through lectures, field trips, publications,
exhibitions on the various issues they take up, meetings and workshops.
Statement 3: Down to Earth - a fortnightly environment magazine is a
prominent publication of CSE.

251. With reference to a Marine Protected Area (MPA), consider the following
statements.

1) It is a space in the ocean where all human activities are prohibited.


2) All coastal states in India have at least one MPA.
3) An estuary or coastal area can be regulated as an MPA.

Select the correct answer using the codes below.

a) 1 and 3 only
b) 2 and 3 only
c) 1, 2 and 3
d) 1 only

Correct Answer : B
Answer Justification :

Justification: A marine protected area (MPA) is essentially a space in


the ocean where human activities are more strictly regulated than the
surrounding waters - similar to parks we have on land.
These places are given special protections for natural or historic marine
resources by local, state, territorial, native, regional, or national
authorities.
MPAs can be conserved for a number of reasons including economic
resources, biodiversity conservation, and species protection.
They are created by delineating zones with permitted and non-permitted
uses within that zone.
MPAs can span a range of habitats such as the open ocean, coastal areas,
inter-tidal zones, estuaries etc.
252. Consider the following about Animal Welfare Board of India (AWBI).

1) It is an advisory body established by an executive resolution.


2) It frames rules and regulations on the humane treatment of animals.
3) It recognises Animal Welfare Organisations (AWOs) by granting them
recognition if they meet its guidelines.
4) It provides financial assistance to recognised AWOs.

Select the correct answer using the codes below.

a) 1 and 2 only
b) 1, 2 and 3 only
c) 3 and 4 only
d) 2, 3 and 4 only

Correct Answer : C
Answer Justification :

Justification: AWBI is a statutory advisory body established in 1962


under Section 4 of The Prevention of Cruelty to Animals Act, 1960.
The Board consists of 28 Members, who serve for a period of 3 years.
Several government organisations, along with animal rights activists and
parliamentarians, are represented on the Board.
Functions of AWBI:

It keeps the law in force in India for the Prevention of Cruelty to Animals
under constant study and to advise the government on the amendments to
be undertaken in any such law from time to time.
It recognises Animal Welfare Organisations by granting them recognition
if they meet its guidelines.
It also appoints key people to the positions of (Hon) Animal Welfare
Officers, who serve as the key point of contact between the people, the
government and law enforcement agencies.
It suggests changes to laws and rules about animal welfare issues. It also
offers guidance to organisations and officials such as police to help them
interpret and apply the laws.
It issues publications to raise awareness of various animal welfare issues
and engages in capacity building of stakeholders.
It provides financial assistance to recognised AWOs, who submit
applications to the Board

253. Arrange the following in decreasing order of protection and


funding given by the government in connection with
safeguarding wildlife, biodiversity and forests:

a) Protected forests> National Park> Wildlife Sanctuary


b) Reserved forests> National Park> Protected Forests> Wildlife
Sanctuary
c) National Park> Wildlife Sanctuary> Reserved Forests >
Protected forests
d) All of the above enjoy the same level of protection and funding.
Correct Answer : C

Answer Justification :

Justification: Typically, reserved forests are often upgraded to


the status of wildlife sanctuaries, which in turn may be upgraded
to the status of national parks, with each category receiving a
higher degree of protection and government funding.

For example, Sariska National Park was declared a reserved


forest in 1955, upgraded to the status of a wildlife sanctuary in
1958, becoming a Tiger Reserve in 1978.

Sariska became a national park in 1992, though primary


notification to declare it as a national park
was issued as early as 1982.

254. Gaj Yatra Campaign is being organized by

a) Wildlife Trust of India (WTI)


b) International Union for Conservation of Nature (IUCN)
c) United Nations Environment Programme (UNEP)
d) World Wildlife Fund (WWF)
Correct Answer : A

Answer Justification :

Learning: It is a 15 months nationwide campaign led by the WTI to


protect elephants on the occasion of World Elephant Day.

During the period of the campaign, artists and craftsmen will create life-
size works on the theme of elephants in places along the route of the
roadshow, covering 12 states that have wild elephants, using local art and
craft.

Specially fabricated vehicles will be deployed to display these on pre-


determined routes with campaigners.

255. Consider the following about The Energy Resources Institute (TERI).

1) It is a non-profit policy research organisation.


2) It is an attached agency of the International Centre for Climate Governance
(ICCG).

Which of the above is/are correct?


a) 1 only
b) 2 only
c) Both 1 and 2
d) None
Correct Answer : A

Answer Justification :

Justification: Statement 1: TERI conducts research work in the fields of energy,


environment and sustainable development.
Statement 2: The Energy Resources Institute (TERI) was ranked second among
world’s best climate think tank by the International Centre for Climate
Governance (ICCG).

ICCG is an internationally renowned centre founded in 2009.

The main focus of ICCG activities is the design of climate policy and related
governance issues.
256. With reference to Indian Green Building Council (IGBC),
consider the following statements.

1) It is a body under Department of Urban Development.


2) It organizes the Green Building Congress.
3) It offers green building rating programmes.

Select the correct answer using the codes below.


a) 1 only
b) 2 and 3 only
c) 2 only
d) 3 only
Correct Answer : B

Answer Justification :

Justification: The IGBC is part of the Confederation of Indian


Industry (CII) formed in 2001.

It offers a wide array of services including developing new green


building rating programmes, green building training
programmes and certification services.

It also organises Green Building Congress, its annual flagship


event on green buildings.
It closely works with several State Governments, Central
Government, World Green Building Council etc.

257. The National Tiger Conservation Authority (NTCA) is set up under the
Chairmanship of the

a) Prime Minister of India


b) Minister for Environment, Forests and Climate Change (MoEFCC)
c) Director, Wildlife Institute of India (WII)
d) Secretary to the Government of India, MoEFCC
Correct Answer : B

Answer Justification :

Learning: NTCA is a statutory body under the MoEFCC constituted


under enabling provisions of the Wildlife (Protection) Act, 1972, as
amended in 2006, for strengthening tiger conservation, as per powers
and functions assigned to it under the said Act.

The Authority, inter alia, would lay down normative standards,


guidelines for tiger conservation in the Tiger Reserves, apart from
National Parks and Sanctuaries.
It would provide information on protection measures including future
conservation plan, tiger estimation, disease surveillance, mortality survey,
patrolling, report on untoward happenings and such other management
aspects as it may deem fit, including future plan for conservation.

258. National Water Development Agency (NWDA) carries out which of


these functions?

1) Carry out detailed surveys and investigations of possible reservoir sites


and interconnecting links for peninsular rivers
2) Monitor, regulate and control water flows between inter-state river basin
embankments
3) Assessing groundwater potential in the country and making adequate
arrangements for exploration of groundwater

Select the correct answer using the codes below.


a) 1 only
b) 2 only
c) 1 and 3 only
d) 1, 2 and 3
Correct Answer : A

Answer Justification :

Justification: It was set up in July, 1982 as Autonomous Society


under the Societies Registration Act, 1860, to carry out the water
balance and other studies on a scientific and realistic basis for optimum
utilisation of Water Resources.

Functions include:

To prepare feasibility report of the various components of the scheme


relating to Peninsular Rivers development and Himalayan Rivers
development.

To prepare detailed project report of river link proposals under National


Perspective Plan for Water Resources Development after concurrence of
the concerned States.

259. Consider the following statements. National Mission for Clean


Ganga (NMCG)

1) is the implementation wing of National Ganga Council


2) is a statutory authority
3) authorized to approve projects for cleaning Ganga
Select the correct answer using the codes below.

a) 1 and 2 only
b) 1 and 3 only
c) 2 only
d) 2 and 3 only

Correct Answer : B

Answer Justification :

Justification: Statement 1: The Council is also called National


Council for Rejuvenation, Protection and Management of River
Ganga.

In 2016, National Ganga Council has replaced National Ganga


River Basin Authority(NGRBA) which was constituted under the
provisions of the Environment (Protection) Act (EPA), 1986.

Statement 2 and 3: It was established in 2011 as a registered


society under Societies Registration Act, 1860. It has a two-tier
management structure and comprises of Governing Council and
Executive Committee.

Both of them are headed by Director General (DG), NMCG.


Executive Committee is authorized to approve projects under
mission up to Rs.1000 crore.

Statement 3: Similar to structure at national level, State


Programme Management Groups (SPMGs) acts as
implementing arm of State Ganga Committees.

This structure attempts to bring all stakeholders on one platform


to take a holistic approach towards the task of Ganga cleaning
and rejuvenation.

REPORTS

260. Consider the following statements.

1) Forest cover in India is continuously declining since the beginning of this


millennium.
2) The carbon stock in the Indian forests has been showing a declining trend
in recent decade due to climate change.

Which of the above is/are correct?


a) 1 only
b) 2 only
c) Both 1 and 2
d) None

Correct Answer : D
Answer Justification :

Justification: Statement 1: India is one of the few countries of the world,


where forest cover is on the rise.

The total Forest and Tree cover of the country as per 2015 assessment in
the state of Forest Report is nearly 8 lakh sq km which is 24.16 per cent of
the geographical area of the country

Statement 2: The carbon stock in the Indian forests estimated to be


around 7 billion tones and has been showing an upward trend.

261. Global Biodiversity Outlook (GBO) is the flagship publication of

a) IUCN
b) World Wildlife Fund (WWF)
c) United Nations Environment Programme (UNEP)
d) None of the above

Correct Answer : D
Answer Justification :
Justification: It is published by the Convention on Biological Diversity.
It is a periodic report that summarizes the latest data on the status and
trends of biodiversity and draws conclusions relevant to the further
implementation of the Convention.

The report draws on various sources of information to provide a mid-term


assessment of progress towards the implementation of the Strategic Plan
for Biodiversity 2011-20 which is to supplemented with the Aichi
biodiversity targets.

262. Our Planet, Atlas and Global Environment Outlook (GEO) are flagship
publications of

a) United Nations Environment Programme (UNEP)


b) International Union for Conservation of Nature (IUCN)
c) Greenpeace
d) World Wildlife Fund (WWF)

Correct Answer : A
Answer Justification :

Learning: Our Planet is UNEP's magazine for environmentally


sustainable development. Each issue of Our Planet concentrates on one
specific theme, tying into international conferences (for e.g. the 10th
anniversary of the Basel Convention), meetings (e.g. the Commission on
Sustainable Development), and events.
The Atlas of Our Changing Environment publications use a combination
of ground photographs, current and historical satellite images, and
narrative based on extensive scientific evidence to illustrate how humans
have altered their surroundings.
Global Environment Outlook (GEO) is a series of reports on the
environment issued periodically by the United Nations Environmental
Programme (UNEP). The GEO project was initiated in response to the
environmental reporting requirements of UN Agenda 21.

263. Consider the following statements with reference to the: Climate Change
Performance Index (CCPI) 2018

1) has been released by Greenpeace


2) evaluates all the parties of the UNFCCC
3) major part of evaluation is based on objective indicators of emissions
trend and emissions level
4) India is ranked in the bottom 10 countries

Select the correct answer using the codes below.

a) 1 and 4 only
b) 3 only
c) 1, 2 and 3 only
d) 2 and 3 only

Correct Answer : B
Answer Justification :

Justification: The report was made public on the sidelines of the UN


Climate Change negotiations (COP23) in Bonn.
CCPI is an instrument (released by Germanwatch – a NGO) covering 58
countries and supposed to enhance transparency in international climate
politics.
Its aim is to encourage political and social pressure on those countries
which have, up to now, failed to take ambitious actions on climate
protection as well as to highlight countries with best-practice climate
policies.
Statement 3: On the basis of standardised criteria, the index evaluates and
compares the climate protection performance of 58 countries that are,
together, responsible for more than 90% of global energy-related CO2
emissions.
80% of the evaluation is based on objective indicators of emissions trend
and emissions level.
20% of the index results are built upon national and international climate
policy assessments by more than 200 experts from the respective
countries.
Statement 4: India is ranked 14th, an improvement from its 20th position
last year. China, with its high emissions and growing energy use over the
past five years, still ranks 41st.

264. Consider the following about the Wildlife Picture Index (WPI).

1) It is a biodiversity index that uses camera trap data to quantitatively


measure changes in species variation over time.
2) It was developed jointly by the World Wildlife Fund and Society for
Conservation of Mammals.

Which of the above is/are correct?

a) 1 only
b) 2 only
c) Both 1 and 2
d) None

Correct Answer : A
Answer Justification :

Justification: WPI was developed jointly by the Wildlife Conservation


Society and the Zoological Society of London as an indicator derived from
primary camera trap data.
It monitors ground-dwelling tropical medium and large mammals and
birds, species that are important economically, aesthetically and
ecologically.
The Tropical Ecology and Assessment Monitoring (TEAM) Network,
which implements the largest global camera trap network in tropical
forests (16 sites, 14 countries and approximately 1000 camera traps
deployed over 2000km 2 that are monitored annually), adopted this
indicator as a way to synthesize and understand the effects of climate
change and land use change on tropical terrestrial mammal and bird
diversity.
Tropical Ecology Assessment and Monitoring (TEAM) adopted the WPI as
a way to quickly synthesize and understand how tropical wildlife is
changing and why.
The WPI is sensitive to changes in the number of species, their relative
occurrence and evenness over time.

The WPI is not limited to camera trap data, but can also be used for other
presence/absence data, such as information collected using sound
sensors.
The WPI can be aggregated at the level of a site (e.g., a TEAM Site), sites
within a region, sites within a continent or globally.

265. Consider the following about the Ocean Health Index (OHI ).

1) OHI assessments are independently-led assessments using the OHI


framework that are conducted by any group external to the OHI team
2) OHI assessments are often conducted at national and continental scales to
assess the overall health of oceans.

Which of the above is/are correct?

a) 1 only
b) 2 only
c) Both 1 and 2
d) None

Correct Answer : A
Answer Justification :

Justification: OHI is the first assessment tool that scientifically


measures key elements from all dimensions of the ocean’s health —
biological, physical, economic and social — to guide decision makers
toward the sustainable use of the ocean.
OHI is operational and included in the CBD's list of indicators
First completed in 2012, more than 25 countries and regions till now have
embarked on independent Ocean Health Assessments.
OHI assessments are independently-led assessments using the OHI
framework that are conducted by any group external to the OHI team.
OHI assessments are often conducted at smaller, subnational spatial
scales.
Independent assessments use the same framework as the global
assessments, but allow for exploration of factors influencing ocean health
at the smaller scales where policy and management decisions are made.
Conservation International is a founder of the Ocean Health Index and
serves as the managing partner.

266. ‘State of the World's Forests’ report is published by

a) United Nations Environment Programme (UNEP)


b) Food and Agriculture Organization (FAO)
c) United Nations Framework Convention on Climate Change (UNFCCC)
d) United Nations Development Programme (UND)
Correct Answer : B
Answer Justification :

Learning: The State of the World's Forests reports on the status of


forests, recent major policy and institutional developments and key issues
concerning the forest sector.
Some of its observations in 2014 report were:
Forest products make a significant contribution to the shelter of at least
1.3 billion people, or 18 percent of the world’s population.
A major contribution of forests to food security and health is the provision
of woodfuel to cook and sterilize water.
The formal forest sector employs some 13.2 million people across the
world and at least another 41 million are employed in the informal sector.
Recognition of the value of forest services, such as erosion protection and
pollination, is essential to sound decision-making.

267. The Global Climate Risk Index 2018 was published by

a) United Nations Environment Programme


b) Conservation International
c) Germanwatch
d) UNFCCC Secretariat Correct Answer : C

Answer Justification :

Learning: The index has put India amongst the six most vulnerable
countries in the world.

This year’s 13th edition of the analysis reconfirms earlier results of the
Climate Risk Index: less developed countries are generally more affected
than industrialised countries.
Regarding future climate change, the Climate Risk Index may serve as a
red flag for already existing vulnerability that may further increase in
regions where extreme events will become more frequent or more severe
due to climate change.
Germanwatch, is a non-profit, non-governmental organization based in
Bonn, Germany. It seeks to influence public policy on trade, the
environment, and relations between countries in the industrialized north
and underdeveloped south

268. India State of Forest Report (ISFR) 2017 was published by

a) IUCN – India
b) WWF – India
c) Union Ministry of Environment, Forests and Climate Change (MoEFCC)
d) Centre for Science and Environment (CSE)

Correct Answer : C
Answer Justification :

Learning: The environment ministry has released the India State of


Forest Report (ISFR) 2017.
The ISFR report is published every two years.
The India State of Forest Report 2017 is the 15th such report in the series.
The report for the first time contains information on decadal change in
water bodies in forest during 2005-2015, forest fire, production of timber
from outside forest, state wise carbon stock in different forest types and
density classes.
India’s forest cover increased by 6,778 sq km over the last two years. The
increase, based on satellite data and subsequent ‘ground truthing’, has put
the total forest cover at 7,08,273 sq km which is 21.54% of the country’s
geographical area.
Andhra Pradesh, Karnataka, Kerala, Odisha and Telangana saw increase
in their green footprint during the last two years though there is a
worrying decline in six northeastern states, including a shrinkage of 630
sq km in the eastern Himalayas.

269. Bruntdland Report is usually associated with

a) Non-Aligned Movement
b) Sustainable Development
c) Five Principles of Peaceful Coexistence – India and China
d) Boundary dispute between India and Pakistan

Correct Answer : B
Answer Justification :

Learning: This was a report on the concept of sustainable development


and one of the first sources that made the use of the term ‘sustainable
development’ popular.
It was the basis of further negotiations in several bodies of the 1970s and
1980s followed up by the Earth Summit 1992.
It was the Earth Summit that acknowledged the global environmental
consciousness internationally, perhaps for the first time.

INTERNATIONAL EFFORTS

270. The objective of the National Carbonaceous Aerosols Programme


(NCAP) is to

1) Prepare inventory of the carbonaceous aerosols including black carbon


2) Study the impact of aerosols on regional and global climate

Which of the above is/are correct?

a) 1 only
b) 2 only
c) Both 1 and 2
d) None

Correct Answer : C
Answer Justification :

Background: Carbonaceous aerosol emissions arise from energy use and


the burning of forest, grasslands and agricultural residues. The emissions
lead to air-quality degradation and related health-risks on local to regional
scales and to climate impacts on regional to global scales. In south Asia,
there is dominance of small combustion sources (e.g. residential cooking
and heating), less- developed industry (e.g. brick kilns), and vehicular
emission.

Learning: It was launchedunder Climate Change Action Programme


(CCAP), has been launched. The objective of the project is to prepare
inventory of the carbonaceous aerosols including black carbon, develop
national emission factors, conduct modelling studies and their impact on
regional and global climate.

A consortium of 17 leading research institutions of the country are


engaged in the task which is coordinated by IIT Bombay.

271. The New Urban Agenda is an important outcome document agreed upon
at the

a) Paris Climate Conference, 2016


b) Adis Ababa Climate Conference, 2014
c) Habitat III cities conference, 2016
d) 2015 United Nations Climate Change Conference

Correct Answer : C
Answer Justification :

Learning: It will guide the efforts around urbanization of a wide range of


actors — nation states, city, United Nations programmes and civil society
— for the next 20 years laying the groundwork for policies.
Habitat III, the United Nations Conference on Housing and Sustainable
Urban Development, took place in Quito, Ecuador, from 17 – 20 October
2016.
Habitat III was one of the first United Nations global summits after the
adoption of the Post-2015 Development Agenda.
It gave the opportunity to open discussions on important urban challenges
and questions, such as how to plan and manage cities, towns and villages
for sustainable development.

272. The blue label certification granted by the Marine Stewardship Council
essentially depicts that the seafood that is labelled blue is

a) Seafood extracted in a sustainable and eco-friendly way


b) Legally extracted from international waters
c) The most exotic seafood available in the international market
d) Safe for human consumption

Correct Answer : A
Answer Justification :

Learning: Marine Stewardship Council is an international non-profit


organisation.
It grants blue label certification to the fishery across the world and helps
create more sustainable seafood market.
MSC certified fisheries minimise their impacts on the whole marine
environment to ensure healthy, thriving oceans for the future.
Purchases of MSC labelled seafood create an incentive for more fisheries,
retailers and restaurants to produce and sell certified sustainable seafood.
India has special reason this time to celebrate the World Fisheries Day on
November 21, as it received international fund for the promotion of
sustainable fishing practices.
MSC has awarded a fund for promoting sustainable practices in India’s
fisheries sector recently.
Based on this funding, WWF-India and the CMFRI will create fishery
management and action plans to ensure the sustainability of both baitfish
and tuna fisheries in Lakshadweep, a region which depends on fishing for
income and as a food source.
Learning: Besides India, the Council also awarded the fund pre-
assessment and action plan development for octopus fisheries in Senegal,
improvement action plan for stone crab fishing in Chile, stock assessment
for baitfish fisheries in Indonesia, and crayfish project in China.
In all, the MSC has granted a total funds £200,000 to five projects that
would aid small scale and Developing World fisheries in achieving
sustainability.
The award is part of the MSC’s Global Fisheries Sustainability Fund
(GFSF) that was established in 2015 in recognition of the difficulty that
these fisheries have in reaching the MSC Standard.
More than 300 fisheries in 34 countries are certified to the MSC’s
Standard.

273. The Restoration Opportunities Assessment Methodology (ROAM),


produced by IUCN and the World Resources Institute (WRI)

1) provides a flexible and affordable framework for countries to rapidly


identify and analyse areas that are primed for forest landscape restoration
(FLR)
2) Gives a method for Environmental Impact Assessment (EIA) for
developing countries that are parties to the Convention on Biological
Diversity (CBD)

Which of the above is/are correct?

a) 1 only
b) 2 only
c) Both 1 and 2
d) None

Correct Answer : A
Answer Justification :

Justification: ROAM will also enable countries to define and implement


national or subnational contributions to the Bonn Challenge and
concurrently allow nations to meet existing international commitments
under the Convention on Biological Diversity, United Nations Convention
to Combat Desertification and the United Nations Framework to Combat
Climate Change.
A ROAM assessment can be undertaken by a small team through
collaborative engagement with stakeholders, and can deliver the following
products:
Identified priority areas for restoration;
A shortlist of the most relevant and feasible restoration intervention types
across the assessment area;
Quantified costs and benefits of each intervention type;
Estimated values of additional carbon sequestered by these intervention
types;
Analysis of the finance and investment options for restoration in the
assessment area; and
A diagnostic of ‘restoration readiness’ and strategies for addressing major
policy and institutional bottlenecks.
Learning: and to identify specific priority areas at a national or sub-
national level.
Forest landscape restoration (FLR) is an initiative supported by the IUCN
that focuses on regaining ecological functionality and enhancing human
well-being across deforested or degraded forest landscapes.
FLR is more than just planting trees – it is restoring a whole landscape to
meet present and future needs.
It is long-term because it requires a multi-year vision of the ecological
functions.
It integrates a number of guiding principles, including: Focus on
landscapes, restore
functionality, involve stakeholders, tailor to local conditions and Avoid
further reduction of natural forest cover.

274. Consider the following: Circumarctic Environmental Observatories


Network (CEON)

1) is a network of terrestrial and freshwater observation platforms, science


experts and network partners
2) promotes the collection and dissemination of environmental data from the
Arctic
3) is located hundred feet underneath the thick layer of arctic sea

Select the correct answer using the codes below.

a) 1 and 2 only
b) 2 and 3 only
c) 1 and 3 only
d) 1, 2 and 3

Correct Answer : A
Answer Justification :

Background and Learning: The Arctic is experiencing dramatic


changes in climate and patterns of human land use. Environmental and
socioeconomic drivers associated with these changes originate both within
and outside of the Arctic system.
Change detection & predictive power of these changes is low and are
limited/ threatened by the lack of sustained environmental observation
time series in northern high latitudes.
A circum-arctic environmental observatories network that can provide
adequate, diverse and sustained time series observations has the potential
to dramatically improve our understanding of the Arctic system and how it
may continue to respond to a variety of environmental and societal
changes forecast for the region.
So, CEON's mission is to strengthen the capacity for emerging monitoring,
research and policy needs at high northern latitudes by making data
available that is adequate and suitable for answering and addressing a
series of well-defined key scientific questions and uncertainties.
CEON observation platforms include land and freshwater observatories,
research infrastructures, former research sites where retrospective
analyses are being or can be undertaken, data and image archive centres
and community monitoring programs.

275. The Climate and Development Knowledge Network (CDKN) is a/an

a) Wing of the United Nations Environment Programme (UNEP)


b) Alliance of private organisations and think tanks that links developing
countries with experts on climate change
c) Inter-Governmental organization established at the Kigali meet of the
Conference of Parties (COP) to the UNFCCC
d) Non-governmental organization established by the IUCN

Correct Answer : B
Answer Justification :

Learning: CDKN is run by an alliance of organisations, headed by


PricewaterhouseCoopers and including the Overseas Development
Institute, a think tank on development and humanitarian issues.
It links developing countries with experts on climate change. Funded by
the UK's Department for International Development and the Directorate-
General for International Cooperation, its aim is to help developing
nations adapt to the consequences of climate change and build capacity
for a low- carbon economy.
CDKN does this by combining research, advisory services and knowledge
management in support of locally owned and managed policy processes.

276. The initiative ‘Future Earth’ was launched in

a) Cancun Declaration 2016 – to the Convention on Biological Diversity


(CBD)
b) First Earth Summit
c) Rio 20 Summit 2012
d) Bali Climate Change Conference, 2007

Correct Answer : C
Answer Justification :
Learning: It is a 10-year international research programme which aims
to build knowledge about the environmental and human aspects of Global
change, and to find solutions for sustainable development.
Future Earth was launched in June 2012, at the UN Conference on
Sustainable Development (Rio 20).
It aims to increase the impact of scientific research on sustainable
development. Scientific research and synthesis in Future Earth is carried
out by a number of international networks, known as ‘core projects’, many
of which were launched under the umbrella of the existing four global
environmental change programmes, DIVERSITAS, the International
Geosphere-Biosphere Programme (IGBP), the International Human
Dimensions Programme (IHDP) and the World Climate Research
Programme (WCRP).

277. The “Transformative Carbon Asset Facility (TCAF)”, that would help
developing countries pay for emission reductions, was launched by

a) World Bank
b) UNFCCC
c) WWF
d) UNEP

Correct Answer : A
Answer Justification :

Learning: The World Bank has launched a $500 million dollars


Transformative Carbon Asset Facility (TCAF) to help developing countries
pay for emission reductions and combat climate change.
It is market-based scheme specially designed to help developing countries
and would establish the world’s first programmatic carbon market.
The scheme will reward countries for reducing emissions by paying a fee
for each tonne of carbon dioxide (CO2) reduced. The facility will help
countries implement their Intended Nationally Determined Contributions
(INDCs).

278. Leadership in Energy and Environmental Design (LEED)

a) is a rating system to evaluate the environmental performance of a building


b) is one of the sub-forums of UN Habitat
c) is a offshoot of the GRIHA Council that certifies green buildings in India
d) A non-profit organization vouching for sustainable building design

Correct Answer : A
Answer Justification :

Learning: It was devised by the United States Green Building Council


(USGBC) to evaluate the environmental performance of a building and
encourage market transformation towards sustainable design.
The system is credit-based, allowing projects to earn points for
environmentally friendly actions taken during construction and use of a
building. LEED was launched in an effort to develop a “consensus-based,
market-driven rating system to accelerate the development and
implementation of green building practices.
LEED is one of the most popular green building certification programs
used worldwide.
Developed by the non-profit U.S. Green Building Council (USGBC) it
includes a set of rating systems for the design, construction, operation,
and maintenance of green buildings, homes, and neighborhoods that aims
to help building owners and operators be environmentally responsible and
use resources efficiently.

279. World Environment Day (WED) was established by

a) World Wildlife Fund


b) IUCN
c) United Nations General Assembly
d) Conservation International

Correct Answer : C
Answer Justification :

Learning: India will be the Global Host for World Environment Day
(WED) 2018 which is observed annually on 5 June.

The central theme for this year is ‘Plastic Pollution”.

WED is observed annually on 5 June for encouraging worldwide


awareness and action for the protection of our environment.

It was established by United Nations General Assembly to mark opening


of United Nations Conference on the Human Environment in 1972.

The first World Environment Day was observed in 1973 and since then it
is being held every year with different themes.

WED serves as a flagship campaign for raising awareness on emerging


environmental issues from marine pollution, human overpopulation, and
global warming

280. ValuES is a global project that concerns with

a) Assessing ecosystem services


b) Integrating global trade supply networks
c) Promoting the benefits of economic development to LDCs
d) Intergovernmental committee for Ethical Systems

Correct Answer : A
Answer Justification :

Justification: Assessing Ecosystem Services – ValuES project aids


decision-makers in our partner countries in recognizing and integrating
ecosystem services into policy making, planning and implementation of
specific projects.
It works to:
Analyse existing approaches to ecosystem service assessment and
valuation. Identify best practice case studies and generate lessons learned.
Develop an inventory of methods, tools, and indicators to guide
practitioners and decision makers in the selection of approaches to
integrating ecosystem services into different sectoral and other policies
Provide country-specific advisory services to practitioners and decision
makers to identify and include the value of ecosystem services in specific
policies and projects.
Develop and facilitate trainings regarding the selection and application of
methods and tools for the as-sessment and valuation of ecosystem services
and how to integrate them into decision-making processes.

281. The Tropical Ecology, Assessment and Monitoring (TEAM) Network


serves as a global public resource by making all data and derived works
publicly available. It was established by

a) Conservation International (CI)


b) United Nations Environment Programme (UNEP)
c) International Union for Conservation of Nature (IUCN)
d) Green Earth Foundation

Correct Answer : A
Answer Justification :

Learning: Originally created by Conservation International (CI), it is


now a partnership among CI, the Missouri Botanical Garden, the
Smithsonian Institution and the Wildlife Conservation Society.
TEAM's global network of scientists is collecting and distributing near-
real-time data on trends in biodiversity, climate, land cover change and
ecosystem services
The idea behind TEAM is to measure and compare plants, terrestrial
mammals, ground-dwelling birds and climate using a standard
methodology in a range of tropical forests, from relatively pristine places
to those most affected by people.

TEAM currently operates in sixteen tropical forest sites across Africa, Asia
and Latin America supporting a network of scientists committed to
standardized methods of data collection to quantify how plants and
animals respond to pressures such as climate change and human
encroachment
282. With reference to the Biodiversity Indicators Partnership (BIP), consider
the following about it.

1) It coordinates the development and delivery of biodiversity indicators for


use by the Convention on Biological Diversity (CBD).
2) Its secretariat is hosted at the UNEP - World Conservation Monitoring
Centre.
3) BIP brings together several organizations on indicator development to
provide the most comprehensive information on biodiversity trend.

Select the correct answer using the codes below.

a) 1 only
b) 2 and 3 only
c) 1 and 2 only
d) 1, 2 and 3

Correct Answer : D
Answer Justification :

Justification: The primary role of BIP is to serve the global user


community by responding to the indicator requests of the CBD and other
biodiversity-related Conventions, for IPBES, for reporting on the
Sustainable Development Goals, and for use by national and regional
governments.
The BIP aims to:
Support the development and use of indicators to measure progress in
achieving all the Aichi Biodiversity Targets of the Strategic Plan for
Biodiversity 2011-2020.
Support the development and use of biodiversity-related indicators in
progress reporting of other Multilateral Environmental Agreements
(MEAs), as well as to support statistical requirements, intergovernmental
processes and development processes, including the Intergovernmental
Platform for Biodiversity and Ecosystem Services (IPBES) and the
Sustainable Development Goals (SDGs).
Strengthen capacity at the national level for indicator development and
use in implementation and reporting of National Biodiversity Strategies
and Action Plans (NBSAPs) and the SDGs.
The Partnership currently brings together over fifty organizations working
internationally on indicator development to provide the most
comprehensive information on biodiversity trends.
As well as the Partners, the Partnership has a secretariat, hosted at UNEP-
WCMC, and a Steering Committee.

283. The Earth Systems Science Partnership (ESSP) consists of

1) International Geosphere-Biosphere Programme (IGBP)


2) United Nations Framework Convention on Climate Change (UNFCCC)
3) International Human Dimensions Programme on Global Environmental
Change (IHDP)
4) World Climate Research Programme (WCRP)

Select the correct answer using the codes below.


a) 1 and 3 only
b) 2 and 4 only
c) 1, 3 and 4 only
d) 1, 2, 3 and 4

Correct Answer : C

Answer Justification :

Justification: The Earth System Science Partnership (ESSP) is a


partnership under the auspices of the International Council for Science
(ICSU) for the integrated study of the Earth system, the ways that it is
changing, and the implications for global and regional sustainability. It
includes Diversitas (an integrated programme of biodiversity science),
IGBP, WCRP and IHDP.

The central activities of the ESSP are Joint Projects on issues of global
sustainability, designed to address the global environmental change
aspects of four critical issues for human well-being: energy and the carbon
cycle (GCP), food security (GECAFS), water resources (GWSP) and human
health (GEC&HH).

284. The United Nations designated the first Monday of October of every year
as World Habitat Day. UN- Habitat’s focal areas include

1) Inclusive housing and social services.


2) Affordable and sustainable transport and energy.
3) Safe and clean drinking water and sanitation.
4) Healthy air quality
5) Job creation

Select the correct answer using the codes below.

a) 1, 2 and 5 only
b) 3 and 4 only
c) 2, 3 and 5 only
d) 1, 2, 3, 4 and 5

Correct Answer : D
Answer Justification :

Justification: Each year, World Habitat Day takes on a new theme to


promote sustainable development policies that ensure adequate shelter for
all.
2017 theme is Housing Policies: Affordable Homes It also covers:
Promotion, protection, and restoration of green urban spaces.
Improved urban planning and slum upgrading.
Better waste management.
A safe and healthy living environment for all — with particular
consideration for children, youth, women, elderly and disabled.
285. Tiger range countries (TRCs) are those where the tigers roam freely.
These countries include

1) Bangladesh
2) Indonesia
3) Malaysia
4) Nepal
5) Russia

Select the correct answer using the codes below.

a) 1, 3 and 4 only
b) 1, 2, 4 and 5 only
c) 2 and 5 only
d) 1, 2, 3, 4 and 5

Correct Answer : B
Answer Justification :

Justification: The 13 tiger range countries include Bangladesh, Bhutan,


Cambodia, China, India, Indonesia, Lao PDR, Malaysia, Myanmar, Nepal,
Russia, Thailand, and Vietnam.
Tiger is an endangered animal listed in the Convention on International
Trade in Endangered Species (CITES). According to the World Wildlife
Fund (WWF), tigers have lost 93% of their historical range.

286. With reference to the Protocol on Environmental Protection to the


Antarctic Treaty, consider the following:

1) Any activity relating to mineral resources, other than scientific research,


shall be prohibited.
2) Environmental assessment for all activities, including tourism, shall be
done.
3) Member states should be prepared for emergency response actions in the
area.

Select the correct answer using the codes below.

a) 1 only
b) 2 and 3 only
c) 2 only
d) 1, 2 and 3

Correct Answer : D
Answer Justification :

Justification: Also known as the Antarctic-Environmental Protocol, or


the Madrid Protocol, it is a part of the Antarctic Treaty System.
It provides for comprehensive protection of the Antarctic environment
and dependent and associated ecosystems.
Other provisions (apart from the ones in the Q) are:
Article 3 states that protection of the Antarctic environment as a
wilderness with aesthetic and scientific value shall be a "fundamental
consideration" of activities in the area.
Article 11 creates a Committee for Environmental Protection for the
continent.
Articles 18-20 arrange for arbitration of international disputes regarding
Antarctica.
Learning: The main mineral resource known on the continent is coal.
The most valuable resources of Antarctica lie offshore, namely the oil and
natural gas fields found in the Ross Sea. Exploitation of all mineral
resources is banned until 2048 by the Protocol on Environmental
Protection to the Antarctic Treaty.

287. Consider the following statements.

1) UNESCO’s Man and Biosphere (MAB) Programme identifies human


heritages and supports them financially and technically.
2) World Network of Biosphere Reserves (WNBR) covers internationally
designated protected areas as recognized by UNESCO.

Which of the above is/are correct?

a) 1 only
b) 2 only
c) Both 1 and 2
d) None

Correct Answer : B
Answer Justification :

Justification: Statement 1: UNESCO MAB aims to establish a scientific


basis for the improvement of relationships between people and their
environments.
It does not fund human heritages. It instead provides a unique platform
for cooperation on R&D, capacity-building and networking to share
knowledge on three interlinked issues: biodiversity loss, climate change
and sustainable development.
Statement 2: They are recognized under UNESCO's MAB.
There are 142 biosphere reserves recognized as part of the World Network
of Biosphere Reserves in Asia.
Nilgiri (2000), Gulf of Mannar (2001), Sunderbans (2001) are some of the
reserves included in WNBR.

288. The United Nations Programme on Reducing Emissions from


Deforestation and Forest Degradation (or UN-REDD Programme) is a
collaborative programme of the

1) Food and Agriculture Organization of the United Nations (FAO)


2) United Nations Educational, Scientific and Cultural Organization
(UNESCO)
3) United Nations Framework Convention on Climate Change (UNFCCC)
4) United Nations Development Programme (UNDP)

Select the correct answer using the codes below.


a) 1 and 2 only
b) 3 and 4 only
c) 1 and 4 only
d) 1, 3 and 4 only

Correct Answer : C
Answer Justification :

Justification: It was created in 2008 in response to the UNFCCC


decisions on the Bali Action Plan and REDD at COP.
It should not be confused with "REDD ", a voluntary climate change
mitigation approach that has been developed by Parties to the UNFCCC
It is a collaborative programme of FAO, UNDP and UNEP.
It supports nationally-led REDD processes and promotes the informed
and meaningful involvement of all stakeholders, including Indigenous
Peoples and other forest-dependent communities, in national and
international REDD implementation.
REDD is currently one of the most crucial and contested topics of the UN
Framework Convention on Climate Change (UNFCCC) negotiations.

289. The Green Climate Fund (GCF) is a fund within the framework of the

a) Convention on Biological diversity


b) UNFCCC
c) Montreal Convention
d) IUCN

Correct Answer : B
Answer Justification :

Learning: It is a fund within the framework of the UNFCCC founded as a


mechanism to redistribute money from the developed to the developing
world, in order to assist the developing countries in adaptation and
mitigation practices to counter climate change.
The Green Climate Fund was designated as an operating entity of the
financial mechanism of the UNFCCC, in accordance with Article 11 of the
Convention. Arrangements will be concluded between the Conference of
the Parties (COP) and the Fund to ensure that it is accountable to, and
functions under the guidance of, the COP.
The Fund is governed and supervised by a Board that will have full
responsibility for funding decisions and that receives the guidance of the
COP.

290. Peace and Biodiversity Dialogue Initiative (PBDI) is concerned with

a) Investigating, documenting, exposing and increasing awareness the causes


of biodiversity decline
b) Local waste management in biodiversity hotspots
c) Transboundary cooperation in protected areas globally
d) Funding small seed community conservation projects

Correct Answer : C
Answer Justification :

Justification: Transboundary protected areas (TBPAs) are important for


biodiversity conservation. They can also provide a focus for facilitating
cooperation and peace-building because the efficient management of
transboundary protected areas involves dialogue, cooperation, and
transparency.
This requires building participatory and multi-stakeholder management
processes, including civil society and scientists, and taking into account
changing factors and cultural circumstances.
The latest global inventory of transboundary protected areas and other
sites linked by various
transboundary conservation initiatives has been implemented by the
UNEP World Conservation Monitoring Centre (UNEP-WCMC) in 2007.
Objectives are:
Showcase the value of Peace Parks (defined below) and their conservation
benefits of biodiversity, especially how conservation also helps alleviating
conflict;
Update information on transboundary conservation complexes in the
world including in areas that could be established as Peace Parks;
Catalyze the creation of new Peace Parks and strengthen existing ones in a
number of regions, giving priority to areas declared as UNESCO World
Heritage Sites;
Organize small workshops with three or four countries to explore
opportunities of furthering their transboundary collaboration, and
possibilities, including through partners, to step up political hype;
Disseminate Best Practice Guidelines, Code of Conduct on Trans Frontiers
Conservation Complexes to facilitate their wider acceptance and
application; and
Facilitate the exchange of data and information, provide outside expertise,
capacity building as well as technical assistance for planning and
implementing the Peace Park concept.

Learning: A "Park for Peace" is a special designation that may be applied


to any of the three types of Transboundary Conservation Areas, and is
dedicated to the promotion, celebration and/or commemoration of peace
and cooperation (IUCN, 2015).

291. IUCN's World Commission on Protected Areas (WCPA) has which of the
following objectives?

1) Create a global buffer zone by inter-connecting protected areas for


improved corridor movement of migratory species
2) Bring recognition to all the protected areas of the global hotspot regions
under the UNESCO World Heritage Sites (Natural)

Which of the above is/are correct?

a) 1 only
b) 2 only
c) Both 1 and 2
d) None

Correct Answer : D
Answer Justification :

Justification: WCPA works by helping governments and others plan


protected areas and integrate them into all sectors; by providing strategic
advice to policy makers; by strengthening capacity and investment in
protected areas; and by convening the diverse constituency of protected
area stakeholders to address challenging issues.
It is administered by IUCN's Global Programme on Protected Areas and
has over 2,000 members, spanning 140 countries.
The objectives of the IUCN World Commission on Protected Areas for
2017-2020, aligned with the Promise of Sydney, are to:
Catalyze and support global efforts to expand and effectively manage
systems of protected areas to achieve the Aichi Targets for halting
biodiversity loss, in particular through meeting Aichi Target 11;
Recognise and mainstream protected areas as natural solutions to global
challenges, such as climate change, land degradation, food and water
security, health and well-being;
Make the case for investment in protected area systems, supported by
public policy, incentives, capacity development and sustainable funding;
Inspire all people, across generations, geography and cultures to
experience and value the wonder of nature through protected areas and to
promote more support for conservation of natural ecosystems;
Use our acknowledged role as a key global knowledge-broker and standard
setter for protected areas to support efforts to raise the standards and
practices of protected area governance and management globally.

292. Consider the following statements. International Ocean Discovery


Program (IODP)

1) Intends to improve scientific understanding of changing climate and


ocean conditions
2) Explores Earth's history and dynamics using ocean-going research
platforms

Which of the above is/are correct?

a) 1 only
b) 2 only
c) Both 1 and 2
d) None

Correct Answer : C
Answer Justification :

Justification: The International Ocean Discovery Program (IODP) is an


international marine research collaboration that explores Earth's history
and dynamics using ocean-going research platforms to recover data
recorded in seafloor sediments and rocks and to monitor subseafloor
environments.
IODP began in 2013 and builds on the research of four previous scientific
ocean drilling programs: Project Mohole, Deep Sea Drilling Project, Ocean
Drilling Program, and Integrated Ocean Drilling Program.
IODP depends on facilities funded by three platform providers with
financial contributions from five additional partner agencies. Together,
these entities represent twenty-three nations whose scientists are selected
to staff IODP research expeditions conducted throughout the world's
oceans.
Together, these programs represent the longest running and most
successful international Earth science collaboration
The research enabled by IODP samples and data improves scientific
understanding of changing climate and ocean conditions, the origins of
ancient life, risks posed by geohazards, and the structure and processes of
Earth’s tectonic plates and uppermost mantle.
On behalf of Ministry of Earth Sciences (MoES), National Centre for
Antarctic and Ocean Research (NCAOR), Goa has been designated as the
nodal agency to deal with various facets of the IODP program in India.

293. The Great Green Wall Initiative aims to

a) restore Africa’s degraded landscapes


b) increase blue carbon cover in tropical regions
c) manage seaweed and seagrass cover sustainably along major coasts in the
World
d) introduce green bonds in major Global exchanges such as Wall Street

Correct Answer : A
Answer Justification :

Learning: The Great Green Wall is a symbol of hope in the face of one of
the biggest challenges of our time\u2028– desertification.
Launched in 2007, this game-changing initiative aims to restore Africa’s
degraded landscapes and in the process transform millions of lives in one
of the world’s poorest regions, the Sahel.
Once complete, the Wall will be the largest living structure on the planet –
an 8000km natural wonder of the world stretching across the entire width
of the Continent.
By 2030, the ambition is to restore 100 million hectares of currently
degraded land, sequester 250 million tonnes of carbon and create a
minimum of 350,000 jobs in rural areas.
The Global Mechanism of the UNCCD is implementing a flagship initiative
under the Great Green Wall called FLEUVE – The Local Environmental
Coalition for a Green Union. The project is financed by the European
Commission to the tune of Euro 7 million and is being implemented from
2014-18.

294. Consider the following statements.

1) The Global Drylands Initiative (GDI) is an IUCN initiative to support


countries to implement their commitments to the UNCCD Convention.
2) The Global Island Partnership (GLISPA) is a voluntary partnership for all
islands, regardless of size or political status, towards building sustainable
island communities.

Which of the above is/are correct?


a) 1 only
b) 2 only
c) Both 1 and 2
d) None

Correct Answer : B
Answer Justification :

Justification: Statement 1: Drylands are found in tropical and temperate


latitudes and account for 41.3% of the global terrestrial area.
he Global Drylands Initiative supports the sustainable management of
these dryland ecosystems by adapting ecosystem management for
neglected and threatened ecosystems, restoring degraded land and
strengthening local governance for effective natural resource
management.
Specifically, the Global Drylands Initiative aims to:
Generate evidence for targeting and monitoring of dryland conditions and
trends
Strengthen governance for sustainable land management by strengthening
of resource rights and establishment of institutional mechanisms for
ecosystem management
Promote policy implementation for dryland ecosystem management by
enhancing the knowledge and capacity of various stakeholders
Statement 2: It provides a global platform that enables islands to work
together to develop solutions to common problems and to take high-level
commitments and actions that address these global challenges.
It is an IUCN initiative led by Presidents of Palau and Seychelles as well as
the Prime Minister of Grenada.

295. Biodiversity Finance Initiative (BIOFIN) is a global partnership that


helps governments cost, plan and pay for action on biodiversity
conservation and its sustainable use. The initiative is run by the

a) United Nations Development Programme (UNDP)


b) World Bank
c) International Union for Conservation of Nature (IUCN)
d) United Nations Environment Programme (UNEP)

Correct Answer : A
Answer Justification :

Learning: Available evidence and the decisions adopted by Parties to the


Convention on Biological Diversity (CBD) indicate that a significant gap
remains in finance for biodiversity management, for countries to
drastically scale up their efforts and achieve the 20 Aichi Targets defined
in the CBD’s Strategic Plan for 2011-2020.
In this context, UNDP in 2012 launched the Biodiversity Finance Initiative
– BIOFIN, as a new global partnership seeking to address the biodiversity
finance challenge in a comprehensive manner
– building a sound business case for increased investment in the
management of ecosystems and biodiversity, with a particular focus on the
needs and transformational opportunities at the national level.
BIOFIN is managed by the UNDP Ecosystems and Biodiversity
Programme, in partnership with the European Union and the
Governments of Germany and Switzerland.
296. Global Tiger recovery programme (GTRP) is an initiative of

a) UNEP
b) IUCN
c) WWF
d) Conservation International

Correct Answer : C
Answer Justification :

Learning: The overarching goal adopted in the Hua Hin declaration and
supported by the Global Tiger Recovery Program (GTRP) is to reverse the
rapid decline of wild tigers and to strive to double the number of wild
tigers across their range by 2022.
The Tiger Range Country's (TRCs) as a group is ready to take on this
challenge although not all
TRCs individually will be able to achieve this goal.
To solve the tiger crisis, which represents the larger Asian biodiversity
crisis, the TRCs, international organizations, and civil society have come
together on a collaborative platform within the framework of the Global
Tiger Initiative (GTI).
After a two-year process of sharing knowledge and best practices and
developing a common vision, the GTRP was developed, with the shared
goal of doubling the number of wild tigers globally by 2022 through
actions to:
effectively manage, preserve, protect, and enhance tiger habitats;
eradicate poaching, smuggling, and illegal trade of tigers, their parts, and
derivatives; cooperate in transboundary landscape management and in
combating illegal trade; engage with indigenous and local communities;
increase the effectiveness of tiger and habitat management; and restore
tigers to their former range.

297. Consider the following about Global Wildlife Program (GWP).

1) It was launched by the Global Environment Facility (GEF).


2) It is a World-Bank led global partnership.
3) It promotes wildlife conservation by combatting illicit trafficking in
wildlife.
4) India will host the GWP for the year 2017.

Select the correct answer using the codes below.

a) and 3 only
b) 1 and 4 only
c) 2 and 4 only
d) 1, 2, 3 and 4

Correct Answer : D
Answer Justification :

Justification: To respond to the growing crisis and international call for


action, the GEF in 2015 launched the “Global Partnership on Wildlife
Conservation and Crime Prevention for Sustainable Development”
program also known as the GWP.
The GWP is a World-Bank led global partnership that promotes wildlife
conservation and sustainable development by combatting illicit trafficking
in wildlife.
This seven-year, $131 million grant program is expected to leverage an
additional $704 million in additional co-financing from a wide range of
partners to promote investments across Africa and Asia.

ENVIRONMENTAL ORGANIZATIONS

298. Consider the following statements.

1) The South Asia Co-operative Environment Programme (SACEP) is an


inter-governmental organization of which India is a member.
2) The South Asian Seas Programme (SASP) is a part of the global Regional
Seas Programme established under the auspices of UN Environment
Programme (UNEP).

Which of the above is/are correct?

a) 1 only
b) 2 only
c) Both 1 and 2
d) None

Correct Answer : C
Answer Justification :

Justification: Statement 1: It was established in 1982 by the South Asian


governments to promote and support protection, management and
enhancement of the environment in the region.

Environmental degradation caused by factors like poverty, over


population, over consumption and wasteful production threatening
economic development and human survival can be corrected by co-
operative action in the South Asian region where many ecological and
development problems transcend national and administrative boundaries.

SACEP member countries are Afghanistan , Bangladesh , Bhutan , India ,


Maldives , Nepal , Pakistan and Sri Lanka.

Statement 2: SASP is a regional agreement – formally adopted in 1995 –


among the five maritime countries of South Asia sharing the Indian
Ocean.

SASP is part of the global Regional Seas Programme established under the
auspices of UN Environment.

SACEP also serves as the secretariat of South Asian Seas Programme


(SASP).

Learning: Launched in 1974 by UNEP, the main aim of the Regional Seas
Programme is to address the degradation of the seas by engaging
neighbouring countries in a collaboration to support the achievement of
international environmental and development targets (e.g. the Aichi
Biodiversity Targets and the Millennium Development Goals).

The UNEP Regional Seas Programme now supports more than 140
countries in marine resource management through their participation as
Contracting Parties to 18 regional level programmes around the world.

299. Consider the following funds under UNFCCC and their parent
management fund.

1) Special Climate Change Fund (SCCF): Green Climate Fund (GCF)


2) Adaptation Fund (AF): Green Environment Facility (GEF)
3) Least Developed Countries Fund (LDCF): Kyoto Protocol

Select the correct answer using the codes below.

a) 1 only
b) 2 and 3 only
c) 1 and 2 only
d) 1 and 3 only

Correct Answer : A
Answer Justification :

Background: The UNFCCC, under its Article 11, states that the operation
of the Financial Mechanism is entrusted to one or more existing
international entities.
The operation of the Financial Mechanism is partly entrusted to the
Global Environment Facility (GEF).
At COP 17 Parties decided to designate the Green Climate Fund (GCF) as
an operating entity of the Financial Mechanism of the Convention, in
accordance with Article 11 of the Convention.
The Financial Mechanism is accountable to the COP, which decides on its
climate change policies, programme priorities and eligibility criteria for
funding.
The Kyoto Protocol also recognizes, under its Article 11, the need for the
Financial Mechanism to fund activities by developing country Parties.
Justification: In addition to providing guidance to the GEF, Parties have
established four special funds: the Special Climate Change Fund (SCCF),
the Least Developed Countries Fund (LDCF), both managed by the GEF,
and the GCF under the Convention; and the Adaptation Fund (AF) under
the Kyoto Protocol.
The Adaptation Fund (AF) was established in 2001 to finance concrete
adaptation projects and programmes in developing country Parties to the
Kyoto Protocol that are particularly vulnerable to the adverse effects of
climate change.
The Adaptation Fund is financed with a share of proceeds from the clean
development mechanism (CDM) project activities and other sources of
funding. The share of proceeds amounts to 2 per cent of certified emission
reductions (CERs) issued for a CDM project activity
300. United Nations High-level Political Forum on Sustainable Development
(HLPF) was established as a result of the mandate in

a) Rio 20 Conference, 2012


b) UN Conference on the Human Environment (Stockholm 1972)
c) Convention on Biological Diversity (1992)
d) Paris Conference, 2014

Correct Answer : A
Answer Justification :

Learning: In order to ensure compliance to the agreements at Rio


(particularly the Rio Declaration on Environment and Development and
Agenda 21), delegates to the Earth Summit established the Commission on
Sustainable Development (CSD).
In 2013, the CSD was replaced by the High-level Political Forum on
Sustainable Development that meets every year as part of the ECOSOC
meetings, and every fourth year as part of the General Assembly meetings.
The Forum meets annually under the auspices of the Economic and Social
Council for eight days, including a three-day ministerial segment and
every four years at the level of Heads of State and Government under the
auspices of the General Assembly for two days.
The HLPF is the main United Nations platform on sustainable
development and it has a central role in the follow-up and review of the
2030 Agenda for Sustainable Development the Sustainable Development
Goals (SDGs) at the global level.

301. Which of the following issues are covered by the United Nations
Convention on the Law of Seas (UNCLOS)?

1) Laws and regulations of the coastal State relating to innocent passage


2) Sea lanes and traffic separation schemes in the territorial sea
3) Immunities of warships and other government ships operated for non-
commercial purposes
4) Submarines and other underwater vehicles

Select the correct answer using the codes below.

a) 1 and 4 only
b) 2 and 3 only
c) 1 and 3 only
d) 1, 2, 3 and 4

Correct Answer : D
Answer Justification :

Justification: The Law of the Sea Convention defines the rights and
responsibilities of nations with respect to their use of the world's oceans,
establishing guidelines for businesses, the environment, and the
management of marine natural resources.
These are some of the important articles of the UNCLOS:
Article 17. Right of innocent passage
Article 18. Meaning of passage
Article 19. Meaning of innocent passage
Article 20. Submarines and other underwater vehicles
Article 21. Laws and regulations of the coastal State relating to innocent
passage
Article 22. Sea lanes and traffic separation schemes in the territorial sea
Article 31. Responsibility of the flag State for damage caused by a warship
or other government ship operated for non-commercial purposes
Article 32. Immunities of warships and other government ships operated
for non-commercial purposes
The UN has no direct operational role in the implementation of the
Convention.
There is, however, a role played by organizations such as the International
Maritime Organization, the International Whaling Commission, and the
International Seabed Authority (ISA). (The ISA was established by the UN
Convention.)

302. The first World Parliamentary Forum on Sustainable Development was


hosted by

a) United Nations Environment Programme (UNEP)


b) World Economic Forum (WEP)
c) Indonesian House of Representatives
d) Senate of the United States of America

Correct Answer : C
Answer Justification :
Learning: Indonesian House of Representatives hosted the first world
Parliamentary Forum on Sustainable Development on 6-7 September 2017
in Bali.
The forum was open to all national parliaments.
The Forum will carry out the vision to facilitate the sharing of policy
analysis, experience, best practices from cross-country and cross-sectors,
discuss preparedness and also deepen the understanding of parliament in
each country of SDGs.

303. The Global Environment Facility (GEF) serves as financial mechanism


for the following conventions/protocols

1) UN Convention to Combat Desertification (UNCCD)


2) Stockholm Convention on Persistent Organic Pollutants (POPs)
3) Minamata Convention on Mercury

Select the correct answer using the codes below.

a) 1 and 2 only
b) 2 and 3 only
c) 1, 2 and 3
d) 1 only

Correct Answer : C
Answer Justification :

Learning: An independently operating financial organization (with 183


countries), the GEF provides grants for projects related to biodiversity,
climate change, international waters, land degradation, the ozone layer,
and persistent organic pollutants. It was established in 1992 on the eve of
Rio Summit.
The GEF also serves as financial mechanism for the following
conventions:
Convention on Biological Diversity (CBD)
United Nations Framework Convention on Climate Change (UNFCCC)
UN Convention to Combat Desertification (UNCCD)
Stockholm Convention on Persistent Organic Pollutants (POPs)
Minamata Convention on Mercury
The GEF, although not linked formally to the Montreal Protocol on
Substances that Deplete the Ozone Layer (MP), supports implementation
of the Protocol in countries with economies in transition.
The GEF also manages two separate adaptation-focused funds under the
UNFCCC— the Least Developed Countries Fund and the Special Climate
Change Fund— which mobilize funding specifically earmarked for
activities related to adaptation.

304. The Intergovernmental Panel on Climate Change (IPCC) is a scientific


and intergovernmental body under the

a) Auspices of the United Nations


b) Conservation International NGO
c) World Wildlife Fund (WWF)
d) Climate division of IUCN
Correct Answer : A
Answer Justification :

Learning: It was set up at the request of member governments,


dedicated to the task of providing the world with an objective, scientific
view of climate change and its political and economic impacts.
It was first established in 1988 by two United Nations organizations, the
World Meteorological Organization (WMO) and the United Nations
Environment Programme (UNEP), and later endorsed by the United
Nations General Assembly.
Membership of the IPCC is open to all members of the WMO and UNEP.
The IPCC produces reports that support the United Nations Framework
Convention on Climate Change (UNFCCC), which is the main
international treaty on climate change

305. With reference to the Conference of Parties (COP), consider the


following statements.

1) It is the supreme decision-making body of the UNFCCC.


2) All States that are Parties to the Convention are represented at the COP.
3) COP reviews the implementation of the Convention and any other legal
instruments that the COP adopts.

Select the correct answer using the codes below.

a) 1 only
b) 2 and 3 only
c) 1 and 3 only
d) 1, 2 and 3

Correct Answer : D
Answer Justification :

Justification: A key task for the COP is to review the national


communications and emission inventories submitted by Parties.
Based on this information, the COP assesses the effects of the measures
taken by Parties and the progress made in achieving the ultimate objective
of the Convention.
The COP meets every year, unless the Parties decide otherwise.

306. As per IUCN, what are the defining characteristics of Indigenous and
Community Conserved Areas (ICCAs)?

1) A community is closely connected to this well-defined ecosystem.


2) The area is untouched by human civilization other than the local
indigenous communities.
3) The community is the major player in decision-making (governance) and
implementation regarding the management of the site.

Select the correct answer using the codes below.


a) 1 and 3 only
b) 2 only
c) 2 and 3 only
d) 1 only

Correct Answer : A
Answer Justification :

Justification: ICCAs are natural and/or modified ecosystems containing


significant biodiversity values, ecological services and cultural values,
voluntarily conserved by Indigenous peoples and local communities, both
sedentary and mobile, through customary laws or other effective means.
Statement 2: Several of them are inviolate zones ranging from very small
to large stretches of land and waterscapes. But, not all of them are such
zones.
ICCAs can include ecosystems with minimum to substantial human
influence as well as cases of continuation, revival or modification of
traditional practices or new initiatives taken up by communities in the face
of new threats or opportunities.
Learning: Similarly, in 2004, WWF-India introduced the concept of a
Community Conserved Area (CCA) to empower local communities to
become active decision-makers and implement conservation initiatives.
Local communities in these CCAs form committees to undertake wildlife
monitoring, patrolling and community-based tourism activities.

307. IFAW (International Fund for Animal Welfare)

1) is a non-profit organization
2) provides hands-on assistance to animals
3) campaigns to end commercial whaling
4) was founded by the World Wildlife Fund (WWF)

Select the correct answer using the codes below.


a) 1 and 4 only
b) 2 and 3 only
c) 1, 2 and 3 only
d) 2 and 4 only
Correct Answer : C

Answer Justification :

Justification: The International Fund for Animal Welfare


(IFAW) is one of the largest animal welfare and conservation
charities in the world.

The group's declared mission is to "rescue and protect animals


around the world.

The International Fund for Animal Welfare (IFAW) was founded


by a small group of people in 1969, to stop the commercial hunt
for seal pups in Canada.
The International Tiger Day (also known as Global Tiger Day) is
celebrated every year on 29 July to raise awareness for tiger
conservation.

The IFAW recently organized major events on International Tiger


conservation day.

308. The Clean Energy Ministerial (CEM) is

1) A high-level global forum to share lessons learnt and best practices for
the transition to a global clean energy economy
2) An initiative of the World Trade Organization (WTO)
3) Financing Green Carbon Credits (GCC) allotted to Least Developed
Countries (LDCs)
4) The main implementing agency for the programs of the United Nations
Environment Programme (UNEP)

Select the correct answer using the codes below.


a) 1, 2 and 3 only
b) 2 and 4 only
c) 1 only
d) 2, 3 and 4 only
Correct Answer : C

Answer Justification :

Justification: Statement 2: The CEM is the initiative of the USA.


Presently, 23 countries are members of CEM. There are 13 initiatives
under CEM covering energy efficiency, clean energy supply and cross
cutting areas.

Statement 3: It does not finance carbon credits.

Concept: A carbon credit is a generic term for any tradable certificate or


permit representing the right to emit one tonne of carbon dioxide or
the mass of another greenhouse gas with a carbon dioxide equivalent
(tCO2e) equivalent to one tonne of carbon dioxide.
309. The main foci of the World Climate Research Programme (WCRP)
are

1) Observing changes in the components of the Earth system


2) Developing and improving numerical models that are capable of
simulating and assessing the climate system
3) Investigating the sensitivity of the climate system to natural and
human-induced forces

Select the correct answer using the codes below.


a) 2 and 3 only
b) 1 only
c) 1 and 3 only
d) 1, 2 and 3
Correct Answer : D

Answer Justification :
Learning: WCRP’s mission is to facilitate the analysis and
prediction of Earth system variability and change for use in an
increasing range of practical applications of direct relevance,
benefit and value to society.

WCRP research is centred on:

observing changes in the atmosphere, oceans, land and


cryosphere and in the interfaces between these components;

improving our knowledge and understanding of global and


regional climate variability and change, and of the mechanisms
responsible for this change;

assessing and attributing significant trends in global and


regional climates;

developing climate models for a wide range of space and time


scales; and

investigating the sensitivity of the climate system to natural and


human-induced forcing and estimating the changes resulting
from specific disturbing influences.
INTERNATIONAL ENVIRONMENTAL CONVENTIONS

310. The Clearing-House Mechanism (CHM) of the Convention on Biological


Diversity

a) provides effective global information services to facilitate the


implementation of the Strategic Plan for Biodiversity
b) reduces the impact of invasion of exotic species by providing technical and
financial assistance to developing economies for their ejection
c) facilitates the cleaning of the Ballast water brought mainly by ships
engaged in international trade
d) ensures the sharing of genetic resources between and across state parties

Correct Answer : A
Answer Justification :

Learning: The Clearing-House Mechanism consists of the following


components:

The CBD website, acting as the central node.

The network of national Clearing-House Mechanisms. Various partner


institutions.

It was established to further an Article of the CBD.

This is in order to promote and facilitate scientific and technical


cooperation, knowledge sharing and information exchange, and to
establish a fully operational network of Parties and partners.

The central clearing-house mechanism provides effective global


information services to facilitate the implementation of the Strategic Plan
for Biodiversity 2011-2020 and the achievement of the Aichi Biodiversity
Targets.

Partners significantly expand the clearing-house mechanism network and


services.

311. International Plant Protection Convention (IPPC)

1) Provides the mechanisms to develop the International Standards for


Phytosanitary Measures
2) Protects only plants that are cultivated on an agricultural land
3) IPPC strategic objectives primarily relate to the work under some of the
FAO Strategic Objectives

Select the correct answer using the codes below.

a) 1 and 2 only
b) 1 and 2 only
c) 1 and 2 only
d) 1, 2 and 3

Correct Answer : C
Answer Justification :

Justification: Statement 1 and 2: The IPPC aims to protect world plant


resources, including cultivated and wild plants by preventing the
introduction and spread of plant pests and promoting the appropriate
measures for their control.

The convention provides the mechanisms to develop the International


Standards for Phytosanitary Measures (ISPMs), and to help countries to
implement the ISPMs and the other obligations under the IPPC, by
facilitating the national capacity development, national reporting and
dispute settlement.

The Secretariat of the IPPC is hosted by the Food and Agriculture


Organization of the United Nations (FAO).

Statement 3: These IPPC strategic objectives primarily relate to the work


under the FAO Strategic Objectives 2 (sustainable production) and 4
(trade facilitation).

While the IPPC's primary focus is on plants and plant products moving in
international trade, the Convention also covers research materials,
biological control organisms, germplasm banks, containment facilities,
food aid, emergency aid and anything else that can act as a vector for the
spread of plant pests – for example, containers, packaging materials, soil,
vehicles, vessels and machinery.

312. The Convention on the Conservation of Migratory Species of Wild


Animals is the only global convention specializing in the conservation of
migratory species, their habitats and migration routes. It was signed
under the aegis of

a) World Wildlife Network (WWN)


b) United Nations Environment Programme (UNEP)
c) Conservation International
d) International Union for Conservation of Nature (IUCN)

Correct Answer : B

Learning: As an environmental treaty under the aegis of the United


Nations Environment Programme, CMS provides a global platform for the
conservation and sustainable use of migratory animals and their habitats.

CMS brings together the States through which migratory animals pass, the
Range States, and lays the legal foundation for internationally coordinated
conservation measures throughout a migratory range. It also coordinates
with NGOs and other non-state actors.

Migratory species threatened with extinction are listed on Appendix I of


the Convention.
CMS acts as a framework Convention. The agreements may range from
legally binding treaties (called Agreements) to less formal instruments,
such as Memoranda of Understanding, and can be adapted to the
requirements of particular regions.

313. Aichi Biodiversity Targets are a set of 20 global targets under the

a) Strategic Plan for Biodiversity 2011-2020


b) Sustainable Development Goals (SDGs)
c) UNFCCC Climate Commitments
d) UNEP Plan for Global Action Correct Answer : A

Answer Justification :

Learning: They are grouped under five strategic goals:


Address the underlying causes of biodiversity loss by mainstreaming
biodiversity across
government and society.
Reduce the direct pressures on biodiversity and promote sustainable use.

Improve the status of biodiversity by safeguarding ecosystems, species and


genetic diversity.
Enhance the benefits to all from biodiversity and ecosystem services.
Enhance implementation through participatory planning, knowledge
management and capacity building.

The above information comes from the Secretariat of the Convention on


Biological Diversity (CBD Secretariat) (2014).
314. MARPOL Convention is related to

a) Prevention of Pollution from Ships


b) International Civil Aviation Protocol
c) Transnational mining regulation
d) Use of water of transnational rivers

Correct Answer : A
Answer Justification :

Learning: It is also known as International Convention for the


Prevention of Pollution from Ships (MARPOL Convention).
It is the main international convention that deals with this subject. It was
adopted in 1973 and entered into force in 1983.
Its biodiversity relevance stems from the highly detrimental effect that
pollution can have on marine organisms, therefore by preventing the
pollution of the sea the Convention contributes towards the maintenance
of marine biodiversity.

315. Access and Benefit-sharing (ABS) is a key idea in which of the following
environmental conventions?
a) Convention on Biological Diversity
b) CITES Convention
c) United Nations Framework Convention on Climate Change (UNFCC)
d) Washington Convention, 2001

Correct Answer : A
Answer Justification :

Learning: The Nagoya Protocol on access to genetic resources and the


fair and equitable sharing of benefits arising from their utilisation is a
supplementary agreement to the Convention on Biological Diversity,
which provides a transparent legal framework for the effective
implementation of one of the three objectives of the CBD: the fair and
equitable sharing of benefits arising out of the utilization of genetic
resources.
The ABS Clearing-House is a key tool for facilitating the implementation
of the Nagoya Protocol, by enhancing legal certainty and transparency on
procedures for access and benefit-sharing, and for monitoring the
utilization of genetic resources along the value chain, including through
the internationally recognized certificate of compliance.

316. The targets for the first commitment period of the Kyoto Protocol cover
emissions of the six main greenhouse gases. It did NOT include which of
these gases?

a) Methane (CH4);
b) Nitrous oxide (N2O);
c) Sulphur hexafluoride (SF6)
d) Carbon Monooxide (CO)

Correct Answer : D
Answer Justification :
Justification: CO is not a greenhouse gas, so not covered under Kyoto
protocol. Other gases that were covered are:
Hydrofluorocarbons (HFCs);
Perfluorocarbons (PFCs); and
Carbon dioxide (CO2);
During the first commitment period, 37 industrialized countries and the
European Community committed to reduce GHG emissions to an average
of five percent against 1990 levels. During the second commitment period,
Parties committed to reduce GHG emissions by at least 18 percent below
1990 levels in the eight-year period from 2013 to 2020; however, the
composition of Parties in the second commitment period is different from
the first.

317. Action for Climate Empowerment (ACE) is a term adopted in which of


following important climate change mitigation
treaties/protocols/conventions?

a) Montreal Protocol
b) Convention on Biological Diversity
c) UNFCCC
d) Paris convention 2015

Correct Answer : C
Answer Justification :

Learning: It is a part of UNFCCC and refers to Article 6 of the


Convention’s original text (1992), focusing on six priority areas: education,
training, public awareness, public participation, public access to
information, and international cooperation on these issues.
The implementation of all six areas has been identified as the pivotal
factor for everyone to understand and participate in solving the complex
challenges presented by climate change.
ACE calls on governments to develop and implement educational and
public awareness programmes, train scientific, technical and managerial
personnel, foster access to information, and promote public participation
in addressing climate change and its effects.
It also urges countries to cooperate in this process, by exchanging good
practices and lessons learned, and strengthening national institutions.

318. The Union cabinet has approved a proposal for ratification of the
Minamata Convention.Consider the following about it.

1) It is a global treaty to protect human health and the environment from


adverse effects of mercury.
2) After joining the Convention, it will be easier for India to get technological
or financial assistance to address Mercury related issues.

Which of the above is/are correct?

a) 1 only
b) 2 only
c) Both 1 and 2
d) None
Correct Answer : C
Answer Justification :

Justification: Statement 1: The Minamata Convention on Mercury is a


multilateral environmental agreement that addresses specific human
activities which are contributing to widespread mercury pollution and
eventually reduce its harmful effects.

Statement 2: These are the obligations on Parties of Convention:

Ban on new mercury mine and the phase-out of existing ones

Phase out and phase down of mercury use in a number of products and
processes Control measures on emissions to air and on releases to land
and water Regulation of the informal sector of artisanal and small-scale
gold mining etc.

In return, parties get enhanced assistance to tackle these issues

319. The term 'Bali Road Map, is sometimes seen in news in the context of

a) Intellectual Property Rights


b) Organize crime
c) Wildlife Trade
d) Climate change

Correct Answer : D
Answer Justification :

Learning: The Bali Road Map was adopted at the 13th Conference of the
Parties and the 3rd Meeting of the Parties in 2007 in Bali.
The Road Map is a set of a forward-looking decisions that represent the
work that needs to be done under various negotiating “tracks” that is
essential to reaching a secure climate future.
The Bali Road Map includes the Bali Action Plan, which charts the course
for a new negotiating process designed to tackle climate change.

320. The Rotterdam Convention concerns with

a) Nuclear disarmament
b) Hazardous chemicals
c) Bio-piracy
d) Genetic Engineering

Correct Answer : B
Answer Justification :

Learning: Formally called the Rotterdam Convention on the Prior


Informed Consent Procedure for Certain Hazardous Chemicals and
Pesticides in International Trade), it is a multilateral treaty to promote
shared responsibilities in relation to importation of hazardous chemicals.
The convention promotes open exchange of information and calls on
exporters of hazardous chemicals to use proper labeling, include
directions on safe handling, and inform purchasers of any known
restrictions or bans. Signatory nations can decide whether to allow or ban
the importation of chemicals listed in the treaty, and exporting countries
are obliged to make sure that producers within their jurisdiction comply.

321. Consider the following about the Convention on the Conservation of


Migratory Species of Wild Animals.

1) India is a party to the convention.


2) It was concluded under the WWF-IUCN joint forum.
3) It is the successor of the global convention specializing in the conservation
of migratory species that was established first during 1992 Earth Summit.

Select the correct answer using the codes below.

a) 1 only
b) 1 and 3 only
c) 2 and 3 only
d) 1 and 2 only

Correct Answer : A
Answer Justification :

Learning: The Convention was signed in 1979 in Bad Godesberg, a


suburb of Bonn (hence also the name Bonn Convention), and entered into
force in 1983.

CMS and its daughter agreements determine policy and provide further
guidance on specific issues through their Strategic Plans, Action Plans,
resolutions, decisions and guidelines.

CMS acts as a framework Convention. The Agreements may range from


legally binding treaties (called Agreements) to less formal instruments,
such as Memoranda of Understanding.

Several Agreements have been concluded to date under the auspices of


CMS such as ‘Raptor MoU’ for conservation in Africa and Eurasia.

322. With reference to the Convention on the Conservation of Migratory


Species of Wild Animals (CMS), consider the following.

1) It is also known as the Washington convention.


2) It was signed under the aegis of United Nations Environment Programme
(UNEP).
3) India will host next Convention on CMS in 2018.

Select the correct answer using the codes below.


a) Only
b) 1 and 3 only
c) 2 and 3 only
d) 1 only

Correct Answer : A
Answer Justification :

Justification: Statement 1: CMS COP is also known as a Global Wildlife


Conference.
CMS is an international treaty concluded under aegis of United Nations
Environment Programme (UNEP), concerned with conservation of
wildlife and habitats on a global scale.
It is commonly abbreviated as Convention on Migratory Species (CMS) or
the Bonn Convention.
CMS aims to conserve terrestrial, marine and avian migratory species
throughout their range.
Statement 3: UNEP announced that India will host next Convention on
the CMS Conference of Parties 13 (CMS COP13) in year 2020.

323. With reference to the Convention on Biological Diversity (CBD),


consider the following.

Each contracting Party shall, as far as possible and as appropriate

1) Preserve and maintain knowledge and practices of indigenous


communities with regards to sustainable development.
2) Maintain privacy and a local patent database with regards to innovative
practices of local communities embodying traditional lifestyles relevant
for the conservation of biological diversity

Which of the above is/are correct?

a) 1 only
b) 2 only
c) Both 1 and 2
d) None

Correct Answer : A
Answer Justification :

Justification: As per the CBD, article 8(j) states that subject to national
legislation, parties shall respect, preserve and maintain knowledge,
innovations and practices of indigenous and local communities
embodying traditional lifestyles relevant for the conservation and
sustainable use of biological diversity and promote their wider application
with the approval and involvement of the holders of such knowledge,
innovations and practices and encourage the equitable sharing of the
benefits arising from the utilization of such knowledge innovations and
practices.
Nagoya protocol is a visible manifestation of this provision, which we have
already covered in previous tests.
324. The Rio Conventions which derive directly from the 1992 Earth Summit
do NOT include

a) United Nations Framework Convention on Climate Change (UNFCCC)


b) Convention on Biological Diversity
c) United Nations Convention to Combat Desertification
d) Convention concerning the Protection of the World Cultural and Natural
Heritage

Correct Answer : D
Answer Justification :

Learning: The three Rio Conventions—on Biodiversity, Climate Change


and Desertification—derive directly from the 1992 Earth Summit. Each
instrument represents a way of contributing to the sustainable
development goals of Agenda 21. The three
conventions are intrinsically linked, operating in the same ecosystems
and addressing interdependent issues.
The objectives of the CBD are the conservation of biological diversity, the
sustainable use of its components, and the fair and equitable sharing of
the benefits arising from commercial and other utilization of genetic
resources.
The UNFCCC sets an overall framework for intergovernmental efforts to
tackle the challenge posed by climate change.
The UNCCD aims to combat desertification and mitigate the effects of
drought in countries experiencing serious drought and/or desertification,
particularly in Africa, through effective actions at all levels, supported by
international co-operation and partnership arrangements, in the
framework of an integrated approach which is consistent with Agenda 21,
with a view to contributing to the achievements of sustainable
development in affected areas.

325. Consider the following about Convention on Conservation of Migratory


Species (CMS).

1) It is also known as Bonn Convention.


2) It was concluded under the aegis of the United Nations Environment
Programme (UNEP).
3) India is a party to the CMS.
4) It covers only terrestrial and avian migratory species throughout their
range.

Select the correct answer using the codes below.

a) 1 and 4 only
b) 2 and 3 only
c) 1, 2 and 3 only
d) 1, 2, 3 and 4

Correct Answer : C
Answer Justification :
Learning: The CMS is the only global and UN-based intergovernmental
organization established exclusively for the conservation and management
of terrestrial, aquatic and avian migratory species throughout their range.
It is an intergovernmental treaty, concluded under the aegis of the United
Nations Environment Programme, concerned with the conservation of
wildlife and habitats on a global scale.
Since the Convention's entry into force, its membership has grown steadily
to include over 100 Parties from Africa, Central and South America, Asia,
Europe and Oceania.

326. Consider the following statements. India Biosafety Clearing House


(BCH)

1) Was setup in accordance with the provisions of the Nagoya Protocol


2) is an information exchange mechanism established to assist Parties to
share information and experience with Living Modified Organisms
(LMOs)

Which of the above is/are correct?

a) 1 only
b) 2 only
c) Both 1 and 2
d) None

Correct Answer : B
Answer Justification :

Justification: India is a signatory to the Cartagena Protocol on Biosafety


and ratified it in 2003.
As per Article 20 of the Cartagena Protocol on Biosafety, requires set up of
BCH in order to facilitate the exchange of scientific, technical
environmental and legal information on living modified organisms
(LMOs).
The Biosafety Clearing House (BCH) is an information exchange
mechanism established to assist Parties to implement its provisions and to
facilitate sharing of information on, and experience with, LMOs.
One of the major objectives of projects going under BCH is to strengthen
the biosafety management system in India to ensure adequate protection
of human health and biodiversity from potential
harms arising from all living modified organisms (LMOs) related activities
in agriculture.

327. Rotterdam Convention concerns with the Prior Informed Consent


Procedure for

a) Public Participation in Decision-making and Access to Justice in


Environmental Matters
b) Certain Hazardous Chemicals and Pesticides in International Trade
c) Transboundary movement of shipping goods and related services
d) Conservation of Marine Living Resources

Correct Answer : B
Answer Justification :

Learning: It is a United Nations treaty multilateral treaty signed in 1998


and came into effect in
2004.
It covers pesticides and industrial chemicals that have been banned or
severely restricted for health or environmental reasons by Parties and
which have been notified by Parties for inclusion in the PIC procedure
Convention creates legally binding obligations for the implementation of
the Prior Informed Consent (PIC) procedure
It built on the voluntary PIC procedure, initiated by UNEP and FAO in
1989 and ceased in 2006.

328. Global Wastewater Initiative (GWI) was established by the

a) IUCN
b) WWF
c) UNEP
d) Ramsar Convention

Correct Answer : C
Answer Justification :

Justification: The Initiative promotes good wastewater management


practices and works towards having wastewater viewed as a potentially
valuable resource instead of as a waste product.
The GW2I is a global multiple stakeholder platform comprised of UN
agencies, international organizations, governments, scientists, private
sectors and major groups and stakeholders to provide the foundations for
partnerships to initiate comprehensive, effective and sustained
programmes addressing wastewater management.
It is a voluntary network of stakeholders with an international Steering
Committee and a Secretariat to be provided by UNEP/GPA.

329. CORDEX is a

a) Regional climate change modelling programme


b) Food standardization programme
c) Vaccination programme run by UNICEF
d) Internet sea cable laying programme of the WTU
Correct Answer : A

Answer Justification :
Learning: Coordinated Regional Climate Downscaling Experiment
(CORDEX) is a World Climate Research Programme (WCRP) sponsored
program to produce regional climate change scenarios globally,
contributing to the IPCC Assessment reports and to the climate
community.

CORDEX will produce an ensemble of multiple dynamical and


statistical downscaling models (regional details).
Option B: Codex Alimentarius Commission is related to food standards,
not CORDEX.

Need for CORDEX: A Global Climate Model (GCM) can provide


reliable prediction information on scales of around 1000 by 1000km
covering what could be a vastly differing landscape (from very
mountainous to flat coastal plains for example) with greatly varying
potential for floods, droughts or other extreme events.

Regional Climate Models (RCM) applied over a limited area and driven
by GCMs can provide information on much smaller scales supporting
more detailed impact and adaptation assessment and planning, which
is vital in many vulnerable regions of the world.

330. The 2030 Water Resources Group is

1) A inter-governmental forum founded by United Nations Environment


Programme (UNEP)
2) Hosted by the International Finance Corporation (IFC)
3) Focusing on water resources reform in water stressed countries

Select the correct answer using the codes below.


a) 1 only
b) 2 and 3 only
c) 3 only
d) 1, 2 and 3
Correct Answer : B

Answer Justification :

Justification: Statement 1 and 3: It is a unique public-private-civil


society collaboration.

The 2030 WRG was launched in 2008 at the World Economic Forum
and has been hosted by IFC since 2012.

Statement 2: It facilitates open, trust-based dialogue processes to drive


action on water resources reform in water stressed countries in developing
economies.

The ultimate aim of such reforms and actions is to close the gap between
water demand and supply by the year 2030.
Our global partners include companies like PepsiCo, Coca-Cola,
development banks the World Bank, African Development Bank and
INGOs and IGPs like UNDP, IUCN.
331. Consider the following with reference to major international dates
related to environment.

1) The International Day for Biological Diversity was initiated by the


United Nations General Assembly (UNGA).
2) Host nation for World Environment Day (WED) 2017 was India.
3) Earth Hour is celebrated at noon local time to acknowledge the
importance of the solar cycle.
4) World Wetlands Day marks the date of the adoption of the Ramsar
Convention on Wetlands.

Select the correct answer using the codes below.


a) 1 and 2 only
b) 2, 3 and 4 only
c) 1 and 4 only
d) 1, 2, 3 and 4
Correct Answer : C

Answer Justification :

Justification: Statement 1: It is a United Nations–sanctioned


international day for the promotion of biodiversity issues. It is currently
held on May 22.

Statement 2: The host nation is Canada. The theme for 2017 is


'Connecting People to Nature – in
the city and on the land, from the poles to the equator'.

Statement 3: Earth Hour is a worldwide movement organized by the


World Wide Fund for Nature (WWF). It is celebrated at 8:30pm (LT).

332. Friends of the Earth International (FoEI)

1) is an international network of environmental organizations


2) was founded as an anti-nuclear group
3) picks up causes of economic justice and supports human rights

Select the correct answer using the codes below.


a) Only
b) 1 and 3 only
c) 1 only
d) 1, 2 and 3
Correct Answer : D

Answer Justification :
Justification: Operational in around seventy countries, Friends of the
Earth was founded in 1969 as an anti-nuclear group. FOTE main
mission was to lock up and prevent further development of nuclear
energy.

The current campaign priorities of Friends of the Earth internationally


are:

 economic justice and resisting neoliberalism

 forests and biodiversity

 food sovereignty

 climate justice and energy (Including releasing the song "Love Song
To the Earth")

333. With reference to climate change financing, the principle of ‘common


but differentiated responsibility’ evolved from the notion of

1) Nature as the common heritage of mankind


2) Equity in international law
3) Difference in economic and technical capacity of nations to tackle climate
change
4) Historical differences in the contributions of developed and developing
States in creating environmental problems

Select the correct answer using the codes below.


a) 1 and 4 only
b) 1 and 2 only
c) 2 and 3 only
d) 1, 2, 3 and 4
Correct Answer : D

Answer Justification :

Justification:
Statement 2: Principle of equity implies fairness. Its implication can be
seen in statement 4.
Statement 4: For e.g. USA and China have contributed more to GHG
emissions historically than LDCs like South Sudan or Zimbabwe. So, it
is only fair that the larger share of responsibilities should go to USA and
China.

Statement 3: Moreover, not only responsibilities, but also capabilities are


different. This implies that a larger burden cannot be given to LDCs or
developing countries.

So, the Rio Declaration states: “In view of the different contributions to
global environmental degradation, States have common but differentiated
responsibilities.
334. With reference to CITES (the Convention on International Trade
in Endangered Species of Wild Fauna and Flora), consider the
following:

1) It is an international agreement to which States and regional


economic integration organizations and it is binding in nature.
2) States that have agreed to be bound by the Convention ('joined'
CITES) are known as Parties.
3) CITES was drafted as a result of a resolution adopted at a meeting
of members of IUCN.
Select the correct answer using the codes below.
a) 1 only
b) 2 and 3 only
c) 3 only
d) 1, 2 and 3
Correct Answer : D

Answer Justification :

Justification: Statement 1: It is an international agreement


between governments.

Because the trade in wild animals and plants crosses borders


between countries, the effort to regulate it requires international
cooperation to safeguard certain species from over-exploitation.

Statement 2: Although CITES is legally binding on the Parties – in


other words they have to implement the Convention – it does not
take the place of national laws.

Rather it provides a framework to be respected by each Party,


which has to adopt its own domestic legislation to ensure that
CITES is implemented at the national level.

Statement 3: It was adopted in 1963.

You might also like